Unit 2 practice questions: NURS 172

¡Supera tus tareas y exámenes ahora con Quizwiz!

A patient is admitted to the emergency department in a sickle cell crisis. The nurse assesses the patient and documents the following clinical findings: temperature 102°F, O2 saturation of 89%, and complaints of severe abdominal pain. Based on the assessment findings, which intervention is the greatest priority? 1. Apply oxygen per nasal cannula at 3 L/minute. 2. Assess and document peripheral pulses. 3. Administer morphine sulfate 10 mg IM. 4. Administer Tylenol 650 mg by mouth.

1 : 1. Hypoxia is often the cause of a sickle cell crisis from the clumping of damaged red blood cells, which creates an obstruction and hypoxia distal to the clumping. Administering the oxygen will improve the pain and increase the oxygen saturation of body tissues. Therefore, applying the oxygen should be the first action by the nurse. 2. Full body assessment, including peripheral pulses, is significant to identify the location of the potential obstruction, but this is secondary to treating the hypoxia that is known to be present from the sickling of the cells during sickle cell crisis. 3. Morphine sulfate is a narcotic for pain, but it should be given after the oxygen is started because the symptoms are caused by hypoxia. The morphine will decrease the pain and decrease metabolic oxygen needs by decreasing basal metabolic rates; therefore, supply is increased and demand is increased. 4. Although the temperature is elevated and will increase oxygen demands in the body by increased basal metabolic activity, administering Tylenol is not the first action the nurse should take, because a sickle cell crisis is caused by oxygen deprivation in tissues, not by the fever.

The nurse is planning care for a patient with acute myeloid leukemia (AML). Which is the priority nursing diagnosis to minimize the risk of complications associated with this diagnosis? 1. Risk for Bleeding 2. Impaired Mobility 3. Imbalanced Nutrition 4. Fluid Volume Excess

1 : 1. Leukemia results in neutropenia (decreased neutrophils = risk of infection) and thrombocytopenia (decreased platelets, which leads to increased risk of bleeding). 2. Impaired Mobility may be relevant to the patient with leukemia, but risk of bleeding is the higher priority. 3. Impaired Nutrition may be relevant to the patient with leukemia, but the risk of bleeding is the higher priority. 4. The patient with leukemia is not at risk for fluid volume excess.

The nurse monitors for which diagnostic result in the patient with pericarditis? 1. Elevated sedimentation rate (ESR) 2. Elevated troponin 3. Elevated lipid panel 4. Elevated cholesterol

1 : 1. Positive inflammatory markers such as C-reactive protein or sedimentation rates may indicate the presence of infection or inflammation, leading to the diagnosis of pericarditis when combined with the associated clinical manifestations. 2. Elevated troponin levels are associated with myocardial injury. 3. Elevated lipid panel levels are associated with coronary artery disease and are not a risk factor for pericarditis. 4. Elevated cholesterol levels are associated with coronary artery disease and are not a risk factor for pericarditis.

An older adult patient with renal failure is diagnosed with anemia. Based on this data, which cause of anemia will the nurse include in the plan? 1. Loss of the kidney hormone erythropoietin 2. A loss of appetite related to elevated blood urea nitrogen (BUN) and creatinine levels 3. The renal dialysis used to treat the chronic renal failure 4. Loss of blood through the urine because the failing kidney does not function properly

1 : 1. The anemia associated with renal failure is related to the loss of erythropoietin, which is produced by the healthy kidney and stimulates bone marrow to produce red blood cells. 2. The anemia is not directly related to anorexia or hemodialysis, although these factors may be somewhat associated with the anemia. 3. The anemia is not directly related to anorexia or hemodialysis, although these factors may be somewhat associated with the anemia. 4. Renal failure causes the loss of protein, not blood, through the urine.

The nurse is caring for a patient with pericardial effusion. Which action should the nurse use to relieve shortness of breath and pain? 1. Elevating the head of the bed 2. Having the patient lean forward 3. Placing in the left side-lying position 4. Encouraging deep breathing and coughing

1 : 1. The head of the patient's bed should be elevated. Pericardial effusion exerts pressure on surrounding organs, resulting in orthopnea and dyspnea. Raising the head of bed relieves shortness of breath. Pain is also relieved by sitting in the upright position. 2. Having the patient lean forward assists in differentiating chest pain in pericarditis from pain secondary to myocardial infarction. The patient's shortness of breath would be further exacerbated if left in this position. 3. Elevating the head of the patient's bed, not placing the patient in the left side-lying position, relieves shortness of breath or pain. 4. Deep breathing and coughing will not relieve shortness of breath or pain.

In providing care to a patient with valvular disease, the nurse recognizes which valve disorder is most prevalent? The nurse recognizes that which valve is most often affected in valvular disease? 1. Aortic stenosis 2. Mitral stenosis 3. Pulmonic regurgitation 4. Tricuspid regurgitation

1 : 1. The most common valvular diseases are aortic stenosis and mitral regurgitation. The least commonly affected valves are the tricuspid and pulmonic valves because of the low-pressure system in the right side of the heart. Valvular disease can affect one or more valves at the same time. The most commonly affected valve is the aortic valve. 2. The mitral valve is not identified as being commonly affected by valvular disease. 3. The least commonly affected valves include the pulmonic valve because of the low pressure system in the right side of the heart. 4. The least commonly affected valves include the tricuspid valve because of the low pressure system in the right side of the heart.

Which statement by a 65-year-old patient diagnosed with aplastic anemia secondary to chemotherapy indicates the need for further teaching? 1. "I am looking forward to getting a bone marrow transplant." 2. "I will be receiving medications that stimulate my bone marrow." 3. "I may need blood transfusions." 4. "I need to get my counts up to improve energy and decrease the risk of infection."

1 : 1. This statement indicates the need for further teaching. Although the preferred treatment for aplastic anemia is bone marrow transplantation, in patients older than age 60, immunosuppressive therapy is preferred. Additionally, aplastic anemia secondary to chemotherapy and/or radiation is treated with medications that stimulate bone marrow production. 2. Treatments for aplastic anemia focus on stimulating bone marrow with medications that stimulate bone marrow production of cells and relieve symptoms. 3. During the diagnostic evaluation phase of aplastic anemia, blood transfusions are a primary treatment. The goal of treatment is to minimize transfusions to decrease sensitization that ultimately increases the risk for bone marrow transplantation rejection; however, this patient will most likely not receive a transplant because of age and the cause of aplastic anemia being secondary to chemotherapy. 4. The patient is at risk for infection, bleeding, and impaired oxygenation secondary to the aplastic anemia.

The nurse recognizes which prescribed medication is indicated to prevent the formation of clots for a patient diagnosed with deep vein thrombosis (DVT)? 1. Warfarin 2. Vitamin K 3. Abciximab 4. Streptokinase

1 : 1. Warfarin is an anticoagulant medication that is used to prevent the formation of clots. 2. Vitamin K does not prevent the formation of clots. 3. This medication is administered for an anticoagulant overdose. 4. Abciximab is a platelet inhibitor. 5. Streptokinase is a thrombolytic, or clot buster.

The nurse provides care to a patient who is admitted for a sickle cell disease (SCD) crisis. Which is the priority prescription for the nurse to implement when providing patient care? 1. Administering oxygen 2. Administering antipyretics 3. Administering pain medication 4. Administering intravenous fluid

1 : 1. When prioritizing care for a patient, the nurse often uses the ABCs (airway, breathing, circulation). Oxygen assists in decreasing hypoxia and minimizing the severity of the crisis; therefore, this is the priority nursing action. 2. When prioritizing care for a patient, the nurse often uses the ABCs (airway, breathing, circulation). Antipyretics help reduce fever if present and decrease fluid loss associated with temperature elevation and increased metabolic rate. Although this is an appropriate nursing action, this is not the priority because it addresses circulation. 3. When prioritizing care for a patient, the nurse often uses the ABCs (airway, breathing, circulation). Administering analgesia, usually opiates, helps manage the pain caused by vaso-occlusion. Although this is an appropriate nursing action, this is not the priority because it addresses circulation. 4. When prioritizing care for a patient, the nurse often uses the ABCs (airway, breathing, circulation). Aggressive hydration increases blood volume (easing clumping of debris in capillaries) and flushes the kidneys, minimizing the risk for renal failure. Although this is an appropriate nursing action, this is not the priority because it addresses circulation.

In administering an angiotensin-converting enzyme inhibitor to a patient with heart failure, the nurse correlates its effectiveness to which mechanism of action? 1. Reduce afterload 2. Decrease preload 3. Increase contractility 4. Control sympathetic nervous system response to decreased cardiac output

1 : 1. Angiotensin-converting enzyme inhibitors (ACE-Is) are usually the first line of medications used to reduce afterload. 2. Aldosterone antagonist diuretics such as spironolactone (Aldactone) as well as loop diuretics are used to decrease preload. 3. A mainstay of heart failure management in the past has been digoxin (Lanoxin), an oral positive inotropic medication used to increase cardiac contractility. 4. Beta blockers are used to control the sympathetic nervous system response to decreased cardiac output, such as tachycardia, to decrease cardiac workload.

The nurse formulates the following diagnosis: "Impaired Tissue Perfusion related to Acute inflammation and thrombosis in arteries and veins in the hands and feet." This nursing diagnosis is most relevant to which disease process? 1. Buerger's disease 2. Raynaud's disease 3. Secondary hypertension 4. Deep vein thrombosis

1 : 1. Buerger's disease is an acute inflammation and thrombosis in arteries and veins in the hands and feet. 2. Raynaud's disease is a temporary, severe vasoconstriction in the arteries of the fingers and/or toes in response to cold. 3. Secondary hypertension is characterized by elevations in blood pressure as a result of a specific cause, such as renal parenchymal disease, pheochromocytoma, Cushing's syndrome, primary aldosteronism, congenital adrenal hyperplasia, hyperthyroidism, myxedema, or coarctation of the aorta. 4. Deep vein thrombosis (DVT) is a blood clot in a large vein, usually in the leg or pelvis.

The nurse is concerned that a patient with heart failure is decompensating. Which assessment finding requires an immediate intervention? 1. Dyspnea at rest 2. Dry persistent cough 3. Weak peripheral pulses 4. Jugular vein distention

1 : 1. Dyspnea on exertion indicates deceased cardiac output and worsening heart failure; however, dyspnea at rest indicates even more cardiac decompensation. 2. A dry persistent cough is a side effect of angiotensin-converting enzyme inhibitors. 3. Weak peripheral pulses are a consequence of inadequate cardiac output and are observed in heart failure but are not as much of a priority as dyspnea. 4. Jugular vein distention is a clinical manifestation of right-sided heart failure and is important to monitor but is not the highest priority.

The nurse monitors for which clinical manifestation in the patient diagnosed with valvular disease? 1. Heart murmur 2. Splinter hemorrhages 3. Osler's nodes 4. Displaced point of maximum impulse

1 : 1. Often the first clinical sign of valvular disease is the auscultation of a murmur. 2. Splinter hemorrhages, tiny blood clots that run vertically under nails, are observed in patients with infective endocarditis. 3. Osler's nodes are red, painful nodes in the pads of the fingers and toes, and observed in patients with infective endocarditis. 4. A displaced point of maximum impulse is associated with cardiomyopathy and not valvular disease.

A patient is diagnosed with carotid artery disease. The nurse monitors the patient for which potential complication of this disease process? 1. Stroke 2. Diabetes mellitus 3. Hypertension 4. Dyslipidemia

1 : 1. The major complication of coronary artery disease is stroke. 2. Diabetes is a risk factor for the development of coronary artery disease. 3. Hypertension is a risk factor for the development of coronary artery disease. 4. Dyslipidemia is a risk factor for the development of coronary artery disease.

Which statement by the patient with Raynaud's disease indicates that teaching was effective? 1. "Mittens are better than gloves to prevent episodes." 2. "I need to decrease cigarette smoking." 3. "I will need to take blood thinners for life." 4. "I will need to limit fluid intake."

1 : 1. This statement indicates patient understanding as mittens are better at preventing finger coldness because the fingers are touching. 2. This statement indicates the need for further teaching because the patient needs to stop smoking, not decrease the numbers of cigarettes. 3. This statement indicates the need for further teaching because the anticoagulants are not indicated in the treatment of patients with peripheral arterial disease (PAD). Medication therapies target the risk factors driving the progression of atherosclerosis in PAD and include antihypertensive, antiplatelet, and statin agents. 4. This statement indicates the need for further teaching because fluid intake should not be restricted as this could further impair circulation due to hyperviscosity.

Which statement by the patient regarding the treatment for cystic fibrosis indicates the need for further teaching? 1. "Taking CFTR modulators will cure my cystic fibrosis." 2. "There are medications that will decrease the viscosity of the secretions." 3. "I may need to take vitamins to support my nutritional needs." 4. "I may also have to take pancreatic enzymes for nutritional purposes."

1 : 1. This statement indicates the need for further teaching because cystic fibrosis transmembrane conductance regulator (CFTR) modulators are medications targeting specific defects caused by mutations in the CFTR gene. These treatments do not correct errors in the CFTR gene but attempt to correct the malfunctioning protein made by the CFTR gene. It is not a cure. 2. This statement indicates teaching effectiveness because the patient with cystic fibrosis may take mucolytics to thin the mucus. 3. This statement indicates teaching effectiveness because fat-soluble vitamins (A, D, E, and K) are given as supplements because the long-term antibiotic therapy used to treat cystic fibrosis eradicates the normal flora of the gastrointestinal tract. 4. This statement indicates teaching effectiveness because pancreatic replacement enzymes are given to help with digestion and absorption of food.

The nurse is concerned that a patient is at risk for developing infective endocarditis. What information in the patient's history caused the nurse to have this concern? 1. 70 years of age 2. History of chronic obstructive pulmonary disease 3. Retired company executive 4. Consumes a high protein diet

1 : Risk factors for infective endocarditis include age greater than 60; immunodeficiency; intravenous drug use; diabetes mellitus; presence of prosthetic heart valves, history of endocarditis, congenital or structural heart disease, and the presence of an intravascular access or an implanted cardiac device; poor oral hygiene or periodontal disease; patients on hemodialysis; and patients with frequent exposures to the healthcare system or invasive procedures. Risk factors for infective endocarditis do not include a history of chronic obstructive pulmonary disease.Risk factors for infective endocarditis do not include lifestyle stressors, like some cardiovascular diseases. Risk factors for infective endocarditis does not include dietary factors. Individuals with diabetes mellitus are at increased risk.

During an assessment, a patient describes experiencing chest pain with exercise that disappears with rest. The nurse correlates this finding with which health problem? 1. Stable angina 2. Variant angina 3. Unstable angina 4. Prinzmetal's angina

1 : Stable angina is chest pain or discomfort that is associated with physical activity. Symptoms of stable angina are often alleviated with rest.Variant angina is caused by coronary artery spasm rather than plaque formation and typically occurs at rest and in clusters. Unstable angina refers to chest pain that can occur at rest.Prinzmetal's angina is the same as variant angina, which is caused by coronary artery spasm rather than plaque formation and typically occurs at rest and in clusters

The nurse assesses for which heart sound in the patient diagnosed with mitral valve stenosis? 1. Click 2. Friction rub 3. Atrial gallop 4. Ventricular gallop

1 : A click is a high-pitch heart sound that is anticipated when auscultating the chest of a patient who is diagnosed with mitral valve stenosis. A friction rub is a harsh, scratching sound that is anticipated when auscultating the chest of a patient who is diagnosed with pericarditis.An atrial gallop is anticipated for the patient with decreased ventricular compliance.A ventricular gallop is anticipated for the patient with ventricular dysfunction.

A patient is being assessed for heart disease. The nurse instructs the patient to avoid eating and drinking fluids for 12 hours for which diagnostic test? 1. Lipid panel 2. Homocysteine 3. C-reactive protein 4. Partial thromboplastin time

1 : A lipid panel requires the patient to fast for approximately 8 to 12 hours before the test.Fasting is not required before a homocysteine level. Fasting is not required before a C-reactive protein level. Fasting is not required before a partial thromboplastin time level.

A patient is scheduled for pulmonary function testing. Which nursing action is most appropriate for the patient? 1. Assessing for respiratory distress 2. Scheduling the test after a meal 3. Providing rest before the procedure 4. Administering an inhaled bronchodilator 6 hours before procedure

1 : A nursing action that is appropriate when providing care to a patient who is having pulmonary function tests is to assess the patient for respiratory distress. The patient should not eat for 4 to 6 hours before the test.There are not activity restrictions before the test. The nurse would avoid administering an inhaled bronchodilator 6 hours before the procedure because this may interfere with the test results.

The nurse is caring for a patient newly diagnosed with asthma. Which assessment data indicate the need for an immediate intervention? 1. Diminished breath sounds 2. Decreased respiratory rate 3. Increased crackles 4. Increased respiratory rate

1 : Absent or diminished breath sounds signify a decrease in the movement of air as a result of increased obstruction or respiratory exhaustion.• If a patient's wheezing decreases and he has little or no breath sounds, this indicates the patient is not able to move air throughout the system. This is a medical emergency. It indicates respiratory failure. The patient may require mechanical ventilation. Respiratory status can change rapidly during an acute asthma attack. Slowed, shallow respirations with significantly, not slightly, diminished breath sounds may indicate exhaustion and impending respiratory failure. 2 Respiratory status can change rapidly during an acute asthma attack. Slowed, shallow respirations with significantly diminished breath sounds and decreased wheezing may indicate exhaustion and impending respiratory failure. Immediate intervention is necessary. Increased crackles are usually associated with heart failure and are not an indication of exhaustion.An increased respiratory rate indicates respiratory compromise, but not exhaustion.

Which assessment data collected by the nurse requires an immediate intervention in the patient with laryngeal trauma? 1. Tachypnea 2. Bradycardia 3. Hypotension 4. Complaints of throat pain

1 : An increased respiratory rate, or tachypnea, indicates respiratory distress and issues with airway clearance.Tachycardia, not bradycardia, indicates respiratory distress.The blood pressure may be slightly elevated secondary to stress and anxiety, but the patient would not be hypotensive. Pain in the area of trauma is anticipated because of edema of the soft tissue.

The nurse develops the nursing diagnosis "Ineffective Airway Clearance related to intermittent, reversible airway obstruction secondary to bronchoconstriction and airway inflammation." For which disorder is this nursing diagnosis most relevant? 1. Asthma 2. Bronchiectasis 3. Chronic bronchitis 4. Emphysema

1 : Asthma is a chronic lung disease characterized by an intermittent, reversible airway obstruction resulting from inflammation of the lung's airways and a tightening of the muscles that surround the airways. The condition affects the bronchial airways, not the alveoli. 2 Bronchiectasis is the chronic dilation of the bronchioles, often found in patients with cystic fibrosis. 3 Chronic bronchitis affects the small airways and is defined as the presence of cough and sputum production for at least 3 months in each of 2 consecutive years. The chronic airflow limitation is caused by a mixture of small airways disease and destruction of the lung tissue. 4 Emphysema causes loss of lung elasticity leading to hyperinflation of the alveoli. The small airways collapse prematurely causing trapping of air in the alveoli and subsequent distention. Carbon dioxide cannot leave the alveoli, and oxygen cannot enter, resulting in an ineffective exchange of oxygen and carbon dioxide.

The nurse educator is teaching a student nurse how to auscultate the lungs. Which action by the student nurse indicates the need for further education? 1. Listening to sound over the bony structures 2. Asking the patient to sit in an upright position 3. Instructing the patient to breathe slowly through mouth 4. Beginning auscultation from lung apices and moving toward intercostal spaces

1 : Auscultation is performed to identify fluid, mucus, or obstruction in the respiratory system. The nurse should avoid auscultating sound over bony structures because it interferes with the sound quality. Upright position optimizes airflow and allows chest expansion, which facilitates clear respiratory sounds during auscultation.Breathing slowly through an open mouth prevents transmission of turbulent sound and helps to hear clear sound. Beginning auscultation from lung apices and moving toward intercostal spaces to the lung bases helps to compare one lung with the other at the same level.

The nurse provides education to a patient who is diagnosed with atherosclerosis. Which patient statement indicates a need for additional teaching? 1. "I will decrease my intake of folic acid." 2. "I will eat a low-fat, low-cholesterol diet." 3. "I will increase by daily activity to decrease by blood pressure." 4. "I will quit smoking because nicotine increases the buildup of plaque."

1 : Elevated homocysteine levels may be lowered by a diet enriched with B-complex vitamins, particularly folic acid; therefore, this statement indicates a need for additional teaching. 2. A low-fat, low-cholesterol diet helps manage risk factors and slows the progression of atherosclerosis; therefore, this statement indicates a correct understanding of the information presented. 3. Exercise can help reduce other risk factors of atherosclerosis such as high blood pressure, diabetes, obesity, and stress; therefore, this statement indicates a correct understanding of the information presented. 4. The nicotine and carbon monoxide in cigarette smoke damage the endothelium, which sets the stage for the buildup of plaque; therefore, this statement indicates a correct understanding of the information presented.

In reviewing capnography data, the nurse monitors for increased end-tidal volume CO2 in patients with which disorder? 1. Sepsis 2. Hypothermia 3. Esophageal intubation 4. Cardiac arrest

1 : Increases in end-tidal CO2 levels may be from an increase in cellular metabolism, resulting in an increase in CO2 production or hyperventilation that causes an increase in the excretion of CO2 from the lungs. Disease processes that result in increased CO2 levels are hyperthermia, trauma, burns, or sepsis. 2. Decreases in the end-tidal CO2 levels result from inadequate ventilation, respiration, or pulmonary perfusion. Disease processes that result in lower end-tidal CO2 levels are hypothermia, sedation, pulmonary embolism, hypoperfusion of the pulmonary system, endotracheal tube placement in the esophagus, systemic hypotension, and cardiac arrest. 3 Decreases in the end-tidal CO2 levels result from inadequate ventilation, respiration, or pulmonary perfusion. Disease processes that result in lower end-tidal CO2 levels are hypothermia, sedation, pulmonary embolism, hypoperfusion of the pulmonary system, endotracheal tube placement in the esophagus, systemic hypotension, and cardiac arrest. 4 Decreases in the end-tidal CO2 levels result from inadequate ventilation, respiration, or pulmonary perfusion. Disease processes that result in lower end-tidal CO2 levels are hypothermia, sedation, pulmonary embolism, hypoperfusion of the pulmonary system, endotracheal tube placement in the esophagus, systemic hypotension, and cardiac arrest.

On assuming care for a patient being treated for tuberculosis, which assessment finding requires immediate attention by the nurse? 1. Dyspnea 2. Fatigue 3. Night sweats 4. Rust-colored sputum

1 : Initial symptoms of tuberculosis (TB) are relatively nonspecific and consist of fatigue, weight loss, and night sweats, followed by the development of a cough that produces a rusty-colored or blood-streaked sputum. As the disease progresses, dyspnea, orthopnea, and rales become evident as signs of respiratory compromise. Initial symptoms of tuberculosis are relatively nonspecific and consist of fatigue, weight loss, and night sweats.Initial symptoms of tuberculosis are relatively nonspecific and consist of fatigue, weight loss, and night sweats. Patients with symptomatic TB develop a cough that produces a rusty-colored or blood-streaked sputum. This is an expected finding in the patient with TB.

The nurse is providing care to a patient who is diagnosed with rhinitis and prescribed a second generation antihistamine. The nurse includes data regarding which medication? 1. Loratadine 2. Fluticasone 3. Guaifenesin 4. Diphenhydramine

1 : Loratadine is a second generation antihistamine. Fluticasone is a corticosteroid nasal spray. Guaifenesin is a decongestant.Diphenhydramine is a first generation antihistamine.

Which statement made by the patient with an active tuberculosis (TB) infection who is discharged to home receiving directly observed therapy indicates that teaching was effective? 1. "The home-health nurse will come to my home daily to make sure that I take my medications." 2. "I need to be on home isolation for about 2 weeks." 3. "I only have to take these medications for about 6 months." 4. "I will not need to be tested for TB after the infection is cured."

1 : Once the patient with tuberculosis (TB) has been stabilized and has started on therapy, treatment can continue at home. National TB treatment guidelines strongly recommend using directly observed therapy (DOT) when treating persons with active TB disease. DOT is in place when a trained healthcare worker, home healthcare nurse, aide, or designated and trained individual provides the prescribed TB drugs and watches the patient swallow every dose. The need for home isolation may last up to 6 weeks depending on the patient's response to therapy.The medication regimen for TB is between 9 and 12 months. The patient will continue to need to be tested for TB even after treatment is completed because reinfection is possible.

The nurse uses palpation during respiratory assessment to determine which clinical finding? 1. Tracheal position 2. Bronchovesicular sounds 3. Lung density 4. Adventitious sounds

1 : Palpation is used to determine tracheal position.Auscultation is used to determine breath sounds, both normal and adventitious. Percussion is used to assess lung density.Auscultation is used to determine breath sounds, both normal and adventitious.

How does the nurse interpret the following arterial blood gases in the patient admitted with influenza? pH 7.48PaO2 85 mm Hg PaCO2 30 mm Hg HCO3 24 mEq/L 1. Respiratory alkalosis 2. Respiratory acidosis 3. Hypoxemia 4. Normal arterial blood result

1 : Primary respiratory infections may initially cause a respiratory alkalosis (increased pH, decreased carbon dioxide [CO2]) in response to increased respiratory rate. In respiratory acidosis, there is a decreased pH and increased carbon dioxide [CO2]).Because the PaO2 is within normal limits, this is not consistent with hypoxemia. This is not a normal arterial blood gas because the pH is above normal range and the PaCO2 is low.

The nurse develops the nursing diagnosis, "Ineffective Airway Clearance related to thick purulent secretions." This nursing diagnosis is most relevant to the patient with which medical diagnosis? 1. Tuberculosis 2. Bacterial pneumonia 3. Influenza A 4. Influenza C

1 : Secretions/sputum in patients with tuberculosis are typically rust colored. Secretions in bacterial pneumonia are usually purulent and/or bloody secretions that result from buildup of exudate in the alveoli.Secretions that are white are consistent with viral influenza.Influenza C typically causes either no symptoms at all or a very mild respiratory illness.

A patient with a blood pressure of 88/50 mm Hg has a heart rate of 112 beats per minute. The nurse correlates this change in heart rate to which physiological mechanism? 1. Positive chronotropic effect 2. Negative chronotropic effect 3. Force of the mechanical contraction 4. Reaction to ventricular volume at the end of diastole

1 : Stimulus from the sympathetic nervous system initiated by information from baroreceptors in the aortic arch and the carotids that are sensitive to changes in blood pressure increase the heart rate (HR) through the release of norepinephrine. This is called a positive chronotropic effect. The parasympathetic nervous system slows the HR through the release of acetylcholine, a negative chronotropic effect. Contractility is the force of the mechanical contraction.Preload is the amount of blood in the ventricles at the end of diastole.

The nurse correlates which arterial blood gas result to the activation of chemoreceptors in the carotid bodies and aortic arch that lead to an increased respiratory rate? 1. HCO3- 24 mEq/L 2. PaCO2 30 mm Hg 3. PaO2 60 mm Hg 4. pH 7.42

1 : This is a normal bicarbonate level (22-26 mEq/L). Chemoreceptors respond to changes in oxygen and carbon dioxide concentrations. This PaCO2 reflects alkalosis, not acidosis. This result would not lead to an increase in respiratory rate. This PaO2 indicates hypoxemia, and chemoreceptors would be activated to increase blood flow to the lungs, facilitating oxygen and CO2 exchange and increasing the respiratory rate. This is a normal pH (7.35-7.45). Chemoreceptors respond to changes in oxygen and carbon dioxide concentrations.

The nurse provides education to a patient who is being treated for rhinitis. Which patient statement indicates the need for further teaching? 1. "I will use my nasal decongestant spray every day." 2. "I will make an appointment if my nasal drainage is yellow." 3. "I will avoid spending time outside when pollen levels are increased." 4. "I will cover my cough with my arm to decrease the spread of germs."

1 : This statement indicates the need for additional teaching. The nurse instructs patients to use medications as prescribed for best results. The patient should avoid nasal decongestant use for longer than 2 to 3 days at a time because of risk of rebound congestion. 2 Nasal drainage should be clear. Drainage that is yellow or green indicates an infection, which may require additional intervention. This statement indicates a correct understanding of the information presented. Reduced exposure to triggers is the first-line treatment to avoid symptoms. This statement indicates a correct understanding of the information presented. The nurse instructs patients about the importance of hand hygiene and covering the mouth when coughing or sneezing. The nurse also teaches the patient to dispose of facial tissues containing nasal drainage. This statement indicates a correct understanding of the information presented.

Which statement by the patient being discharged after hospitalization for influenza indicated the need for further teaching? 1. "I do not need to get a flu shot next year because I am now immune." 2. "I still need to get flu shots every year." 3. "I understand that the flu vaccine changes every year." 4. "I need to get my flu shot every year in the early fall."

1 : This statement indicates the need for further teaching as this patient is not immune. The influenza virus changes every year and so does the vaccine. Treatment for influenza is directed primarily toward prevention by annual vaccination. Inoculation by inactivated influenza viruses that were identified as causing outbreaks the preceding year can provide up to 80% protection from the projected upcoming year's virus. This statement indicates understanding of teaching as treatment for influenza illness is directed primarily toward prevention by annual vaccination. This statement indicates understanding of teaching as inoculation by inactivated influenza viruses that were identified as causing outbreaks the preceding year can provide up to 80% protection from the projected upcoming year's virus. This statement indicates understanding of teaching because the most appropriate time of year for vaccination is in the early fall before the "flu season" begins.

Which statement by a patient scheduled for a total laryngectomy indicates the need for further teaching? 1. "I will have to eat through a tube in stomach for the rest of my life." 2. "I will not be able to speak after the surgery." 3. "I will be able to eat and drink once I heal from the surgery." 4. "I will have a permanent tracheotomy."

1 : This statement indicates the need for further teaching because swallowing ability remains normal after total laryngectomy. The creation of a tracheostomy is required for breathing because of the risk of aspiration of fluids and food after the removal of the epiglottis. This is a correct statement; a total laryngectomy results in the inability to speak. This is a correct statement; the swallowing ability remains normal after total laryngectomy. This is a correct statement. The creation of a tracheostomy is required for breathing and because of the risk of aspiration of fluids and food after the removal of the epiglottis.

Which of the following statements from a patient having an annual physical indicates the need for further teaching regarding normal blood pressure parameters? 1. "As long as my blood pressure is not above 120/80, I know that I am fine." 2. "The bottom number needs to be below 80 to be considered normal." 3. "The top number needs to be below 120 to be considered normal." 4. "If the bottom number is above 90, I may have hypertension."

1 : This statement indicates the need for further teaching. Blood pressure readings greater than 120/80 indicate elevated blood pressure. This is particularly important teaching for older adults because in the past 120/80 was considered a normal blood pressure reading. 2 This is a correct statement. Diastolic blood pressure (DBP) greater than 80 mm Hg with systolic blood pressure (SBP) greater than 120 mm Hg indicates elevated blood pressure. This is a correct statement. SBP greater than 120 mm Hg with DBP greater than 80 mm Hg indicates elevated blood pressure.This is a correct statement. DBP greater than 90 mm Hg indicates stage 2 hypertension.

The nurse working in the emergency department recognizes that the results of a rapid influenza diagnostic test (RIDT) is usually available within which time frame? 1. 30 minutes 2. 60 minutes 3. 12 hours 4. 24 hours

1 : n emergency departments and outpatient clinics, the most commonly used tests for influenza are rapid influenza diagnostic tests (RIDTs). The identification of an influenza virus infection can be made in less than 30 minutes via nasopharyngeal/throat swab or nasal washings/aspirate. The identification of an influenza virus infection can be made in less than 30 minutes via nasopharyngeal/throat swab or nasal washings/aspirate.The identification of an influenza virus infection can be made in less than 30 minutes via nasopharyngeal/throat swab or nasal washings/aspirate. The identification of an influenza virus infection can be made in less than 30 minutes via nasopharyngeal/throat swab or nasal washings/aspirate.

The nurse is assessing a patient with acute malaise and muscle aches. Which questions should the nurse ask to determine whether the patient is experiencing influenza? Select all that apply. 1. "Have you had a flu shot this year?" 2. "Is your cough productive?" 3. "Have you been exposed to anyone with the flu?" 4. "Have you had a recent weight loss?" 5. "Do you have dizziness?"

1,2,3 This is correct. Based on the presenting symptoms, the nurse would ask whether the patient has had a flu shot or been exposed to the flu. Usually, the cough of an influenza patient is nonproductive. A productive cough may indicate a different diagnosis. 2 This is correct. Based on the presenting symptoms, the nurse would ask whether the patient has had a flu shot or been exposed to the flu. Usually, the cough of an influenza patient is nonproductive. A productive cough may indicate a different diagnosis. 3 This is correct. Based on the presenting symptoms, the nurse would ask whether the patient has had a flu shot or been exposed to the flu. Usually, the cough of an influenza patient is nonproductive. A productive cough may indicate a different diagnosis. 4 This is incorrect. Unexplained weight loss is not typically observed in patients with influenza. This is more symptomatic of tuberculosis. 5 This is incorrect. Dizziness is not a typical clinical manifestation of influenza unless the patient has significant fluid loss.

In providing teaching for a patient recently diagnosed with an active tuberculosis (TB) infection, the nurse incorporates teaching about which medications? Select all that apply. 1. Ethambutol 2. Isoniazid 3. Pyrazinamide 4. Rifampin 5. Vancomycin

1,2,3,4 This is correct. Treatment for tuberculosis (TB) includes a basic four-drug combination that continues for 9 to 12 months. The medications include ethambutol, isoniazid, pyrazinamide, and rifampin. 2 This is correct. Treatment for TB includes a basic four-drug combination that continues for 9 to 12 months. The medications include ethambutol, isoniazid, pyrazinamide, and rifampin. 3 This is correct. Treatment for TB includes a basic four-drug combination that continues for 9 to 12 months. The medications include ethambutol, isoniazid, pyrazinamide, and rifampin. 4 This is correct. Treatment for TB includes a basic four-drug combination that continues for 9 to 12 months. The medications include ethambutol, isoniazid, pyrazinamide, and rifampin. 5 This is incorrect. Vancomycin is not indicated for the treatment of TB. It may be used in the treatment of methicillin-resistant Staphylococcus aureus.

Which actions does the nurse implement in the patient who is neutropenic? Select all that apply. 1. Performing mouth care on the patient frequently 2. Implementing hand hygiene before touching the patient 3. Limiting the number of visitors who can see the patient 4. Teaching visitors to wash hands before entering the patient's room5. Ensuring the use of booties to cover the shoes before entering the patient's room

1,2,3,4: 1. This is correct. Decreased neutrophil counts significantly increase the risk of infection because of a decreased immune response. Protect the patient by using and teaching proper hand hygiene, performing frequent oral care, and limiting visitors. 2. This is correct. Decreased neutrophil counts significantly increase the risk of infection because of a decreased immune response. Protect the patient by using and teaching proper hand hygiene, performing frequent oral care, and limiting visitors. 3. This is correct. Decreased neutrophil counts significantly increase the risk of infection because of a decreased immune response. Protect the patient by using and teaching proper hand hygiene, performing frequent oral care, and limiting visitors. 4. This is correct. Decreased neutrophil counts significantly increase the risk of infection because of a decreased immune response. Protect the patient by using and teaching proper hand hygiene, performing frequent oral care, and limiting visitors. 5. This is incorrect. The use of booties as personal protective equipment (PPE) is not needed for the patient who is on neutropenic precautions.

The nurse monitors for which clinical manifestations in the patient suspected of having cardiac tamponade? Select all that apply. 1. Hypotension 2. Pulsus paradoxus 3. Muffled heart sounds 4. Jugular vein distention 5. Lower extremity edema

1,2,3,4: 1. This is correct. If the fluid causing a pericardial effusion is not addressed, it can increase and cause cardiac tamponade, which is manifested by hypotension. 2. This is correct. If the fluid causing a pericardial effusion is not addressed, it can increase and cause cardiac tamponade. Pulsus paradoxus may occur. 3. This is incorrect. Lower extremity edema is a manifestation of right-sided heart failure. 4. This is correct. If the fluid causing a pericardial effusion is not addressed, it can increase and cause cardiac tamponade, which is manifested by muffled heart sounds. 5. This is correct. If the fluid causing a pericardial effusion is not addressed, it can increase and cause cardiac tamponade, which is manifested by jugular vein distention.

Which interventions does the nurse include in the plan of care for a patient who receives chemotherapy for the treatment of laryngeal cancer? Select all that apply. 1. Assessing daily weights 2. Teaching to avoid crowds 3. Monitoring input and output 4. Encouraging small, frequent meals 5. Instructing to avoid the use of mouthwashes that contain topical anesthetics

1,2,3,4: 1. This is correct. The nurse monitors the patient's fluid and electrolyte balance, weight, intake, and output and encourages intake of fluids and small, frequent meals when providing care for a patient who is receiving chemotherapy for the treatment of laryngeal cancer. 2. This is correct. The nurse monitors the patient's white blood cell count for signs of neutropenia and the platelet count for signs of thrombocytopenia. If the patient has a decrease in either count, the patient is placed on neutropenia precautions or bleeding precautions. Patients are encouraged to avoid crowds and persons with infections and report any signs of infection immediately. 3. This is correct. The nurse monitors the patient's fluid and electrolyte balance, weight, intake, and output and encourages intake of fluids and small, frequent meals when providing care for a patient who is receiving chemotherapy for the treatment of laryngeal cancer. 4. This is correct. The nurse monitors the patient's fluid and electrolyte balance, weight, intake, and output and encourages intake of fluids and small, frequent meals when providing care for a patient who is receiving chemotherapy for the treatment of laryngeal cancer. 5. This is incorrect. The patient may use throat sprays or mouthwashes that contain a local topical anesthetic prescribed by the provider may assist in relieving the sore throat.

The nurse conducts a respiratory assessment for an adult patient who presents with a productive cough. Which additional data require the nurse to request the healthcare provider to test the patient for tuberculosis? Select all that apply. 1. A low-grade fever 2. Reports of night sweats 3. Reports of coughing up blood 4. Reports of heart palpitations 5. Weight loss from previous visit

1,2,3,5 1 This is correct. The patient who presents with a productive cough in addition to a low-grade fever should be tested for tuberculosis. 2 This is correct. The patient who presents with a productive cough in addition to reports of night sweats should be tested for tuberculosis. 3 This is correct. The patient who presents with a productive cough in addition to hemoptysis (coughing up blood) should be tested for tuberculosis. 4 This is incorrect. Heart palpations with a productive cough is not an indicator of tuberculosis. Although the report of palpitations should be reported to the healthcare provider, this does not support a need to test the patient for tuberculosis. 5 This is correct. The patient who presents with a productive cough in addition to reported or actual weight loss should be tested for tuberculosis.

The nurse monitors for which clinical manifestations in the patient with carotid artery disease. Select all that apply. 1. Dizziness 2. Difficulty talking 3. Sudden vision changes 4. Upper extremity edema 5. Sudden weakness on one side of the body

1,2,3,5: 1. This is correct. Carotid artery disease is asymptomatic until the lumen of the vessel is obstructed to the point that cerebral perfusion is impaired. The clinical manifestations of complications resulting from impaired cerebral perfusion include dizziness. 2. This is correct. Carotid artery disease is asymptomatic until the lumen of the vessel is obstructed to the point that cerebral perfusion is impaired. The clinical manifestations of complications resulting from impaired cerebral perfusion include difficulty talking. 3. This is correct. Carotid artery disease is asymptomatic until the lumen of the vessel is obstructed to the point that cerebral perfusion is impaired. The clinical manifestations of complications resulting from impaired cerebral perfusion include sudden vision problems. 4. This is incorrect. Upper extremity is associated with venous obstruction, not arterial disease. 5. This is correct. Carotid artery disease is asymptomatic until the lumen of the vessel is obstructed to the point that cerebral perfusion is impaired. The clinical manifestations of complications resulting from impaired cerebral perfusion include sudden weakness sometimes noted more on one side than the other.

The nurse notes that a patient with heart failure (HF) has a normal ejection fraction (EF). What should this information indicate to the nurse? Select all that apply. 1. It is known as HF with preserved EF. 2. It is associated with older patients with obesity. 3. It occurs in patients with diabetes mellitus and atrial fibrillation. 4. It is exacerbated with invasive procedures and dental examinations. 5. There is less blood in the ventricle to eject because of the impaired filling.

1,2,3,5: 1. This is correct. Patients with clinical manifestations of heart failure but an EF of greater than 45% are diagnosed with heart failure with preserved ejection fraction (HFpEF). 2. This is correct. HFpEF is associated with patients who tend to be older and obese. 3. This is correct. HFpEF occurs in patients with diabetes mellitus and atrial fibrillation. 4. This is incorrect. Infective endocarditis is exacerbated with invasive procedures and dental examinations. 5. This is correct. HFpEF is due to the fact that there is less blood in the ventricle to eject because of the impaired filling.

The nurse correlates which data in a patient's history as a significant risk factor for laryngeal cancer? Select all that apply. 1. Alcohol use 2. Cigarette smoking 3. Gastroesophageal reflux disease (GERD) 4. High-fat diet 5. V oice abuse

1,2,3: 1. This is correct. Alcohol ingestion combined with tobacco use increases the risk of developing laryngeal cancer up to 100 times compared with people who do not use either product, but cigarette smoking is the single highest risk factor. 2. This is correct. Use of tobacco in any form is the most important risk factor for development of cancers of the head and neck, including the larynx. Cigarettes, pipes, cigars, and all forms of chewing tobacco raise the risk of laryngeal cancers because of the chemicals contained in tobacco products. 3 This is correct. Gastroesophageal reflux disease (GERD) is a risk factor for laryngeal, as well as esophageal, cancers. 4 This is incorrect. Although there is a relationship between poor dietary habits and laryngeal cancer, there is not direct association between a high-fat diet and laryngeal cancer. 5 This is incorrect. Vocal misuse or strain can result in acute laryngitis. Vocal strain can occur from prolonged periods of talking, which may occur in teachers or lawyers; from screaming or having to talk loudly as a result of environmental noise; or in attempts to communicate with someone with hearing loss.

Which patient statements accurately reflect the benefits of physical activity during the smoking cessation process? Select all that apply. 1. "Exercise decreases stress." 2. "Exercise decreases anxiety." 3. "Exercise decreases cravings." 4. "Exercise increases weight loss." 5. "Exercise increases my desire to quit smoking."

1,2,3: 1. This is correct. Exercise decreases stress that is often experienced during smoking cessation. 2.. This is correct. Exercise decreases anxiety that is often experienced during smoking cessation. 3 This is correct. Exercise decreases cravings that are often experienced during smoking cessation. 4. This is incorrect. Although exercise is known to reduce weight gain after cessation, it is not known to increase weight loss during the time the person is stopping smoking. 5. This is incorrect. There is no direct evidence to connect smoking cessation with exercise.

Which finding in a patient with dyspnea requires an immediate intervention by the nurse? Select all that apply. 1. Accessory muscle use 2. Cyanosis 3. Hyperventilation 4. Tachypnea 5. Vesicular breath sounds

1,2,4 This is correct. Patients who are reporting dyspnea demonstrate accessory muscle use, adventitious breath sounds, cyanosis, retractions, and tachypnea and should be referred for immediate medical intervention. This is correct. Patients who are reporting dyspnea demonstrate accessory muscle use, adventitious breath sounds, cyanosis, retractions, and tachypnea and should be referred for immediate medical intervention. This is incorrect. Patients with dyspnea who have worsening of respiratory status will manifest tachypnea, not hyperventilation. This is correct. Patients who are reporting dyspnea demonstrate accessory muscle use, adventitious breath sounds, cyanosis, retractions, and tachypnea and should be referred for immediate medical intervention. This is incorrect. Vesicular are normal breath sounds.

The nurse correlates which risk factors with tuberculosis? Select all that apply. 1. Homelessness 2. Incarceration 3. Caucasian race 4. Hispanic race 5. Obesity

1,2,4 This is correct. Populations at risk for tuberculosis (TB) include low socioeconomic groups with obstacles to accessing healthcare, the homeless, and incarcerated populations. There is an increased incidence among blacks, Hispanics, and Asians. Immunosuppression is also a risk factor for TB. This is correct. Populations at risk for tuberculosis include low socioeconomic groups with obstacles to accessing healthcare, the homeless, and incarcerated populations. There is an increased incidence among blacks, Hispanics, and Asians. Immunosuppression is also a risk factor for TB. This is incorrect. Races at increased risk of TB include blacks, Hispanics, and Asians, not Caucasians. This is correct. Populations at risk for tuberculosis include low socioeconomic groups with obstacles to accessing healthcare, the homeless, and incarcerated populations. There is an increased incidence among blacks, Hispanics, and Asians. Immunosuppression is also a risk factor for TB. This is incorrect. Obesity is not a risk factor for TB.

The school nurse is planning a teaching session with the parents of students to reduce the spread of the influenza virus throughout the school. What should the nurse include when teaching the parents about minimizing the chance of spread of influenza? Select all that apply. 1. "Cover your cough" education 2. Appropriate hand hygiene 3. Safe food preparation and storage 4. Sanitizing high-touch items to kill pathogens 5. Getting the influenza vaccine in the early summer

1,2,4 This is correct. Sanitizing high-touch items such as toys and all contact surfaces, teaching children to wash their hands, and appropriate respiratory etiquette such as "cover your cough" education all control the growth and spread of microorganisms. This is correct. Sanitizing high-touch items such as toys and all contact surfaces, teaching children to wash their hands, and appropriate respiratory etiquette such as "cover your cough" education all control the growth and spread of microorganisms. 3 This is incorrect. Teaching parents safe food preparation and storage is another tool to prevent the spread of microorganisms but is not related to the flu virus. This is correct. Sanitizing high-touch items such as toys and all contact surfaces, teaching children to wash their hands, and appropriate respiratory etiquette such as "cover your cough" education all control the growth and spread of microorganisms. To prevent the spread of communicable diseases, microorganisms must be killed or their growth controlled. This is incorrect. The most appropriate time of year for vaccination is in the early fall before the "flu season" begins.

In providing care for a patient who takes 325 mg aspirin daily, the nurse teaches the patient that with long-term aspirin therapy, which laboratory tests need to be periodically monitored? Select all that apply. 1. Hematocrit 2. INR (international normalized ratio) 3. Leukocytes 4. Prothrombin time 5. Serum creatinine

1,2,4,5: 1. This is correct. During prolonged aspirin therapy, periodic assessments of hematocrit (Hct) and hemoglobin (Hgb) levels, prothrombin time (PT), international normalized ratio (INR), and renal function are recommended. 2. This is correct. During prolonged aspirin therapy, periodic assessments of Hct and Hgb levels, PT, INR, and renal function are recommended. 3. This is incorrect. The leukocyte count provides information about infection and inflammation, not clotting/bleeding. 4. This is correct. During prolonged aspirin therapy, periodic assessments of Hct and Hgb levels, PT, INR, and renal function are recommended. 5. This is correct. During prolonged aspirin therapy, periodic assessments of Hct and Hgb level, PT, INR, and renal function are recommended. Serum creatinine provides information regarding renal function.

While planning care, the nurse identified interventions to reduce a patient's risk for developing heart failure. Which assessment findings did the nurse use to make this clinical determination? Select all that apply. 1. Body mass index 31.3 2. Smokes half a pack of cigarettes 3. Employed in a textile factory 4. Blood pressure 168/90 mm Hg 5. Fasting blood glucose 80 mg/dL

1,2,4,5: 1. This is correct. A risk factor for heart failure is obesity. 2. This is correct. Smoking is a risk factor for the development of heart failure. 3. This is incorrect. Working in a textile factory is not a risk factor. Sedentary lifestyle and physical inactivity are risk factors. 4. This is correct. Hypertension is a risk factor for the development of heart failure. 5. This is correct. This is an elevated fasting blood glucose that may occur in patients with poorly controlled diabetes mellitus.

The community health nurse is preparing an educational program for local residents. Which information does the nurse include about chronic obstructive pulmonary disease (COPD)? Select all that apply. 1. Cigarette smoking is the leading cause of COPD. 2. Approximately 80% to 90% of deaths from COPD are related to smoking. 3. There is an increased incidence of COPD in individuals who abuse alcohol. 4. Less than 5% of all COPD cases are caused by deficiency in alpha-1 antitrypsin. 5. COPD is completely reversible if the individual stops smoking.

1,2,4: 1. This is correct. Cigarette smoking is the leading cause of COPD, and approximately 80% to 90% of deaths from COPD are related to smoking. 2. This is correct. Cigarette smoking is the leading cause of COPD, and approximately 80% to 90% of deaths from COPD are related to smoking. 3. This is incorrect. There is no association between incidence of COPD and alcohol use/abuse. 4. This is correct. Fewer than 5% of all COPD cases are caused by a deficiency of an enzyme called alpha-1 antitrypsin (AAT). AAT is produced in the liver and is present in the lungs. When present, it has lung protectant properties. 5. This is incorrect. COPD causes irreversible changes in the lungs. Smoking cessation can slow the progression of the disease but not totally reverse all lung damage.

The nurse teaches a patient to increase the intake of which hematopoietic vitamins to promote red blood cell formation and function? Select all that apply. 1. Folic acid 2. Vitamin B6 3. Vitamin B12 4. Vitamin C 5. Vitamin K

1,2: 1. This is correct. Folic acid and vitamin B6 promote red cell formation and function and are considered hematopoietic vitamins. 2. This is correct. Folic acid and vitamin B6 promote red cell formation and function and are considered hematopoietic vitamins. 3. This is incorrect. Vitamin B12 assists in the division and maturity of red blood cells. 4. This is incorrect. Vitamin C facilitates calcium and nonheme (vegetable) iron absorption by increasing the solubility of nonheme iron and synthesizes leukocytes and other immune cell components. 5. This is incorrect. Vitamin K is necessary for formation of prothrombin and other clotting factors in the liver.

The nurse correlates which assessment finding to activation of the renin-angiotensin-aldosterone system? Select all that apply. 1. Increased blood pressure 2. Increased heart rate 3. Increased sodium reabsorption 4. Increased urine output5. Increased oxygen saturation

1,3 This is correct. Renin reacts with angiotensinogen to create angiotensin I. Angiotensin I is then converted in the lungs to angiotensin II via angiotensin-converting enzyme. Angiotensin II is a potent vasoconstrictor that increases blood pressure. This is incorrect. Heart rate is not controlled by the renin-angiotensin-aldosterone system. Catecholamines increase heart rate. This is correct. Renin reacts with angiotensinogen to create angiotensin I. Angiotensin I is then converted in the lungs to angiotensin II via angiotensin-converting enzyme. Angiotensin II acts on the adrenals to release aldosterone that promotes sodium and water reabsorption in the kidneys, which increases circulating fluid volume. This is incorrect. Because aldosterone promotes sodium and water reabsorption in the kidneys, urine output does not increase. This is incorrect. The renin-angiotensin-aldosterone system does not impact oxygen saturation.

The nurse assesses for fine crackles (fine rales) in patients admitted with which respiratory disorders? Select all that apply. 1. Pneumonia 2. Pulmonary edema 3. Fibrosis 4. Chronic obstructive pulmonary disease (COPD) 5. Asthma

1,3,4 This is correct. Patients with bronchitis, COPD, fibrosis, and pneumonia may have fine crackles (fine rales) that are described as the sound of rubbing hair follicles together caused by inflation of previously deflated lung tissue. This is incorrect. Patients with pulmonary edema usually manifest with coarse crackles as a result of fluid in the lower airways. This is correct. Patients with bronchitis, COPD, fibrosis, and pneumonia may have fine crackles (fine rales) that are described as the sound of rubbing hair follicles together caused by inflation of previously deflated lung tissue. This is correct. Patients with bronchitis, COPD, fibrosis, and pneumonia may have fine crackles (fine rales) that are described as the sound of rubbing hair follicles together caused by inflation of previously deflated lung tissue. This is incorrect. Patients with asthma typically develop wheezing that is described as a squeaky musical instrument caused by bronchoconstriction and inflammation.

The nurse assesses for stridor in patients admitted with which respiratory disorders? Select all that apply. 1. Allergic reactions 2. Chronic obstructive pulmonary disease (COPD) 3. Epiglottis 4. Laryngitis 5. Pleurisy

1,3,4 This is correct. Stridor, described as a high-pitched sound during inspiration caused by airway obstruction of the throat or upper airway or spasm of the airway, may be auscultated in patients with allergic reactions, epiglottis, and laryngitis. 2 This is incorrect. Patients with COPD typically have fine or coarse crackles on auscultation. 3 This is correct. Stridor, described as a high-pitched sound during inspiration caused by airway obstruction of the throat or upper airway or spasm of the airway, may be auscultated in patients with allergic reactions, epiglottis, and laryngitis. 4 This is correct. Stridor, described as a high-pitched sound during inspiration caused by airway obstruction of the throat or upper airway or spasm of the airway, may be auscultated in patients with allergic reactions, epiglottis, and laryngitis. 5 This is incorrect. Pleural friction rubs are auscultated in patients with pleurisy. These sounds, described as grating or squeaking, are caused by inflammation in the pleural space.

Which information should the nurse include when documenting the characteristics of normal heart sounds? Select all that apply. 1. Pitch 2. Clicks 3. Quality 4. Intensity 5. Location

1,3,4,5 This is correct. When describing heart sounds, it is important to note pitch. 2 This is incorrect. Clicks would be an abnormal finding, necessitating more thorough documentation. 3 This is correct. When describing heart sounds, it is important to note quality 4 This is correct. When describing heart sounds, it is important to note intensity. 5 This is correct. When describing heart sounds, it is important to note location.

A patient is experiencing manifestations of infective endocarditis. Which diagnostic tests are indicated for this patient? Select all that apply. 1. Blood cultures 2. Ejection fraction 3. Electrocardiogram 4. Transthoracic echocardiogram (TTE) 5. Transesophageal echocardiogram (TEE)

1,3,4,5: 1. This is correct. Tests used to confirm infective endocarditis include blood cultures. 2. This is incorrect. Ejection fraction is used to diagnose the type and extent of heart failure. 3. This is correct. Tests used to confirm infective endocarditis include an electrocardiogram. 4. This is correct. Tests used to confirm infective endocarditis include a transthoracic echocardiogram (TTE). 5. This is correct. Tests used to confirm infective endocarditis include a transesophageal echocardiogram (TEE).

The nurse correlates which diagnostic results to the patient with pancytopenia? Select all that apply. 1. Decreased reticulocyte count 2. Increased immature neutrophils 3. Platelets 100,000 103/mm3- 4. RBC 3.2 million cells/mm3 5. WBC 12.0 103/mm3-

1,3,4: 1. This is correct. The complete blood count (CBC) results in the patient with aplastic anemia reveal pancytopenia, including decreased reticulocyte, white blood cell (WBC), and platelet counts. 2. This is incorrect. Increased immature neutrophils are associated with infection. 3. This is correct. The CBC results in the patient with aplastic anemia reveal pancytopenia, including decreased reticulocyte, WBC, and platelet counts. 4. This is correct. The CBC results in the patient with aplastic anemia reveal pancytopenia, including decreased reticulocyte, WBC, and platelet counts. 5. This is incorrect. This is an elevated white blood count.

The nurse is providing care to a patient diagnosed with chronic obstructive pulmonary disease (COPD). Which factors in the patient's history support the current diagnosis? Select all that apply. 1. Working in an industrial environment 2. Working in an office setting with air conditioning 3. History of asthma 4. Current cigarette smoking 5. Playing golf several times a week

1,3,4: 1. This is correct. Risk factors associated with the development of chronic obstructive pulmonary disease (COPD) include working in an industrial environment, a history of asthma, and cigarette smoking. 2. This is incorrect. Working in an office setting with air conditioning and playing golf several times a week are not risk factors for the development of COPD. 3. This is correct. Risk factors associated with the development of COPD include working in an industrial environment, a history of asthma, and cigarette smoking. 4. This is correct. Risk factors associated with the development of COPD include working in an industrial environment, a history of asthma, and cigarette smoking. 5. This is incorrect. Working in an office setting with air conditioning and playing golf several times a week are not risk factors for the development of COPD.

The nurse correlates which variables as directly influencing stroke volume? Select all that apply. 1. Afterload 2. Conductivity 3. Contractility 4. Coronary circulation 5. Preload

1,3,5 This is correct. Stroke volume is the amount of blood ejected with each ventricular contraction and is influenced by three variables: preload, afterload, and contractility. This in incorrect. Conductivity allows cardiac tissue to transmit the impulses to neighboring connected cells. This is correct. Stroke volume is the amount of blood ejected with each ventricular contraction and is influenced by three variables: preload, afterload, and contractility. This is incorrect. Coronary circulation is the blood that perfuses the cardiac tissue. It may indirectly impact cardiac output if the heart is not pumping effectively. This is correct. Stroke volume is the amount of blood ejected with each ventricular contraction and is influenced by three variables: preload, afterload, and contractility.

The nurse monitors for which clinical manifestations in the patient diagnosed with malignant lymphoma? Select all that apply. 1. Night sweats 2. High fevers 3. Swollen lymph nodes 4. Painful lymph nodes 5. Weight loss

1,3,5: 1. This in correct. Clinical manifestations include painless swelling of the lymph nodes in the neck, underarm, and groin; low-grade fevers for no apparent reason; drenching night sweats; unexplained weight loss of more than 10% in less than 6 months; and fatigue.This in correct. Clinical manifestations include painless swelling of the lymph nodes in the neck, underarm, and groin; low-grade fevers for no apparent reason; drenching night sweats; unexplained weight loss of more than 10% in less than 6 months; and fatigue. 2. This is incorrect. The patient with malignant lymphoma has low grade fevers for no known reason. 3. This in correct. Clinical manifestations include painless swelling of the lymph nodes in the neck, underarm, and groin; low-grade fevers for no apparent reason; drenching night sweats; unexplained weight loss of more than 10% in less than 6 months; and fatigue. 4. This is incorrect. Clinical manifestations of malignant lymphoma include painless, not painful, swelling of the lymph nodes in the neck, underarm, and groin. 5. This in correct. Clinical manifestations include painless swelling of the lymph nodes in the neck, underarm, and groin; low-grade fevers for no apparent reason; drenching night sweats; unexplained weight loss of more than 10% in less than 6 months; and fatigue.

In providing care to a patient with an acute exacerbation of heart failure, the nurse prepares to administer prescribed medication that provide which actions? Select all that apply. 1. Decrease preload 2. Increase preload 3. Decrease afterload 4. Increase afterload 5. Increase contractility

1,3,5: 1. This is correct. An acute exacerbation of heart failure (HF) is typically treated with intravenous (IV) medications that can quickly and effectively decrease preload and afterload and increase contractility. Nitroglycerin and nitroprusside (Nitropress), potent vasodilators, are commonly used. IV inotropic agents (dopamine) can be used to increase contractility, whereas inodilators, agents with both positive inotropic and vasodilator effects (dobutamine and milrinone), provide positive inotropic effects and reduce afterload. 2. This is incorrect. Both preload and afterload need to be decreased in patients with an acute exacerbation of heart failure. 3. This is correct. An acute exacerbation of HF is typically treated with IV medications that can quickly and effectively decrease preload and afterload and increase contractility. Nitroglycerin and nitroprusside (Nitropress), potent vasodilators, are commonly used. IV inotropic agents (dopamine) can be used to increase contractility, whereas inodilators, agents with both positive inotropic and vasodilator effects (dobutamine and milrinone), provide positive inotropic effects and reduce afterload. 4. This is incorrect. Both preload and afterload need to be decreased in patients with an acute exacerbation of heart failure. 5. This is correct. An acute exacerbation of HF is typically treated with IV medications that can quickly and effectively decrease preload and afterload and increase contractility. Nitroglycerin and nitroprusside (Nitropress), potent vasodilators, are commonly used. IV inotropic agents (dopamine) can be used to increase contractility, whereas inodilators, agents with both positive inotropic and vasodilator effects (dobutamine and milrinone), provide positive inotropic effects and reduce afterload.

Which systems should the nurse anticipate will be affected when planning care for a patient diagnosed with cystic fibrosis? Select all that apply. 1. Respiratory 2. Neurological 3. Reproductive 4. Cardiovascular 5. Gastrointestinal

1,3,5: 1. This is correct. Cystic fibrosis is a multisystem disease that produces increased amounts of thick mucus in the respiratory, gastrointestinal (GI), and reproductive systems. The disease is characterized by thick, viscous mucus that clogs the lungs and obstructs the pancreas. Other organs that are affected include the liver, salivary glands, and testes. 2. This is incorrect. The neurological system is not directly affected by cystic fibrosis. 3. This is correct. Cystic fibrosis is a multisystem disease that produces increased amounts of thick mucus in the respiratory, GI, and reproductive systems. The disease is characterized by thick, viscous mucus that clogs the lungs and obstructs the pancreas. Other organs that are affected include the liver, salivary glands, and testes. 4. This is incorrect. The cardiovascular system is not directly affected by cystic fibrosis. 5. This is correct. Cystic fibrosis is a multisystem disease that produces increased amounts of thick mucus in the respiratory, GI, and reproductive systems. The disease is characterized by thick, viscous mucus that clogs the lungs and obstructs the pancreas. Other organs that are affected include the liver, salivary glands, and testes.

The nurse caring for a homeless patient at risk for tuberculosis (TB) includes which clinical manifestations of the disease when educating the patient? Select all that apply. 1. Fatigue 2. Green-tinged sputum 3. Productive cough that later turns to a dry, hacking cough 4. Weight loss 5. Night sweats

1,4,5 This is correct. Manifestations of tuberculosis often develop insidiously and are initially nonspecific. Fatigue, weight loss, and night sweats are common. It is often at this stage that the patient first seeks medical attention. This is incorrect. Rusty-colored, not greenish, sputum occurs as a result of the destruction of lung tissue during granuloma formation. This is incorrect. A dry cough develops, which later becomes productive of purulent and/or blood-tinged sputum. This is correct. Manifestations of tuberculosis often develop insidiously and are initially nonspecific. Fatigue, weight loss, and night sweats are common. It is often at this stage that the patient first seeks medical attention. 5 This is correct. Manifestations of tuberculosis often develop insidiously and are initially nonspecific. Fatigue, weight loss, and night sweats are common. It is often at this stage that the patient first seeks medical attention.

The nurse is preparing content for a community health fair on risk factors for heart disease. What should the nurse include as nonmodifiable risk factors? Select all that apply. 1. Age 2. Weight 3. Alcohol intake 4. Ethnic background 5. Parents' health history

1,4,5 This is correct. Nonmodifiable risk factors include age, sex, race/ethnicity, and family history of health issues and diseases. 2 This is incorrect. Modifiable risk factors for heart disease include weight, tobacco use, alcohol use, dietary habits, and activity level. 3 This is incorrect. Modifiable risk factors for heart disease include weight, tobacco use, alcohol use, dietary habits, and activity level. 4 This is correct. Nonmodifiable risk factors include age, sex, race/ethnicity, and family history of health issues and diseases. 5 This is correct. Nonmodifiable risk factors include age, sex, race/ethnicity, and family history of health issues and diseases.

The nurse correlates an intact immune system with the function of which stem cells? Select all the apply. 1. B lymphocytes 2. Granulocytes 3. Neutrophils 4. Thrombocytes 5. T lymphocytes

1,5: 1. This is correct. Lymphoid stem cells produce either T or B lymphocytes, which are the main functional cells of the immune system. Myeloid stem cells differentiate into three cell types: red blood cells (RBCs; erythrocytes), white blood cells (WBCs; leukocytes), and platelets (thrombocytes). 2. This is incorrect. Granulocytes play a key role in protecting the body from harmful microorganisms during acute inflammation and infection by means of their individual functions. 3. This is incorrect. Neutrophils are white blood cells involved in phagocytosis. 4. This is incorrect. Thrombocytes are platelets and have no role in immunity. 5. This is correct. Lymphoid stem cells produce either T or B lymphocytes, which are the main functional cells of the immune system. Myeloid stem cells differentiate into three cell types: RBCs (erythrocytes), WBCs (leukocytes), and platelets (thrombocytes).

The nurse recognizes which as a function of red blood cells? 1. Facilitating osmosis between O2 and CO2 2. Transporting O2 from the lungs to the tissues 3. Transporting CO2 from the lungs to the tissues 4. Facilitating active transport between O2 and CO2

2 : 1. A shift to the left refers to an increased number of immature neutrophils. Erythrocytes are red blood cells. Red blood cells have two principal functions: (1) to pick up oxygen from the lungs and transport it to systemic tissues and (2) to pick up carbon dioxide from the tissues and deliver it in the lungs. 2. Bands (immature neutrophils) generally account for a small percentage (3%- 5%) of circulating granulocytes. An increase in band level is called a left shift, which occurs with acute infection, inflammation, or some other significant physical stress. Because they are immature, the bands are not very effective in combating infection. 3. "Segs" are mature neutrophils, and an increase is not indicative of a shift to the left. 4. Thrombocytes are platelets and have no role in infection or inflammation.

The home healthcare nurse is preparing a care plan for a patient with severe anemia. The patient currently lives alone and states, "I can't even walk to the kitchen without getting winded." What would be the priority nursing diagnosis for this patient? 1. Self Care Deficit 2. Activity Intolerance 3. Impaired Mobility 4. Anxiety

2 : 1. Although Self Care Deficit, Impaired Mobility, and Anxiety are appropriate nursing diagnoses for this patient, they are not the priority. 2. Activity Intolerance is a priority diagnosis for this patient. Maslow's Hierarchy of Needs indicates physiological needs take priority over psychosocial needs. 3. Although Self Care Deficit, Impaired Mobility, and Anxiety are appropriate nursing diagnoses for this patient, they are not the priority. 4. Although Self Care Deficit, Impaired Mobility and Anxiety are appropriate nursing diagnoses for this patient, they are not the priority.

In providing care for a patient with mitral stenosis, the nurse correlates which hemodynamic parameter most directly with this disease of this valve? 1. Increased right atrial pressure 2. Increased left atrial pressure 3. Increased right ventricular pressure 4. Increased left ventricular pressure

2 : 1. An increased right atrial pressure occurs most directly in patients with tricuspid stenosis that restricts blood flow out of the right atrium. 2. An increased left atrial pressure occurs most directly in patients with mitral stenosis that restricts blood flow out of the right atrium. 3. An increased right ventricular pressure occurs most directly in patients with pulmonic stenosis that restricts blood flow out of the right atrium. 4. An increased left ventricular pressure occurs most directly in patients with aortic stenosis that restricts blood flow out of the right atrium.

The nurse prioritizes which nursing diagnosis for the patient with a decreased neutrophil count? 1. Activity Intolerance 2. Fatigue 3. Risk for Bleeding 4. Risk for Infection

2 : 1. Blood is made up of plasma (a clear yellow, protein-rich fluid) and the cells and cell fragments suspended in it: red blood cells (RBCs), white blood cells (WBCs), and platelets. 2. Plasma is a clear yellow, protein-rich fluid that is found within the blood. Plasma is a complex mixture of water, proteins, nutrients, electrolytes, nitrogenous wastes, hormones, and gases. 3. Blood is made up of plasma (a clear yellow, protein-rich fluid) and the cells and cell fragments suspended in it: RBCs, WBCs, and platelets. 4. Blood is made up of plasma (a clear yellow, protein-rich fluid) and the cells and cell fragments suspended in it: RBCs, WBCs, and platelets.

The nurse monitors for which diagnostic result in the patient with glucose-6-phosphate dehydrogenase (G6PD) deficiency?1. Decreased reticulocyte count 2. Presence of Heinz bodies 3. White blood cells in the urine 4. Elevated platelets

2 : 1. In glucose-6-phosphate dehydrogenase (G6PD) deficiency, the increased reticulocyte count reflects to production of new red blood cells secondary to hemolysis. 2. Diagnostic testing for G6PD deficiency includes complete blood count (CBC), reticulocyte count, and blood smear analysis for Heinz bodies (accumulation of degraded hemoglobin in red blood cells). The CBC in patients with this disorder reveals a mild to severe anemia. 3. White blood cells in urine are not associated with G6PD deficiency. 4. Platelets are not elevated in G6PD deficiency.

The nurse develops the nursing diagnosis "Risk for Bleeding related to lack of intrinsic factor" for the patient with which hematological disorder? 1. Iron deficiency anemia 2. Pernicious anemia 3. Folic acid deficiency anemia 4. Glucose-6-phosphate dehydrogenase (G6PD) deficiency

2 : 1. Patients with iron deficiency anemia are at risk for Inadequate Tissue Perfusion, Fatigue, and Activity Intolerance. 2. Pernicious anemia is caused by the lack of intrinsic factor. Although pernicious anemia is often erroneously used as a synonym for vitamin B12 deficiency anemia, in actuality pernicious anemia is an autoimmune disease that leads to a vitamin B12 deficiency because of the inability to absorb vitamin B12 without intrinsic factor. 3. Patients with folic acid deficiency anemia are at risk for Inadequate Tissue Perfusion and Activity Intolerance. 4. Patients with glucose-6-phosphate dehydrogenase (G6PD) deficiency are at risk for Inadequate Tissue Perfusion, Fatigue, and Activity Intolerance.

A patient diagnosed with polycythemia is prescribed radiation, and asks the rationale for this treatment. Which response by the nurse is accurate? 1. "It stimulates red blood cell production." 2. "It suppresses the bone marrow." 3. "It provides vitamin supplementation." 4. "It decreases the risk of transfusion reactions."

2 : 1. Radiation decreases, not increases, the production of red blood cells. 2. Radiation suppresses the bone marrow in an attempt to decrease the production of red blood cells. 3. Radiation does not provide vitamin supplementation. 4. Radiation does not decrease the risk for a transfusion reaction.

The nurse correlates which diagnostic finding to a diagnosis of multiple myeloma? 1. Serum calcium 8.5 mg/dL 2. Serum creatinine 2.2 mg/dL 3. Hemoglobin 18 g/dL 4. Serum BUN 15 mg/dL

2 : 1. Serum calcium levels are increased in multiple myeloma. 2. The CRAB criterion is useful when formulating a diagnosis of multiple myeloma and includes hypercalcemia (serum calcium > 11.5 mg/dL), renal insufficiency (serum creatinine > 2 mg/dL), anemia (hemoglobin < 10 g/dL or > 2 g/dL less than the lower limit of normal range), and bone disease (lytic lesions, severe osteopenia, or pathological fractures). 3. The hemoglobin is low. This value is elevated. 4. Renal insufficiency is associated with multiple myeloma, but this blood urea nitrogen (BUN) level is within the normal range.

The nurse encourages the patient with iron deficiency anemia to increase which food in the diet? 1. Shellfish 2. Lima beans 3. Citrus fruits 4. Milk products

2 : 1. Shellfish is a source of copper, not iron. 2. Lima beans are a source of iron. 3. Citrus fruits are a source of vitamin C, not iron. 4. Milk products are a source of calcium, not iron.

In monitoring a patient at the completion of a bleeding time test, which finding requires an intervention? 1. Bleeding does not stop within 5 minutes of the completion of the test. 2. Bleeding does not stop within 15 minutes of the completion of the test. 3. The average bleeding time is calculated as 6 minutes. 4. The average bleeding time is calculated as 8 minutes.

2 : 1. This is a normal finding. Bleeding time should stop between 3 to 8 minutes of the initial cuts. 2. This indicates an excessive bleeding time. 3. The normal bleeding time is 3 to 8 minutes. This is a normal finding. 4. The normal bleeding time is 3 to 8 minutes. This is a normal finding.

The nurse is caring for a male patient with hypersplenism. Which laboratory result does the nurse correlate to this disorder? 1. Hemoglobin 18.6 g/dL 3 2. Platelet count 120,000/mm 3. White blood count 13.6 103/mm3- 4. Hematocrit 55%

2 : 1. This is an elevated hemoglobin for a male patient and not consistent with hypersplenism. 2. Decreased levels of platelets (thrombocytopenia) are caused by suppressed bone marrow function, autoimmune disease, hypersplenism, or splenomegaly. 3. This is an elevated white blood count and not consistent with hypersplenism. Elevations in the white blood cell count are associated with inflammation and infection. 4. This is an elevated hematocrit for a male patient and not consistent with hypersplenism.

A patient complaining of mouth soreness had gastric bypass surgery 1 year ago. During the assessment, the nurse notes the patient's tongue is beefy, red, and smooth and the patient's skin appears yellowish. The nurse correlates these findings to a decreased value of which diagnostic test? 1. Vitamin B6 levels 2. Vitamin B12 levels 3. Platelet count 4. Iron levels

2 : 1. Vitamin B6 deficiencies are not typically seen with gastric bypass surgeries and are not manifested with a beefy, red, smooth tongue. 2. Vitamin B12 deficiency is associated with gastric bypass surgery. A deficiency of vitamin B12 levels will result in pernicious anemia. This deficiency will manifest as pallor, jaundice, and weakness, and a beefy, smooth red tongue. 3. The patient's reports are not consistent with a thrombocytopenia. 4. Iron deficiency anemia is assessed through iron levels.

The nurse is providing care to a patient who has impaired platelet aggregation. What does the nurse anticipate based on this data? 1. The patient is taking vitamin K supplements. 2. The patient is taking anti-inflammatory agents. 3. The patient is taking folic acid. 4. The patient is taking B12 supplements and has a red blood cell count 4.6 million cells/μL.

2 : 1. Vitamin K causes hemolysis, but it does not impair platelet aggregation. 2. Anti-inflammatory agents reduce the platelet aggregation. 3. Folic acid promotes red cell formation and is classified as a hematopoietic vitamin. 4. Vitamin B12 assists in the division and maturity of red blood cells. It has no impact on clotting.

A patient is admitted for treatment of pericarditis. For which additional health problem should the nurse expect the patient to be evaluated? 1. Asthma 2. Myocardial infarction 3. Infective endocarditis 4. Chronic obstructive pulmonary disease

2 : Pericarditis occurs more in men and young and middle-aged persons, and recurrence is common with 20% to 30% of persons having an additional episode. 2. Pericarditis is not associated with asthma. Acute pericarditis is a common occurrence after myocardial infarction (MI). It has been reported to occur in 7% to 41% of post-MI patients. 3. Pericarditis occurs more in men and young and middle-aged persons, and recurrence is common with 20% to 30% of persons having an additional episode. Pericarditis is not identified as routinely occurring after infective endocarditis. 4. Pericarditis occurs more in men and young and middle-aged persons, and recurrence is common with 20% to 30% of persons having an additional episode. Pericarditis is not associated with chronic obstructive pulmonary disease.

In a patient with coronary artery disease with elevated liver function test results, it is a priority for the nurse to follow up with the healthcare provider about which prescription? 1. Aspirin (Ecotrin) 2. Atorvastatin (Lipitor) 3. Metoprolol (Lopressor) 4. Cholestyramine (Questran)

2 : 1. Aspirin (Ecotrin) therapy is not contraindicated for this patient. It is prescribed to inhibit platelet aggregation and clot development. 2. Atorvastatin (Lipitor), a statin used to reduce cholesterol, is contraindicated for patients with active liver disease. The nurse follows up with the provider based on the current elevation of liver function tests. 3. Metoprolol (Lopressor) is a beta blocker that is prescribed because it inhibits the sympathetic nervous system response to physical activity, which decreases cardiac workload and oxygen consumption. There is no reason to follow up with the prescriber regarding this prescription. 4. Cholestyramine (Questran) is a bile acid sequestrant that may be prescribed in addition to a statin (atorvastatin) to lower low-density lipoprotein (LDL) cholesterol. There is no reason to follow up with the prescriber regarding this prescription.

Based on an auscultated bruit over a patient's carotid artery, the nurse explains to the patient which test as the most commonly ordered noninvasive diagnostic test to evaluate atherosclerotic plaque? 1. Carotid angiography 2. Carotid duplex ultrasound 3. Magnetic resonance angiography 4. Computed tomography angiography

2 : 1. Carotid angiography is an invasive test currently used to diagnosis the severity of carotid artery disease. 2. Duplex ultrasound is the most commonly used noninvasive screening tool to evaluate for atherosclerotic plaque and stenosis of the external carotid artery. 3. Magnetic resonance angiography (MRA) is increasingly being used to evaluate for atherosclerotic carotid occlusive disease, but it is an invasive procedure. 4. Computed tomography angiography (CTA) has gained increasing popularity in the evaluation of carotid disease, but it is an invasive procedure. The advantages of CTA over MRA include faster data acquisition time and better spatial resolution even in very tortuous vessels.

In monitoring a patient immediately after a carotid endarterectomy (CEA), the patient demonstrates impaired swallowing. The nurse correlates this finding to compression of which cranial nerve (CN)? 1. CN VII, Facial 2. CN X, V agus 3. CN XI, Spinal accessory 4. CN XII, Hypoglossal

2 : 1. Cranial nerve (CN) VII: Facial nerve controls symmetry of face when smiling and showing teeth. 2. CN X: Vagus nerve controls swallowing, gag reflex, and is assessed by asking the patient to say "Ah." 3. CN XI: Spinal accessory nerve controls the ability to shrug shoulders and rotate head side to side. 4. CN XII: Hypoglossal nerve controls tongue movement and is assessed by asking the patient to stick out tongue.

The nurse correlates which diagnostic result as consistent with the diagnosis of cystic fibrosis? 1. Elevated serum chloride 2. Elevated sodium levels in sweat 3. Decreased urine chloride 4. Decreased urine specific gravity

2 : 1. Cystic fibrosis does not impact serum or urine chloride levels. Elevated levels of chloride in the sweat are found in patients with cystic fibrosis. 2. The sweat chloride test measures the concentration of chloride in the patient's sweat. A high level of chloride is an indication of cystic fibrosis. The normal value is less than 30 mEq/L. The test is positive for cystic fibrosis when the chloride level is greater than or equal to 60 mEq/L. 3. Cystic fibrosis does not impact serum or urine chloride levels. Elevated levels of chloride in the sweat are found in patients with cystic fibrosis. 4. Urine specific gravity is not directly impacted by cystic fibrosis. A decreased urine specific gravity is associated with fluid overload and decreased urine osmolarity

A nurse is providing discharge instructions to a patient with iron deficiency anemia who is experiencing glossitis. Which patient statement indicates the need for further education? 1. "I will monitor my lips and tongue daily." 2. "I will use an alcohol-based mouthwash twice per day." 3. "I will apply a petroleum-based lubricating ointment to my lips." 4. "I will use a soft toothbrush when brushing my teeth each day."

2 : 1. Glossitis, inflammation of the tongue that may cause the tongue and lips to turn red, and cheilosis (fissures or cracks at the corners of the mouth) may occur with nutritional deficiencies of iron, folate, and vitamin B12. Patient education should include monitoring the condition of lips and tongue daily. 2. The patient should not use an alcohol-based mouthwash because this would worsen the glossitis. 3. The patient should use a petroleum-based lubricating jelly or ointment on the lips after oral care. 4. The patient should practice frequent oral hygiene with a soft-bristle toothbrush or sponge.

A patient is recovering from mechanical valve replacement surgery for valvular disease. Which statement by the patient indicates understanding of medication teaching? 1. "I will be receiving diuretics for the long term." 2. "I will need to be on anticoagulant therapy for life." 3. "I will need to take antibiotics for 4 to 6 weeks." 4. "I may need to be on antiarrhythmic medications."

2 : 1. Long-term use of diuretics may be indicated in the patient with heart failure. 2. Patients who undergo valve replacement with a mechanical prosthetic valve will need to be anticoagulated for life to prevent thrombotic events. 3. Antibiotic therapy for 4 to 6 weeks is indicated for the patient with infective endocarditis. 4. Antiarrhythmic medication is not routinely prescribed for the patient with a mechanical heart valve.

The nurse monitors for which early clinical manifestation in the patient admitted with Buerger's disease? 1. Bounding pulses 2. Claudication 3. Gangrene in affected extremity 4. Pitting edema in the lower extremities

2 : 1. Pulses are weak, suggesting poor flow through the arteries, in patients with Buerger's disease. 2. Claudication (exercise induced pain) in numbness/tingling in the hands/feet are earlier clinical manifestations of Buerger's disease. 3. Gangrene in affected extremity is a later clinical manifestation of Buerger's disease. 4. Pitting edema is not associated with Buerger's disease. It may be observed in patients with venous obstruction.

A patient recovering from a carotid endarterectomy (CEA) has a blood pressure of 90/48 mm Hg. What should the nurse do first? 1. Raise the head of the bed 2. Lower the head of the bed 3. Assess cranial nerve function 4. Place the head in neutral position

2 : 1. Raising the head of the bed would be appropriate if the patient's blood pressure was elevated. 2. After a CEA, a new "normal" pressure may be sensed as a high pressure signaling the vagus nerve to respond, resulting in vasodilation, bradycardia, and hypotension. The head of the bed should be lowered and the patient placed in a flat position to ensure cerebral perfusion. 3. Assessing cranial nerve function is important, but the priority is treating the hypotension. 4. Maintaining the head in the neutral position decreases strain on incision site and carotid artery. It does not raise blood pressure.

In completing an admission assessment on a patient admitted for evaluation of peripheral arterial disease (PAD), which statement by the patient correlates with stage III PAD? 1. "The pain usually goes away when I lie down." 2. "The pain frequently awakens me at night." 3. "I do not feel sharp objects when I step on them." 4. "The pain is the same when I take long walk."

2 : 1. Stage II (claudication) is described as muscle pain, burning, and cramping that is experienced with exercise and relieved by rest. 2. Stage III peripheral arterial disease (PAD) (rest pain) is characterized as pain that is experienced at rest, that often awakens the patient at night, and is described as numbness and burning and usually occurs in the distal portion of the extremity. 3. The total loss of feeling and pain is associated with stage IV and gangrene and/or infection may occur. 4. Stage II (claudication) is pain that is reproducible with the same amount of exercise.

The nurse is preparing teaching material to help a patient with atherosclerosis manage lifestyle changes. What should the nurse emphasize in this teaching? 1. "You need to limit cigarette smoking." 2. "You need to follow a low-fat, low-cholesterol diet." 3. "You should consider adopting an active lifestyle." 4. "You may have dizziness at times which is expected."

2 : 1. The patient should be encouraged to stop smoking because the nicotine and carbon monoxide in cigarette smoke damage the endothelium, which sets the stage for the buildup of plaque. 2. A low-fat, low-cholesterol diet helps manage risk factors and slows the progression of atherosclerosis. 3. The person with atherosclerosis should be encouraged to increase activity and not just consider adding activity. 4. Because patients with peripheral arterial disease have an increased risk for developing chronic angina, myocardial infarction, or stroke, chest pain or neurological changes need to be reported immediately.

In reviewing healthcare provider admission orders for a patient admitted for treatment of a deep vein thrombosis of the left lower leg, which order should the nurse question? 1. Compression stockings on both legs 2. Sequential compression devices on both legs 3. Elevate the left leg 10 to 20 degrees above the heart 4. Encourage fluid intake

2 : 1. This is an appropriate order. Compression stockings should be worn at all times because they promote venous return and decrease leg swelling. 2. This order should be questioned. Avoid use of sequential compression devices (SCDs) in affected extremity. An SCD may cause the thrombus to break away, resulting in an embolus. 3. This is an appropriate order. When at rest, the affected extremity should be elevated at least 10 to 20 degrees above heart level to enhance venous return and reduce swelling. 4. This is an appropriate order. Fluid intake prevents dehydration and sluggish blood flow, which exacerbates deep vein thrombosis growth.

The nurse provides education to a patient who is prescribed the DASH diet. Which patient statement indicates a need for additional teaching? 1. "I will eat four servings of fruit each day." 2. "I will eat one serving of nonfat dairy each day." 3. "I will eat five servings of vegetables each day." 4. "I will eat at least three whole-grain foods each day."

2 : 1. This statement indicates a correct understanding of the DASH diet. Four to five servings of fruit per day are recommended. 2. Two to three servings of low-fat or nonfat dairy is recommended each day with the DASH diet. This patient statement indicates a need for additional teaching. 3. This statement indicates a correct understanding of the DASH diet. Four to five servings of vegetables per day are recommended. 4. This statement indicates a correct understanding of the DASH diet. Six to eight servings of grain and grain products are recommended each day. Three of these servings should be whole-grain foods.

The nurse auscultates a cardiac click and recognizes that this may be caused by which cardiac disorder? 1. Aortic regurgitation 2. Mitral valve stenosis 3. Pericarditis 4. Aortic stenosis

2 : A diastolic murmur is associated with aortic regurgitation. A click is associated with mitral valve stenosis.A pleural friction rub is associated with pericarditis.A systolic murmur is associated with aortic stenosis.

After palpating a patient's radial pulses, the nurse proceeds to palpate the brachial, carotid, femoral, popliteal, and posterior tibial pulses. The nurse completes this more comprehensive assessment based on which finding? 1. Skin warm and dry 2. V arious skin temperatures 3. Edema of the left lower extremity 4. Respiratory rate of 24 and labored

2 : Adequate cardiac output produces warm skin temperatures. Variations in temperature between different parts of the body may indicate vasoconstriction or vascular disease in the affected extremities. A more extensive examination includes femoral, popliteal, and posterior tibial pulses. Edema of the left lower extremity would cause the nurse to assess the pulses on the left leg and not necessarily the brachial and carotid pulses.Respiratory rate does not influence pulse assessment.

The nurse is providing care to a patient who is diagnosed with rhinosinusitis and prescribed fluticasone. The nurse recognizes that the rationale for this medication is which of the following actions?1. Decreasing sinus drainage 2. Decreasing inflammation 3. Decreasing viscosity of secretions 4. Decreasing risk of infection

2 : Antihistamines decrease rhinorrhea.Intranasal corticosteroids, like fluticasone, decrease sinus inflammation. Normal saline irrigation and mucolytics decrease viscosity of secretions. Antibiotics may be indicated if bacterial infection is suspected in patients with rhinosinusitis.

The nurse develops the nursing diagnosis "Ineffective Airway Clearance related to increased mucus production secondary to chronic inflammation of small airways." For which disorder is this nursing diagnosis most relevant? 1. Asthma 2. Chronic bronchitis 3. Cystic fibrosis 4. Emphysema

2 : Asthma is a chronic lung disease characterized by an intermittent, reversible airway obstruction resulting from inflammation of the lung's airways and a tightening of the muscles that surround the airways. The condition affects the bronchial airways, not the alveoli. 2. Chronic bronchitis affects the small airways and is defined as the presence of cough and sputum production for at least 3 months in each of 2 consecutive years. The chronic airflow limitation is caused by a mixture of small airways disease and destruction of the lung tissue. 3. Cystic fibrosis is a genetic disease of the exocrine glands (glands that secrete hormones into ducts also called duct glands) such as the sweat, salivary, or pancreas glands. It is a multisystem disease that produces increased amounts of thick mucus in the respiratory, gastrointestinal (GI), and reproductive systems. 4. Emphysema causes loss of lung elasticity leading to hyperinflation of the alveoli. The small airways collapse prematurely causing trapping of air in the alveoli and subsequent distention. Carbon dioxide cannot leave the alveoli, and oxygen cannot enter, resulting in an ineffective exchange of oxygen and carbon dioxide.

The nurse notes that a patient is scheduled for a brain natriuretic peptide level to be drawn. What patient teaching should the nurse prepare for this patient? 1. Low-fat diet 2. Signs of heart failure 3. Symptoms of a heart attack 4. Lung versus heart problems

2 : Brain natriuretic peptide does not measure the amount of lipids in the blood. A low-fat diet would not be applicable for this level.Brain natriuretic peptide is released from overstretched ventricular tissue. Elevations are an indicator of heart failure. Brain natriuretic peptide does not determine amounts of cardiac tissue damage. Brain natriuretic peptide does not discern between lung or heart problems.

The nurse recognizes which patient at greatest risk for chronic rhinosinusitis? 1. Female patient with history of otorrhea 2. Female patient with history of nasal polyps 3. Male patient who has a tonsillectomy at age 12 4. Male patient with history of nasal trauma

2 : Chronic rhinosinusitis is more common in females, but there is no association with otorrhea. Chronic rhinosinusitis is more common in females, and the presence of nasal polyps is seen in 19% to 36% of patients. This patient has two risk factors—female gender and history of nasal polyps. Chronic rhinosinusitis is more common in females. Chronic rhinosinusitis is more common in females.

The nurse monitors for which clinical manifestation in the patient admitted with tuberculosis? 1. Greenish-colored sputum 2. Rust-colored sputum 3. Whitish-colored sputum 4. Yellowish-colored sputum

2 : Green- and yellow-colored sputum is consistent with bacterial infections. Blood-tinged or rust-colored sputum is present as a result of the destruction of lung parenchyma tissue.Secretions that are white are consistent with viral infections and often associated with influenza. Green- and yellow-colored sputum is consistent with bacterial infections.

The nurse is conducting a respiratory assessment for a patient who is diagnosed with asthma. Which assessment finding does the nurse monitor for with this diagnosis? 1. Hemoptysis 2. Dry cough 3. Productive cough 4. Coarse crackles

2 : Hemoptysis often occurs with tuberculosis and is not typically observed in patients with asthma.A dry cough may be from asthma, a viral infection, or seasonal allergies.A productive cough may occur in patients with disorders that result in thickened secretions such as pneumonia and tuberculosis. Coarse crackles are associated with fluid or secretions in lower airways and may be auscultated in patients with chronic obstructive pulmonary disease (COPD), pneumonia, and pulmonary edema.

In administering bronchodilator therapy to a patient with asthma, the nurse correlates the effectiveness of this classification of medication to which mechanism of action? 1. Bronchodilators widen the airways because they act on the parasympathetic nervous system. 2. Bronchodilators widen the airways because they stimulate the sympathetic nervous system. 3. Bronchodilators decrease inflammation of the airways. 4. Bronchodilators decrease the production of mucus that narrows the airways.

2 : Leukotriene receptor antagonists are another class of drugs that may be used to enhance asthma control if the usual medications are not effective. They are not steroids; they inhibit the leukotriene-mediated inflammatory process. During the fight-or-flight response, beta2-adrenergic receptors of the sympathetic nervous system are stimulated, the bronchiolar smooth muscle relaxes, and bronchodilation occurs. Corticosteroids and leukotrienes decrease the inflammatory response associated with airway.Anti-inflammatories, typically in the form of inhaled corticosteroids, reduce mucus production.

The nurse provides dietary teaching for a patient who is being treated for laryngeal cancer. Which patient statement indicates a need for additional information? 1. "I should eat foods that are soft and easy to swallow." 2. "I should eat three large high-calorie meals each day." 3. "I will try to drink a liquid nutritional supplement each day." 4. "I will avoid spicy foods as they may increase my pain with swallowing."

2 : Nutrition management for patients with laryngeal cancer includes consuming easy to eat, soft, non-spicy, non-acid-containing foods and small, frequent feedings. This patient statement indicates the need for additional teaching. Eating small, frequent meals throughout the day will assist in ensuring adequate intake. Liquid nutritional supplements may be added to the diet of the patient with laryngeal to increase caloric intake. Nutrition management for patients with laryngeal cancer includes consuming easy to eat, soft, non-spicy, non-acid-containing foods and small, frequent feedings.

Which prescribed first-line therapy does the nurse include in a teaching session for a patient who wants to quit smoking? 1. The use of oral clonidine 2. The use of nicotine patches 3. The importance of taking daily walks 4. The importance of behavioral therapy

2 : Oral clonidine is a second-line therapy for smoking cessation. 2 First-line medication therapy includes nicotine replacement therapies (nicotine gum, patches, or lozenges) and non-nicotinic medications (varenicline [Chantix] or bupropion [Zyban]). 3 Increasing physical activity decreases cravings and symptoms of anxiety, depression, and stress that may be experienced during smoking cessation. However, this is not a prescribed first-line therapy. 4 Providing support increases the likelihood of success and can be in the form of group or individual counseling or telephone conversations. However, this is not a prescribed first-line therapy.

The nurse correlated respiratory acidosis in a patient with which arterial blood gas result? 1. pH 7.50 2. PaCO2 50 mm Hg 3. PaO2 80 mm Hg 4. HCO3- 20 mEq/L

2 : PaCO2 less than 35 mm Hg and pH greater than 7.45 correlate with respiratory alkalosis.PaCO2 greater than 45 mm Hg and pH less than 7.35 correlate with respiratory acidosis. This is a normal PaO2 and does indicate an acid-base imbalance. Abnormal or low PaO2 levels indicate hypoxemia.HCO3- is the metabolic component of the arterial blood gas result. Normal range is 22 to 26 mEq/L. This is a low value that is consistent with metabolic acidosis.

The nurse provides education to a patient who is prescribed an antiviral medication, oseltamivir (Tamiflu), for the treatment of influenza. Which patient statement indicates to the nurse a correct understanding of the administration of this medication? 1. "This medication is administered by an IV." 2. "This medication is administered by mouth." 3. "This medication is administered by injection." 4. "This medication is administered by inhalation."

2 : Peramivir (Rapivab), not oseltamivir, is administered intravenously.This statement indicates a correct understanding for the administration of oseltamivir.This medication is administered orally, not by injection.Zanamivir (Relenza), not oseltamivir, is administered by inhalation.

The nurse is caring for a patient who is admitted to the unit with tuberculosis (TB). The patient is placed in isolation. To protect the caregivers and other patients on the unit, which type of isolation room is most appropriate? 1. Single-door room with positive air flow (air flows out of the room) 2. Isolation room with an anteroom and negative air flow (air flows into the room) 3. Isolation room with an anteroom and normal airflow 4. Single-door room with normal airflow

2 : Positive flow rooms are used for those patients who are immunosuppressed so that microorganisms from the unit are not drawn into the room. Patients with airborne infections such as meningococcemia, severe acute respiratory syndrome (SARS), or tuberculosis are placed in an isolation room with an anteroom and negative pressure airflow. Air flows into the room and is vented in a special manner to prevent the organism from entering the rest of the unit. Single-door isolation with normal airflow might be used for a patient with droplet or wound infection.Single-door rooms are not equipped to have positive or negative airflow.

Which patient does the nurse assess to have the greatest risk for obstructive sleep apnea? 1. 35-year-old obese female with a history of type 2 diabetes 2. 42-year-old obese male with history of cigarette smoking 3. 55-year-old obese female who works as an administrative assistant 4. 68-year-old obese male who worked in a chemical plant for 45 years

2 : Risk factors for developing obstructive sleep apnea (OSA) include male gender, obesity, cigarette smoking, alcohol use, age between 40 and 65, craniofacial or upper airway soft tissue abnormalities, menopause, atrial fibrillation, nocturnal dysrhythmias, type 2 diabetes mellitus (DM), heart failure, and pulmonary hypertension. This patient has two risk factors (obesity and history of type 2 DM). 2 Risk factors for developing OSA include male gender, obesity, cigarette smoking, alcohol use, age between 40 and 65, craniofacial or upper airway soft tissue abnormalities, menopause, atrial fibrillation, nocturnal dysrhythmias, type 2 diabetes mellitus, heart failure, and pulmonary hypertension. This patient has four risk factors (age between 40 and 65, male gender, obesity, and history of tobacco use). 3 4 Risk factors for developing OSA include male gender, obesity, cigarette smoking, alcohol use, age between 40 and 65, craniofacial or upper airway soft tissue abnormalities, menopause, atrial fibrillation, nocturnal dysrhythmias, type 2 diabetes mellitus, heart failure, and pulmonary hypertension. This patient has two risk factors (age between 40 and 65 and obesity). Risk factors for developing OSA include male gender, obesity, cigarette smoking, alcohol use, age between 40 and 65, craniofacial or upper airway soft tissue abnormalities, menopause, atrial fibrillation, nocturnal dysrhythmias, type 2 diabetes mellitus, heart failure, and pulmonary hypertension. This patient has one risk factor (male gender).

The nurse is providing care to a patient who is diagnosed with rhinosinusitis and prescribed azelastine. The nurse recognizes that the rationale for this medication is which of the following actions? 1. Decreasing fever 2. Decreasing inflammation 3. Decreasing nasal drainage 4. Decreasing viscosity of secretions

2 : Saline is a nasal spray; however, saline is not an antihistamine. Azelastine is an antihistamine nasal spray.Fluticasone is a corticosteroid nasal spray.Oxymetazoline is a decongestant nasal spray.

In reviewing the prescribed medications for a patient being treated for rhinitis, the nurse correlates patient complaints of sleepiness, dry mouth eyes and mouth, and constipation to which medication? 1. Alavert 2. Benadryl 3. Claritin 4. Flonase

2 : Sedation, performance impairment, and anticholinergic effects such as dry mouth and eyes, constipation, and increased heart rate are common side effects of first generation antihistamines. Loratadine (Alavert, Claritin) is a second generation antihistamine. Sedation, performance impairment, and anticholinergic effects such as dry mouth and eyes, constipation, and increased heart rate are common side effects of first generation antihistamines. Diphenhydramine (Benadryl) is a first generation antihistamine. 3 Sedation, performance impairment, and anticholinergic effects such as dry mouth and eyes, constipation, and increased heart rate are common side effects of first generation antihistamines. Loratadine (Claritin, Alavert) is a second generation antihistamine. Fluticasone (Flonase) is a corticosteroid nasal spray.

The nurse is providing education to the patient who is receiving treatment for laryngeal cancer. Which patient statement regarding nutrition requires further education from the nurse? 1. "I will eat small, frequent meals to ensure I get enough calories each day." 2. "I can eat any type of food as long as it is easy to swallow." 3. "Liquid supplements are easy to swallow and will increase my caloric intake." 4. "I will eat foods that taste good and are easy to swallow to get enough calories each day."

2 : Small, frequent meals throughout the day ensure an adequate caloric intake. Although it is important to eat foods easy to swallow, the patient needs to limit consumption of spicy and acidic foods.Liquid supplements are easy to swallow and increase the patient's caloric intake when used in addition to solid foods. The patient is encouraged to eat foods that taste good and are easy to eat and swallow to ensure an adequate caloric intake.

Which statements by a patient with asthma indicates effective teaching? 1. "We'll use the fireplace to keep the house warm in the winter." 2. "We will replace the carpet in our bedroom with tile." 3. "We'll make sure that we dust our plants frequently." 4. "We're glad we can keep our dog."

2 : Smoke from fireplaces should be eliminated. Control of dust in the bedroom is an important aspect of environmental control for asthma management, and replacing the carpeting in the bedroom with tile flooring will reduce dust. Plants are often an allergen that can induce symptoms of asthma; therefore, this is not appropriate.When possible, pets and plants should not be kept in the home.

While auscultating a patient's heart rate the nurse hears scratching sounds. What is most likely causing this sound? 1. Epicardium adhering to the heart surface 2. Inflammation of the pericardium 3. Parietal pericardium adhering to the sternum 4. Endocardium adhering to the heart chambers

2 : The epicardium or visceral pericardium covers the heart surface. Between the two layers of the heart is a pericardial cavity containing serous fluid that provides a lubricant that allows the heart to beat without friction. A friction rub is described as a scratching or grating sound heard both during systole and diastole. The sound is produced by inflammation of the pericardium. It is diagnostic for pericarditis and is referred to as the pericardial friction rub. The parietal pericardium is the outer layer of the heart. It is a tough fibrous layer that does not adhere to the sternum.The endocardium is supposed to adhere to the heart chambers.

Patients with asthma initially develop respiratory alkalosis. Which arterial blood gas value is consistent with this acid-base disorder? 1. pH 7.32 2. PaCO2 30 mm Hg 3. PaO2 78 mm Hg 4. SpO2 90%

2 : The pH is elevated in respiratory alkalosis. 2. Initially respiratory alkalosis develops due to hyperventilation (decreased PaCO2). As respiratory difficulty increases, respiratory acidosis may develop (increased PaCO2). This PaCO2 is low, consistent with respiratory alkalosis. 3. This PaO2 indicates very mild hypoxemia, which is not unusual in patients with chronic obstructive pulmonary disease (COPD). 4. This SpO2 is within the normal range.

Which statement by the patient recently diagnosed with an influenza infection indicates that teaching was effective? 1. "I am infectious for 4 days." 2. "I am infectious for up to 10 days." 3. "I am not infectious now that I am on antibiotics." 4. "I am not infectious unless I cough up a lot of secretions."

2 : The patient is infectious for up to 7 to 10 days, not 4 days. Virus shedding usually ends 2 to 5 days after symptoms first appear; therefore, it is important to remember that individuals are infectious for up to 7 to 10 days. The patient is still infectious for up to 7 to 10 days even on antibiotics. Because influenza is an airborne pathogen, it does not require a cough to spread.

The nurse provides education to a patient who is prescribed an aerochamber for the treatment of asthma. Which patient statement indicates the need for additional teaching? 1. "I will place the mouthpiece over my nose and mouth." 2. "I should hear a whistling sound as I inhale the medication." 3. "I should wait a few seconds before I administer the second dose." 4. "I will hold my breath for 10 seconds after inhaling the medication."

2 : This patient statement indicates a correct understanding of the information presented for proper use of an aerochamber. When using the aerochamber, the patient is instructed to take a deep, slow breath in (a whistling sound indicates the patient is breathing in too quickly). This patient statement indicates the need for additional teaching. This patient statement indicates a correct understanding of the information presented for proper use of an aerochamber.This patient statement indicates a correct understanding of the information presented for proper use of an aerochamber.

The nurse is caring for a patient who has developed rhinitis medicamentosa. The nurse correlates this finding to which treatment used to in the management of rhinosinusitis? 1. Intranasal corticosteroids 2. Topical decongestants 3. Normal saline irrigation 4. Oral antibiotics

2 : Topical decongestant sprays, not intranasal corticosteroids (prescribed to decrease inflammation), may result in rhinitis medicamentosa. Topical decongestant sprays if used for more than 3 days may result in rebound nasal congestion when discontinued. This is also referred to as rhinitis medicamentosa. Normal saline irrigation does not cause rhinitis medicamentosa and does not dry the nasal mucosa.Oral antibiotics are not related to rhinitis medicamentosa.

The nurse is assessing a patient who is diagnosed with tuberculosis. Which assessment finding supports this diagnosis? 1. Wheezing 2. Hemoptysis 3. Pleural friction rub 4. Slightly whitish sputum

2 : Wheezing is the term used to describe the musical sounds auscultated during assessment and indicate some degree of airway obstruction that occurs with asthma and emphysema. Tuberculosis is characterized by hemoptysis, which is the term for coughing up of blood or blood-tinged sputum from the respiratory tract. Pleural friction rubs are described as grating or squeaking sounds auscultated over the chest and are caused by inflammation in the pleural space usually associated with pneumonia, pleurisy, or lung cancer. Clear, slightly whitish, and viscous sputum are often normal findings.

The nurse is assessing a patient who is diagnosed with tuberculosis. Which assessment finding correlates with this diagnosis? 1. Wheezing 2. Hemoptysis 3. Yellowish sputum 4. Slightly whitish sputum

2 : Wheezing is the term used to describe the musical sounds auscultated during assessment and indicates some degree of airway obstruction that occurs with asthma and emphysema. Tuberculosis is characterized by hemoptysis, which is the term for coughing up of blood or blood-tinged sputum from the respiratory tract.Yellowish sputum is associated with bacterial infections.Clear, slightly whitish, and viscous sputum are often normal findings.

The nurse recognizes which medication is indicated to treat community-acquired pneumonia in the patient who was previously healthy and has not received antibiotics in the last 3 months? 1. β-Lactam antibiotics 2. Macrolides 3. Respiratory fluoroquinolones 4. Vancomycin

2 : β-Lactam antibiotics and respiratory fluoroquinolones are indicated for the treatment of all inpatients with a community-acquired pneumonia (CAP) Macrolides are indicated in the treatment of CAP in patients who were previously healthy and not treated with antibiotics in the previous 3 months. β-Lactam antibiotics and respiratory fluoroquinolones are indicated for the treatment of all inpatients with a community acquired pneumonia. Vancomycin is used to treat methicillin-resistant Staphylococcus aureus community-acquired pneumonia.

The nurse documents a patient's heart sounds. Which descriptors does the nurse use for intensity of heart sounds? Select all that apply. 1. High 2. Faint 3. Loud 4. Harsh 5. Medium

2,3 This is incorrect. High, medium, and low are descriptors for the pitch, not the intensity or heart sounds. 2 This is correct. Descriptors for the intensity of the patient's heart sounds include faint, loud, and quiet. 3 This is correct. Descriptors for the intensity of the patient's heart sounds include faint, loud, and quiet. 4 This is incorrect. Blowing or harsh are descriptors the nurse uses to describe the quality of heart sounds. 5 This is incorrect. High, medium, and low are descriptors for the pitch, not the intensity or heart sounds.

In reviewing diagnostic results for a patient, which values are consistent with a diagnosis of atherosclerosis? Select all that apply. 1. Decreased homocysteine levels 2. HDL-C 30 mg/dL 3. LDL-C 120 mg/dL 4. Triglycerides 175 mg/dL 5. HgbA1c 5%

2,3,4: 1. This is incorrect. High serum levels of homocysteine may block production of nitric oxide on the vascular endothelium, making the cell walls less elastic and permitting plaque to build up. 2. This is correct. Low levels of high-density lipoprotein cholesterol (HDL-C) indicate an increased risk for atherosclerosis. The target HDL-C level is greater than 40 mg/dL. 3. This is correct. High levels of low-density lipoprotein cholesterol (LDL-C) indicate an increased risk for atherosclerosis. The target LDL-C level is less than 100 mg/dL for healthy adults. 4. This is correct. Triglyceride levels may be elevated with atherosclerosis. Elevated triglycerides are considered a marker for other lipoproteins. A level of 150 mg/dL or greater indicates hypertriglyceridemia. 5. This is incorrect. Hyperglycemia is a risk for the development of atherosclerosis. A hemoglobin A1c (HgbA1c) of greater than 7% may indicate poor glycemic control.

Which statements by a 70-year-old patient about elective surgery for repair of an abdominal aortic aneurysm indicate that teaching was effective? Select all that apply. 1. "The size of my aneurysm is not a factor." 2. "I have a greater risk of rupture because I have hypertension." 3. "Because my aneurysm is 7 cm, the risk of rupture is greater that the risk of dying during surgery." 4. "My risk of rupture is greater because my aneurysm is 7 cm wide." 5. "My age is really not a consideration at this time."

2,3,4: 1. This is incorrect. Aneurysm rupture is more likely in aneurysms that are greater than 6 cm. 2. This is correct. Elective surgery needs to be considered in relation to risks and benefits. Rupture is likely with coexisting hypertension and with aneurysms more than 6 cm wide. 3. This is correct. Rupture is likely with coexisting hypertension and with aneurysms more than 6 cm wide. At this point, the risk of rupture is greater than the risk of death during surgical repair. 4.This is correct. Aneurysm rupture is more likely in aneurysms that are greater than 6 cm. 5. This is incorrect. Because most abdominal aortic aneurysms occur in patients between 60 and 90 years of age, the risks and benefits need to be discussed with the patient in order to make an informed decision regarding elective surgery.

In completing a physical assessment in an 80-year old male, the nurse correlates which findings to age-related changes to the hematological system? Select all that apply. 1. Elevated B and T lymphocytes 2. Hemoglobin (Hgb) 16.5 g/dL 3. Hematocrit (Hct) 50% 4. Decreased iron binding 5. Decreased erythrocyte sedimentation rate (ESR)

2,3,4: 1. This is incorrect. Both T- and B-cell levels decrease with age. 2. This is correct. This is a normal hemoglobin for a male, and there is no decline in either hemoglobin or hematocrit with age. 3. This is correct. This is a normal hematocrit for a male, and there is no decline in either hemoglobin or hematocrit with age. 4. This is correct. There is a decrease in iron binding associated with aging. 5. This is incorrect. An increase in the erythrocyte sedimentation rate is an age- related change of the hematological system.

A nurse educator is teaching a group of patients about prevention of sickle cell crises. What should the nurse instruct about the precipitating factors that could contribute to a sickle cell crisis? Select all that apply. 1. Increased fluid intake 2. Altitude 3. Fever 4. Vomiting 5. Regular exercise

2,3,4: 1. This is incorrect. Regular exercise and increased fluid intake are recommended activities that will not contribute to a sickle cell crisis. 2. This is correct. Fever, vomiting, and altitude are some of the precipitating factors that contribute to a sickle cell crisis. 3. This is correct. Fever, vomiting, and altitude are some of the precipitating factors that contribute to a sickle cell crisis. 4. This is correct. Fever, vomiting, and altitude are some of the precipitating factors that contribute to a sickle cell crisis. 5. This is incorrect. Regular exercise and increased fluid intake are recommended activities that will not contribute to a sickle cell crisis.

The nurse correlates which of the following assessment findings to age-related changes of the respiratory system? Select all that apply. 1. Decreased airway reactivity 2. Decreased chest compliance 3. Decreased intercostal strength 4. Increased risk for hypocapnia 5. Increased neutrophils in respiratory tissue

2,3,5 This is incorrect. The airways of older adults are more reactive than those of younger adults. 2 This is correct. Kyphosis and osteoporosis of the thoracic vertebrae cause a decrease in chest wall compliance. 3 This is correct. The decrease in function and strength of the intercostals and diaphragm increases inspiratory effort to maintain adequate ventilation.4 This is incorrect. Older adults have an increased potential for hypercapnia and hypoxia related to age-related changes that depress the cough reflex and ventilatory response. 5 This is correct. There is an increase in neutrophils, along with a decrease in macrophages that create chronic inflammation of the lung tissue in older adults.

The nurse is preparing material about peripheral artery disease (PAD) for a community fair. What should the nurse include about modifiable risk factors for the disease? Select all that apply. 1. History of chronic obstructive pulmonary disease 2. Smoking 3. Hypertension 4. Family history of cardiovascular disease 5. Sedentary lifestyle

2,3,5: 1. This is incorrect. Chronic obstructive pulmonary disease is not a risk factor for PAD. 2. This is correct. Smoking is a modifiable risk factor for PAD. 3. This is correct. Hypertension is a modifiable risk factor for PAD. 4. This is incorrect. Family history is a nonmodifiable risk factor for PAD. 5. This is correct. Sedentary lifestyle is a modifiable risk factor for PAD.

Which statements about lung cancer indicate that teaching was effective? Select all that apply. 1. "A reclining position allows full lung expansion." 2. "Controlled breathing reduces shortness of breath." 3. "I will pace myself when doing things with my family." 4. "I will eat three large meals a day to ensure I get enough calories." 5. "Pain medication administered around the clock will improve my comfort."

2,3,5: 1. This is incorrect. This patient statement indicates a need for additional education. An upright position (e.g., semi-Fowler's) allows for full lung expansion. 2. This is correct. This patient statement indicates a correct understanding of the information presented. 3. This is correct. This patient statement indicates a correct understanding of the information presented. 4. This is incorrect. This patient statement indicates a need for additional education. Small, frequent meals decrease the work of breathing with less impact on the diaphragm. 5. This is correct. This patient statement indicates a correct understanding of the information presented.

The nurse correlates which pathophysiological processes to Virchow's triad? Select all that apply. 1. Vasodilation 2. Stasis of blood flow 3. Hypercoagulability 4. Leukocytosis 5. Endothelial injury

2,3,5: 1. This is incorrect. Vasodilation is not a factor part of Virchow's triad and would not increase the risk of venous formation. 2. This is correct. Virchow's triad describes the factors implicated in the formation of a venous thrombosis: decreased flow rate of the blood or stasis of blood flow, damage to the blood vessel wall or endothelial injury, and an increased tendency of the blood to clot (hypercoagulability). 3. This is correct. Virchow's triad describes the factors implicated in the formation of a venous thrombosis: decreased flow rate of the blood or stasis of blood flow, damage to the blood vessel wall or endothelial injury, and an increased tendency of the blood to clot (hypercoagulability). 4. This is incorrect. Leukocytosis (elevated white blood cell count) is not a factor of Virchow's triad and would not increase risk of venous formation 5. This is correct. Virchow's triad describes the factors implicated in the formation of a venous thrombosis: decreased flow rate of the blood or stasis of blood flow, damage to the blood vessel wall or endothelial injury, and an increased tendency of the blood to clot (hypercoagulability).

Which first-line medications does the nurse include in a teaching session for a patient who wants to quit smoking? Select all that apply. 1. Clonidine 2. Bupropion 3. V arenicline 4. Nortriptyline 5. Nicotine gum

2,3,5: 1 This is incorrect. Clonidine is a second-line, not first-line, drug for smoking cessation. 2 This is correct. Bupropion is a first-line drug for smoking cessation. 3 This is correct. Varenicline is a first-line drug for smoking cessation. 4 This is incorrect. Nortriptyline is a second-line, not first-line, drug for smoking cessation. 5 This is correct. Nicotine gum is a first-line drug for smoking cessation.

In preparing an educational session related to obstructive sleep apnea, the nurse includes which risk factors? Select all that apply. 1. Female gender 2. Male gender 3. Obesity 4. Age older than 655. Type 2 diabetes mellitus

2,3,5: 1 This is incorrect. Males have a higher risk than females for obstructive sleep apnea (OSA). 2. This is correct. Risk factors for OSA include male gender, obesity, history of cigarette smoking, alcohol use, ages between 40 and 65, craniofacial/upperairway abnormalities, and menopause. Additional factors include history or atrial fibrillation, nocturnal dysrhythmias, type 2 DM, heart failure, and pulmonary hypertension. 3. .This is correct. Risk factors for OSA include male gender, obesity, history of cigarette smoking, alcohol use, ages between 40 and 65, craniofacial/upper airway abnormalities, and menopause. Additional factors include history or atrial fibrillation, nocturnal dysrhythmias, type 2 DM, heart failure, and pulmonary hypertension. 4. This is incorrect. There is an increased risk factor for OSA in individuals ages 40 to 65. 5. .This is correct. Risk factors for OSA include male gender, obesity, history of cigarette smoking, alcohol use, ages between 40 and 65, craniofacial/upper airway abnormalities, and menopause. Additional factors include history or atrial fibrillation, nocturnal dysrhythmias, type 2 DM, heart failure, and pulmonary hypertension.

A patient is with a severe influenza infection is placed on droplet precautions. What actions by the nurse are relevant for this type of isolation? Select all that apply. 1. N95 mask 2. Surgical mask 3. Negative airflow room 4. Private room 5. Gown and gloves

2,4,5 This is incorrect. N95 mask respirators, negative airflow rooms, and/or high-efficiency particulate air (HEPA) filtered rooms are required for airborne precautions. Patients with tuberculosis (TB) require airborne precautions. This is correct. Patients are placed on droplet precautions to avoid viral transmission. Personal protective equipment required includes mask, gown, gloves, and eye protection if there is a risk of splash of body fluids. The patient should wear a mask when outside the room. Visitors should wear a mask while in the room. A private room is desirable unless patients with similar infections are cohorted. This is incorrect. N95 mask respirators, negative airflow rooms, and/or HEPA filtered rooms are required for airborne precautions. Patients with TB require airborne precautions. This is correct. Patients are placed on droplet precautions to avoid viral transmission. Personal protective equipment required includes mask, gown, gloves, and eye protection if there is a risk of splash of body fluids. The patient should wear a mask when outside the room. Visitors should wear a mask while in the room. A private room is desirable unless patients with similar infections are cohorted. 5 This is correct. Patients are placed on droplet precautions to avoid viral transmission. Personal protective equipment required includes mask, gown, gloves, and eye protection if there is a risk of splash of body fluids. The patient should wear a mask when outside the room. Visitors should wear a mask while in the room. A private room is desirable unless patients with similar infections are cohorted.

The nurse is caring for an older adult patient with hemolytic anemia. When planning care for this patient, which should the nurse take into consideration regarding this diagnosis? Select all that apply. 1. It causes the red blood cells to be microcytic. 2. It is associated with an increase in the reticulocyte count. 3. It is the result of blood loss. 4. It is a result of the premature destruction of red blood cells. 5. It always requires treatment with folic acid

2,4,5: 1. This is incorrect. Hemolytic anemias are normocytic (red blood cells are normal size), not microcytic. 2. This is correct. Hemolytic anemia is more common with aging and is caused by the premature destruction of the red blood cells. The normal life span of a red blood cell is 120 days. There is an increase in the reticulocyte (immature red blood cells) count because they are released early from the bone marrow to compensate. 3. This is incorrect. It is not associated with blood loss. 4. This is correct. Hemolytic anemia is more common with aging and is cause by the premature destruction of the red blood cells. 5. This is correct. Hemolytic anemia is more common with aging and is caused by the premature destruction of the red blood cells. The normal life span of a red blood cell is 120 days. All hemolytic anemias require treatment with folic acid because this vitamin is consumed by the increased bone marrow production of red blood cells in response to the anemia.

In reviewing teaching about use of the peak flow meter in a patient with asthma, which statements by the patient indicate understanding of correct use? Select all that apply. 1. "I will lie flat in the bed." 2. "I take a deep breath before placing the mouthpiece in my mouth." 3. "I breathe out slowly and continuously." 4. "I should repeat three times and the numbers should be close." 5. "I blow out in one fast, hard blast."

2,4,5: 1. This is incorrect. The patient needs to be standing up straight when using the peak flow meter. 2. This is correct. The patient takes a deep breath, then puts the mouthpiece of the peak flow meter into the mouth, and then tightly closes the lips around the mouthpiece. 3. This is incorrect. The patient blows out as hard and as quickly as possible in one breath, until nearly all of the air is out of the lungs. 4 This is correct. The patient should repeat the entire routine three times. 5 This is correct. The patient blows out as hard and as quickly as possible in one breath, until nearly all of the air is out of the lungs.

The nurse is preparing a teaching tool about the cardiac electrical conduction system. In which order should the nurse explain the route of the action potential? (Enter the number of each step in the proper sequence; do not use punctuation or spaces. Example: 1234) 1. Impulse travels to the bundle of His 2. Sinoatrial node fires in the right atrium 3. Impulse extends through Purkinje fibers 4. Impulse travels through bundle branches 5. Impulse travels to the atrioventricular node 6. Impulse spreads through atrial myocardium

2,6,5,1,4,3 The action potential begins in the sinoatrial node, which fires in the right atrium. Then the impulse spreads through the atrial myocardium and travels to the atrioventricular node. It then travels to the bundle of His, through the bundle branches, and extends through the Purkinje fibers.

Which statement to the nurse by the patient about a planned bone marrow biopsy indicates the need for further teaching? 1. "I may hear a crunching sound as the needle punctures the bone." 2. "I can receive sedation for this test." 3. "I will need to be on strict bedrest for at least 24 hours." 4. "I may feel some pressure during the procedure."

3

A patient diagnosed with aplastic anemia is admitted to the hospital. In teaching the patient and family about this disease process, what information does the nurse include?1. "Aplastic anemia causes a proliferation of white blood cells." 2. "Aplastic anemia is characterized by abnormally shaped red blood cells." 3. "Aplastic anemia is caused by the bone marrow producing inadequate cells." 4. "Aplastic anemia is a disorder that occurs after a viral illness."

3 : 1. All blood cells, not just white blood cells, are affected by aplastic anemia. 2. Aplastic anemia does not cause abnormally shaped red blood cells; this is a description of sickle cell disease. 3. In aplastic anemia, the bone marrow does not produce sufficient numbers of circulating blood cells. 4. There is no known association between aplastic anemia and viral illness.

The nurse plans care for a patient who is admitted to the hospital for newly diagnosed left-sided heart failure. Which is the priority nursing diagnosis when planning care for this patient? 1. Activity Intolerance 2. Deficient Knowledge 3. Impaired Oxygenation 4. Impaired Tissue Perfusion

3 : 1. Although Activity Intolerance is relevant to this patient's care, particularly in matching activity with oxygen supply, Impaired Oxygenation is the priority. 2. Although the patient will require significant teaching, physiological needs take priority over psychosocial needs, and teaching is a psychosocial need. 3. Impaired oxygenation is the priority when planning care for this patient. Accumulation of fluid in the lungs impairs breathing. Breathing is a higher priority than circulation when using the ABCs (airway, breathing, circulation) to prioritize care. 4. Although this is an appropriate topic to include in the patient's plan of care, it is not the priority. When using the ABCs (airway, breathing, circulation), impaired peripheral perfusion indicates a circulatory need. There is another nursing diagnosis with a higher priority.

Which term does the nurse use when documenting an elevated red blood cell (RBC) count? 1. Anemia 2. Neutropenia 3. Polycythemia 4. Thrombocytopenia

3 : 1. Anemia is the term used to describe a decreased red blood cell (RBC) count. 2. Neutropenia is a term used to describe a decreased white blood cell (WBC) count. 3. Polycythemia is a term used to describe an increased RBC count. 4. Thrombocytopenia is a term used to describe a decreased platelet count.

The nurse conducts a teaching session for a patient who is diagnosed with iron deficiency anemia. Which patient statement indicates to the nurse a need for additional teaching? 1. "I will add beets to my salad." 2. "I will increase my intake of meats." 3. "I will eat an egg each day for breakfast." 4. "I will include dark-green vegetables in my diet."

3 : 1. Beets are a source of iron; therefore, this statement indicates a correct understanding of the information presented. 2. Meats are rich in iron; therefore, this statement indicates a correct understanding of the information presented. 3. Eggs are rich in vitamin B12, not iron; therefore, this statement indicates a need for additional teaching. 4. Dark-green vegetables are rich in iron; therefore, this statement indicates a correct understanding of the information presented.

The nurse is preparing teaching for a patient being treated for coronary artery disease. What dietary information should the nurse emphasize? 1. Restrict carbohydrate intake. 2. Limit calorie intake to less than 1,000/day. 3. Reduce saturated fat and sodium intake. 4. Limit fluid intake.

3 : 1. Carbohydrate restriction is not part of dietary teaching for the patient with coronary artery disease (CAD), and the patient is encouraged to eat fresh fruits and vegetables and whole grains. 2. Maintaining a healthy weight is important, but this is a severely calorie-restricted diet. The major focus is on decreasing saturated fat and sodium intake. 3. A diet that is low in saturated fat and sodium as well as high in fruits, whole grains, and vegetables is important for patients with CAD. 4. There is no indication to limit fluid intake unless the patient has clinical manifestations of heart failure.

The nurse provides care for a patient who is diagnosed with a folic acid deficiency anemia. Which clinical manifestation requires healthcare provider notification? 1. Pallor 2. Eupnea 3. Confusion 4. Tachycardia

3 : 1. Clinical manifestations of anemia are related to incomplete DNA synthesis that affects the formation of heme and the maturation of the red blood cells (RBCs), which causes a decreased hemoglobin level. Pallor is an expected clinical manifestation and is not a symptom necessitating healthcare provider notification. 2. Tachypnea, not eupnea, is a clinical manifestation of folic acid deficiency anemia. Eupnea is the medical term for normal breathing. This manifestation does not require healthcare provider notification. 3. Immediately report any clinical manifestations of fatigue, shortness of breath, dizziness, or confusion. These manifestations indicate poor tissue perfusion requiring a resolution of the anemia through supplementation or blood transfusions. 4. Clinical manifestations of anemia are related to incomplete DNA synthesis that affects the formation of heme and the maturation of the RBCs, which causes a decreased hemoglobin level. Tachycardia is an expected clinical manifestation and is not a symptom necessitating healthcare provider notification.

In administering cryoprecipitate to a patient diagnosed with idiopathic thrombocytopenia purpura, the nurse recognizes which of the following actions as the rationale for this treatment? 1. Increases plasma volume 2. Prevents platelet destruction 3. Provides clotting factors 4. Promotes platelet production

3 : 1. Cryoprecipitate provides clotting factors and does not increase plasma volume. 2. Cryoprecipitate provides clotting factors and has no impact on platelets. 3. Cryoprecipitate (plasma product rich in factor VIII) may be employed to maintain hemostasis as it contains clotting factors. 4. Cryoprecipitate provides clotting factors and has no impact on platelet production.

The nurse is providing discharge teaching for a patient with iron deficiency anemia. The patient has been prescribed ferrous sulfate and has been told to increase the intake of foods that are naturally high in iron. Which patient statement indicates the need for additional teaching? 1. "I will increase my intake of green leafy vegetables." 2. "I will increase my fluid intake while I am taking ferrous sulfate." 3. "I will take my ferrous sulfate tablet on an empty stomach." 4. "I will increase dried beans in my diet."

3 : 1. Good dietary sources of iron include meat (especially red), dark green leafy vegetables (spinach, broccoli, peas), beets, dried beans, iron-fortified breakfast cereals and breads, and Cream of Wheat. 2. Increasing fiber (oatmeal) and fluid intake can also help prevent constipation, which is a side effect of iron supplements. 3. Ferrous sulfate can cause gastric irritation and constipation. Taking it with a meal can help minimize gastrointestinal distress. 4. Good dietary sources of iron include meat (especially red), dark green leafy vegetables (spinach, broccoli, peas), beets, dried beans, iron-fortified breakfast cereals and breads, and Cream of Wheat.

The nurse correlates appropriate secondary polycythemia to which of the following conditions? 1. Renal disease 2. Cancer 3. High altitudes 4. Cold temperatures

3 : 1. Inappropriate secondary polycythemia occurs when the body responds to a stimulus in an inappropriate way by secreting additional erythropoietin, which results in increased red blood cell (RBC) production. This inappropriate erythrocytosis may occur in kidney disease and certain tumors. 2. Inappropriate secondary polycythemia occurs when the body responds to a stimulus in an inappropriate way by secreting additional erythropoietin, which results in increased RBC production. This inappropriate erythrocytosis may occur in kidney disease and certain tumors. 3. Secondary polycythemia occurs in response to the body's hypoxic state and may be considered appropriate or inappropriate. Appropriate secondary polycythemia occurs in situations caused by exposure to environments that have low oxygen tension, such as occurs with high altitudes and heavy smoke. The body compensates by producing more RBCs, thus increasing the oxygen-carrying capacity of the blood. 4. Cold temperatures are not associated with secondary polycythemia.

Which is the priority teaching point for the nurse to include in the discharge instructions for the patient being discharged after treatment for sickle cell crisis? 1. Rapid weaning of pain medications 2. A diet high in protein 3. Adequate hydration 4. Restriction of activities

3 : 1. Rapid weaning is not necessary; reduction of pain medication should proceed at a rate dictated by the patient's pain. 2. A high-protein diet is not necessary; a well-balanced diet should be promoted. 3. Adequate hydration will help prevent further sequestration and crisis. 4. Normal activities are not restricted.

The nurse is assisting the healthcare provider with a bone marrow aspiration and biopsy on a patient who has leukemia. The patient also has thrombocytopenia. On completing of the test, which intervention is a priority for the nurse? 1. Teach the patient about signs of infection. 2. Label and refrigerate the specimen obtained by the provider. 3. Hold pressure on the wound for at least 5 to 10 minutes. 4. Assess for post-procedure pain after the bone marrow biopsy.

3 : 1. Teaching the patient about signs of injection is important, but because of the patient's low platelet count, the risk of bleeding is the higher priority. 2. While labelling the specimen is important, the specimen should be promptly transported to the laboratory. However, the risk of bleeding is the higher priority. Properly label and promptly transport all specimens to the laboratory after the patient is stable. 3. The most important task for the nurse is to prevent bleeding after the biopsy. Holding pressure on the wound for 5 minutes is effective. 4. Although pain assessment is important after a bone marrow biopsy, the risk of bleeding is the higher priority.

The nurse is evaluating a patient's understanding of dietary needs to treat anemia. Which patient statement indicates a need for additional teaching? 1. "I will eat more fruits and vegetables, especially green leafy ones, to get more iron in my diet." 2. "I will need to include more protein foods in my diet such as meats, dried beans, and whole-grain breads." 3. "I will decrease foods high in vitamin C because they decrease the absorption of iron." 4. "I will take vitamins with extra iron in addition to eating a balanced diet with meat to correct my anemia."

3 : 1. The lack of iron is the problem that needs to be addressed. Extra iron is needed to help replace red blood cells and treat the dietary deficiency anemia. Green leafy vegetables will increase iron in the diet. 2. Protein foods such as meats, dried beans, and whole-grain breads do contain iron that will help dietary deficiency anemia. 3. Increasing, not decreasing, foods high in vitamin C will increase absorption of iron. 4. Protein foods such as meats, dried beans, and whole-grain breads do contain iron that will help dietary deficiency anemia.

A nurse is planning care for a patient with sickle cell disease. The nurse should contact the provider about which prescribed intervention? 1. Administer ordered analgesic medications around the clock 2. Place patient in position of comfort 3. Use heat or cold packs as tolerated 4. Support the patient's joints and extremities with pillows

3 : 1. The patient with sickle cell disease who is in a sickle cell crisis will likely have extreme pain. To aid in caring for this patient, the nurse will administer ordered analgesic medications around the clock, place the patient in a position of comfort, and support the patient's joints and extremities with pillows. 2. The patient with sickle cell disease who is in a sickle cell crisis will likely have extreme pain. To aid in caring for this patient, the nurse will administer ordered analgesic medications around the clock, place the patient in a position of comfort, and support the patient's joints and extremities with pillows. 3. The use of heat or cold packs is contraindicated in the sickle cell patient. Ischemic tissue is fragile and has reduced sensation, increasing the risk of burn injury from hot compresses, whereas cold compresses promote sickling. 4. The patient with sickle cell disease who is in a sickle cell crisis will likely have extreme pain. To aid in caring for this patient, the nurse will administer ordered analgesic medications around the clock, place the patient in a position of comfort, and support the patient's joints and extremities with pillows.

Which statement by the patient discharge on warfarin (Coumadin) indicates the need for further teaching? 1. "I can take acetaminophen for pain." 2. "I need to limit intake of green leafy vegetables." 3. "It is okay if I see a little blood in my stool." 4. "I need to take the medications at the same time every day."

3 : 1. This indicates patient understanding as acetaminophen does not impact blood clotting, so it is safe to administer to patients who are on anticoagulants. Patients should not take aspirin products without clearance from the healthcare provider. 2. This statement indicates patient understanding because it is important to limit intake of green leafy vegetables that contain vitamin K (kale, spinach, collard greens, broccoli, okra, cabbage) that may counteract the action of warfarin. 3. This statement indicates the need for further teaching because it is important for the patient to immediately report blood in sputum, emesis, stool, or urine. 4. This indicates patient understanding because it is important that the patient take the medication at the same time every day.

The nurse is reviewing laboratory results for a patient scheduled for a bleeding time test. Which value requires an immediate intervention? 1. White blood cell count of 13.2 103/mm3- 2. Hemoglobin 9 g/L 3. Platelet count of 70,000/mm3 4. Prothrombin time 12 seconds

3 : 1. This is a slightly elevated white blood cell count, but it would not be a contraindication to a bleeding time. 2. This is a low hemoglobin, but this is associated with oxygen carrying capacity of red blood cells, not bleeding. 3. A bleeding time is usually not recommended for a patient with a platelet count of less than 75,000 mm3 because of increased risk for massive bleeding and ineffective clotting. 4. This is within the normal range of prothrombin time (10-13 seconds).

The nurse provides care to a patient who presents with clinical manifestations of an ischemic stroke. Which time frame does the nurse anticipate for the prescribed fibrinolytic medication? 1. Within 4 hours of symptoms 2. Within 8 hours of symptoms 3. Within 12 hours of symptoms 4. Within 16 hours of symptoms

3 : 1. This is not the time frame the nurse anticipates for the administration of the prescribed fibrinolytic medication. 2. This is not the time frame the nurse anticipates for the administration of the prescribed fibrinolytic medication. 3. Fibrinolytics break down formed fibrin clots. The nurse anticipates the administration of prescribed fibrinolytic medication within 12 hours of initial symptoms. 4. This is not the time frame the nurse anticipates for the administration of the prescribed fibrinolytic medication.

In completing a physical assessment on a patient, the nurse notes that the patient has a red, swollen, smooth, shiny and tender tongue. The nurse correlates this finding to which disorder? 1. Thrombocytosis 2. Leukopenia 3. Iron deficiency anemia 4. Vitamin K deficiency

3 : 1. Thrombocytosis is an elevated number of platelets and does not cause these tongue changes 2. Leukopenia is a low white blood cell count and does not cause these tongue changes. 3. A red, swollen, smooth, shiny, and tender tongue (glossitis) may indicate iron deficiency anemia or pernicious anemia. 4. Vitamin K deficiency leads to decreased clotting.

The nurse is evaluating teaching provided to a patient with coronary artery disease. Which patient statement indicates that additional teaching is required? 1. "I will adhere to my smoking cessation plan." 2. "I am to reduce my daily intake of saturated fat." 3. "I can take up to three doses of nitroglycerin 15 minutes apart." 4. "I am to follow the exercise plan for 30 minutes, 5 days a week."

3 : Patients with coronary artery disease should engage in smoking cessation activities.Patients with coronary artery disease should reduce the intake of saturated fat.A nitroglycerin administration regimen includes not exceeding three doses 5 minutes apart.Patients with coronary artery disease should engage in regular exercise for 30 minutes per day 5 days a week.

In administering hemorheological agents to a patient with peripheral arterial disease, the nurse correlates this treatment to which mechanism of action? 1. Anticoagulation 2. Diuretic 3. Platelet aggregation inhibitor 4. Vasodilator

3 : 1. Anticoagulants work by inhibiting clot formation by interference with action of specific clotting factors. 2. Diuretics act on the kidneys to help the body eliminate sodium and water, reducing blood volume. 3. Hemorheological agents decrease blood viscosity by inhibiting platelet aggregation leading to increased blood flow to the affected extremity. 4. Vasodilators help relax/dilate blood vessels by blocking the formation of angiotensin II, a vasoconstrictor.

In admitting a patient with a vascular disorder, the nurse correlates which clinical manifestation to femoral vein thrombosis? 1. Calf pain 2. Tenderness in the foot 3. Thigh swelling to the knee 4. Entire leg swollen and painful

3 : 1. Calf tenderness is associated with a thrombosis of the calf. 2. With femoral vein thrombosis, the pain is more prominent in the thigh and popliteal area, not the foot. 3. In a femoral thrombosis, swelling may extend to the knee. 4. Massive swelling of the entire extremity and pain are associated with an iliofemoral thrombosis.

The nurse is monitoring a patient with peripheral arterial disease (PAD). Which clinical manifestations does the nurse correlate to stage III PAD? 1. Decreased pedal pulses 2. Pain, burning, and leg cramping relieved by rest 3. Pain, numbness and burning in the legs/feet at rest 4. Ulcers and blackened tissue on the toes

3 : 1. In stage I the patient has no complaints of pain but does have decreased pedal pulses. 2. In stage II the patient has muscle pain, burning, and cramping with exercise that is relieved with rest. 3. In stage III the patient has pain at rest, with numbness and burning in the distal extremities. 4. In stage IV the patient experiences ulcers and blackened tissue on the toes, the forefoot, or the heel of the foot.

The nurse reviews the ankle-brachial index for a patient admitted for treatment of peripheral arterial disease (PAD). What is the significance of a value of 0.50? 1. Normal 2. Borderline PAD 3. Mild to moderate PAD 4. Severe PAD

3 : 1. Normal ankle-brachial index is 1 to 1.29. 2. Borderline peripheral arterial disease (PAD) ankle-brachial index is 0.91 to 0.99. 3. Mild to moderate PAD ankle-brachial index is 0.41 to 0.9. 4. Severe PAD ankle-brachial index is 0 to 0.4.

The nurse monitors for which complication in the patient with carotid atheroembolization? 1. Shortness of breath 2. Tachycardia 3. Change in level of consciousness 4. Upper extremity edema

3 : 1. Shortness of breath develops when there is embolism that impacts the pulmonary circulation. 2. Atheroembolization of the carotid artery does not cause tachycardia. 3. With increasing degrees of stenosis in the internal carotid artery, flow becomes more turbulent, increasing the risk of atheroembolization, an embolism from atherosclerotic plaque. Changes in level of consciousness occur if there is impaired circulation distal to the carotid artery, including cerebrovascular arteries. 4. Upper extremity edema is associate with venous obstruction, not carotid embolism.

A patient diagnosed with chronic obstructive pulmonary disease (COPD) has a pulse oximetry reading of 93%, increased red blood and white blood cell count, temperature of 101°F, pulse 100 bpm, respirations 35 bpm, and a chest x-ray examination that showed a flattened diaphragm with infiltrates. Based on this data, which prescription does the nurse question for this patient? 1. Antibiotic therapy 2. Nonsteroidal anti-inflammatory agents 3. Oxygen by nasal cannula at 3 to 4 L/minute 4. Bronchodilators such as an adrenergic stimulating drugs or anticholinergic agents

3 : 1. The order for antibiotic therapy is expected because the patient is febrile with an increase in white blood cells. 2. Nonsteroidal anti-inflammatory agents are commonly ordered to decrease the inflammation and swelling of lung tissues to maximize oxygen and carbon dioxide exchange and to improve symptoms and would be expected for this patient. 3. The nurse should be concerned about the order for oxygen to be provided at 3 to 4 L/minute. This amount of oxygen is too much for a patient with COPD because the patient's breaths are stimulated by a hypoxic drive and this disease process causes the body to retain carbon dioxide. Providing this much oxygen can result in an increase in carbon dioxide levels, leading to respiratory failure. Oxygen for this patient should be at a lower rate, such as 1 to 2 L/minute, with close assessments of the patient's breathing status. 4. Bronchodilators will keep the alveoli open and increase exchange of oxygen and carbon dioxide more effectively and would be expected for this patient.

During a home visit the nurse determines that teaching was effective for the patient recovering from infective endocarditis. Which observation supports this evaluation? 1. Drinking skim milk 2. Measuring radial pulse 3. Using a soft toothbrush 4. Taking anticoagulant daily

3 : Drinking skim milk is appropriate for the patient with coronary artery disease. Measuring radial pulse is appropriate for the patient with heart failure taking medication that affects the pulse rate. For the patient with infective endocarditis, good oral hygiene using a soft toothbrush is essential. Bleeding gums provide a portal of entry for bacteria into the bloodstream. Poor dental hygiene exacerbates the problem. Taking an anticoagulant every day is appropriate for the patient with a mechanical heart valve.

In providing care to a patient diagnosed with atherosclerosis, the nurse correlates the initiation of this disease to which process? 1. Oxidation 2. Inflammatory process 3. Injury to the vessel wall 4. Trapping of low-density lipoproteins

3 : Oxidation occurs after low-density lipoproteins are trapped in the tunica intima. The inflammatory process occurs after an injury to the vessel wall. Atherosclerosis begins with an injury to the endothelium.Low-density lipoproteins become trapped after the inflammatory response begins.

In administering oxygen 2 L via nasal cannula to a patient with coronary artery disease, what does the nurse explain as being the primary purpose of the oxygen? 1. Promotes vessel dilation 2. Prevents clot formation 3. Supports myocardial oxygen demand 4. Decreases respiratory complications

3 : Oxygen does not directly impact vessel dilation. Anticoagulants, not oxygen, prevent clot formation. The patient with coronary artery disease (CAD) is prone to experiencing increased myocardial oxygen consumption. Administering oxygen will help supplement the body's need for oxygen. The primary purpose of oxygen in patients with CAD is to provide supplemental oxygen to the myocardium. Shortness of breath may develop in patients with CAD.

A patient with infective endocarditis is being started on intravenous antibiotics. What should the nurse include when teaching about this medication? 1. "The medication may be changed every few days." 2. "The typical course of this medication is 7 to 10 days." 3. "Expect to continue this medication for 4 to 6 weeks at home." 4. "Because this infection is resilient, you might be taking antibiotics for life."

3 : The medication will not be changed every few days in order to maintain a therapeutic level of the antibiotic.The medication will be provided for 4 to 6 weeks.The standard duration of treatment is 4 to 6 weeks. These patients are often discharged to home on intravenous antimicrobial therapy. The patient will not be on antibiotics for life.

A patient's blood pressure is 140/86 mm Hg; however, previous measurements have been within normal limits. The patient denies any other complaints. Which intervention would be appropriate for this patient? 1. Refer the patient to the emergency department for treatment. 2. Prepare teaching on antihypertensive medications. 3. Schedule an additional measurement in a few weeks. 4. Instruct on the effects of hypertension on major body organs.

3 : 1. Because definitive diagnosis of hypertension requires elevated average readings of two or more blood pressure (BP) measurements on two or more different dates, this patient does not require referral to the emergency department. 2. Preparing teaching material on antihypertensive medications would be premature. 3. The diagnosis of hypertension is made on the basis of the average of two or more properly measured BP readings on two or more office visits. 4. The diagnosis of hypertension has not been made. This teaching would be premature.

The nurse is preparing to auscultate the heart sounds of a patient with mitral valve regurgitation. Which sound should the nurse expect to hear? 1. Rub 2. Click 3. Murmur 4. Atrial gallop

3 : A friction rub is described as a scratching or grating sound heard both during systole and diastole. The sound is produced by inflammation of the pericardium. It is diagnostic for pericarditis and is referred to as the pericardial friction rub. A click is a high-pitched sound heard early in diastole typically caused by mitral valve stenosis. Murmurs are usually caused by turbulent flow through the valves. That turbulence can be caused by regurgitation of blood through an incompetent valve. An atrial gallop can indicate decreased ventricular compliance.

The nurse prepares the patient for which diagnostic procedure that is used to remove pleural fluid for analysis? 1. Lung biopsy 2. Bronchoscopy 3. Thoracentesis 4. Sputum studies

3 : A lung biopsy involves taking a sample of tissue, not fluid, for analysis.A bronchoscopy involves the use of a flexible fiberoptic scope for diagnosis, biopsy, or specimen collection.A thoracentesis is a diagnostic procedure used to remove pleural fluid for analysis or to instill medication.Sputum studies are obtained by expectoration and tracheal suction.

The nurse reads tuberculin skin test (TST) results for a group of patients. Which patient does the nurse document as a positive result in the medical record? 1. The adult patient diagnosed with HIV whose induration is 3 mm. 2. The adult who recently had contact with a person with tuberculosis whose induration is 4 mm. 3. The patient who uses intravenous (IV) drugs whose induration is 11 mm. 4. The adult who recently immigrated from a high-prevalence country whose induration is 8 mm.

3 : A patient diagnosed with HIV requires an induration of 5 mm or greater for the nurse to document a positive result in the medical record. A person who recently had contact with a person with tuberculosis requires an induration of 5 mm or greater for the nurse to document a positive result in the medical record. 3. This patient's result is considered positive, necessitating the nurse to document a positive result in the medical record. The patient who uses intravenous drugs has a positive result for an induration of 10 mm or greater. 4. A tuberculin skin test for a patient with immunosuppression is considered positive when the induration is 5 mm or greater.

The nurse is providing care to a patient diagnosed with emphysema. Which clinical manifestation does the nurse correlate with this diagnosis? 1. Tachycardia 2. Cough 3. Barrel chest 4. Wheezing

3 : Although coughing, wheezing, and tachycardia may also be experienced by a patient diagnosed with chronic obstructive pulmonary disease (COPD), these are not specific to COPD caused by emphysema. 2. Although coughing, wheezing, and tachycardia may also be experienced by a patient diagnosed with COPD, these are not specific to COPD caused by emphysema. 3. Barrel chest occurs because the lungs are chronically overinflated with air, so the rib cage stays partially expanded. 4. Although coughing, wheezing, and tachycardia may also be experienced by a patient diagnosed with COPD, these are not specific to COPD caused by emphysema.

Which is the priority nursing diagnosis for a patient who experiences laryngeal trauma? 1. Ineffective Breathing Pattern 2. Impaired Swallowing 3. Ineffective Airway Clearance 4. Impaired Gas Exchange

3 : Although ineffective breathing pattern may develop in the patient with laryngeal trauma, it is not the priority based on the ABCs (airway, breathing, circulation) for prioritization of care. 2. Although impaired swallowing may develop in the patient with laryngeal trauma, it is not the priority based on the ABCs (airway, breathing, circulation) for prioritization of care. 3. Ineffective Airway Clearance related to edema of the soft tissue of the upper airway is the priority nursing diagnosis when planning care for a patient who experiences a laryngeal trauma. 4. Although impaired gas exchange may develop in the patient with laryngeal trauma, it is not the priority based on the ABCs (airway, breathing, circulation) for prioritization of care.

The nurse is caring for a patient who was recently treated for an empyema that developed secondary to pneumonia. Which procedure does the nurse prepare the patient to decrease recurrence once the empyema has been drained? 1. Bronchoscopy 2. Chest tube insertion 3. Pleurodesis 4. Thoracentesis

3 : An empyema is a collection of purulent material in the pleural space, and a bronchoscopy is used to visualize the bronchioles.The chest tube insertion is the procedure used to drain the empyema, a collection of purulent material in the pleural space. Pleurodesis is the injection of a sclerosing or scarring agent into the pleural space, causing the visceral and parietal pleura to "stick together," and may be performed after the empyema has been resolved to prevent recurrence. Thoracentesis is a procedure to remove fluid from the pleural space but does not decrease chances of recurrence.

In assessing the patient immediately after extubation, which finding requires an immediate intervention? 1. Respiratory rate of 22 2. Adventitious breath sounds 3. Stridor 4. Tachycardia

3 : An increased respiratory rate may develop after extubation but is not as emergent as stridor.Adventitious sounds may be auscultated after extubation, indicating the patient needs to be suctioned, but is not as emergent as stridor. Stridor that develops after extubation indicates acute obstruction of the airway and is a medical emergency that may require reintubation.Tachycardia may develop secondary to stridor and stress associated with a partially obstructed larynx after extubation.

A patient is admitted for evaluation of complaints of difficulty breathing and is scheduled for a sputum study to assist in providing data for which diagnosis? 1. Asthma 2. Lung cancer 3. Bacterial lung infection 4. Chronic obstructive pulmonary disease

3 : Asthma is often diagnosed based on pulmonary function tests.Lung cancer is usually definitively diagnosed based on findings from a biopsy. A sputum study is often used to diagnose bacterial lung infections via a culture and sensitivity analysis. A sputum analysis is not typically used in the diagnosis of chronic obstructive pulmonary disease (COPD), which is diagnosed based on physical assessment findings, chest x-ray examination results, and arterial blood gas results.

Where does the nurse auscultate bronchial vesicular sounds? 1. Neck 2. Trachea 3. First to second intercoastal spaces 4. Peripheral lung fields

3 : Bronchial sounds, described as hollow with loud intensity, are auscultated over the neck and trachea. Bronchial sounds, described as hollow with loud intensity, are auscultated over the neck and trachea.Bronchovesicular sounds, described as tubular with moderate intensity are auscultated from the first to the second intercoastal spaces.Vesicular sounds, described as a gentle breeze with soft intensity, are auscultated below the second and peripheral lung fields.

On assuming care for a patient being treated for bacterial pneumonia, which assessment finding requires immediate attention by the nurse? 1. Fever 2. Productive cough 3. Restlessness 4. Arthralgia

3 : Clinical manifestations of bacterial pneumonia include fever, chills, tachypnea, tachycardia, cough (productive or non-productive), pleuritic pain, fatigue, and myalgias. Dyspnea, particularly at rest, is an indication that the patient's respiratory status is deteriorating. 2. Clinical manifestations of bacterial pneumonia include fever, chills, tachypnea, tachycardia, cough (productive or non-productive), pleuritic pain, fatigue, and myalgias. Dyspnea, particularly at rest, is an indication that the patient's respiratory status is deteriorating. 3. Agitation, restlessness, anxiety, lethargy, and fatigue are the result of decreased tissue perfusion from altered alveolar gas exchange and require immediate action by the nurse. 4. Clinical manifestations of bacterial pneumonia include fever, chills, tachypnea, tachycardia, cough (productive or non-productive), pleuritic pain, fatigue, and myalgias. Dyspnea, particularly at rest, is an indication that the patient's respiratory status is deteriorating.

The nurse correlates which clinical manifestation as the earliest compensatory mechanism in the patient with influenza? 1. Oliguria 2. Tachycardia 3. Tachypnea 4. Fever

3 : Decreased urine output may develop secondary to increased insensitive fluid loss but is not a compensatory mechanism.Tachycardia to raise the heart rate is the second compensatory mechanism for the continued impairment of oxygen delivery.Tachypnea, increased respiratory rate and depth, is the body's first compensatory mechanism to decreased oxygen delivery.Fever occurs as a part of the inflammatory response but is not the earliest sign.

The nurse recognizes that which process occurs as oxygen and carbon dioxide are exchanged at the level of the alveoli. 1. Diffusion 2. Perfusion 3. Respiration 4. Ventilation

3 : Diffusion is a general term that relates to the movement of gases across a membrane but is not as specific as respiration.Perfusion is the movement of oxygenated blood into the tissues by the circulatory system. The process of respiration occurs as oxygen and carbon dioxide are exchanged at the level of the alveoli. Ventilation occurs as air moves into and out of the respiratory system through the process of inspiration and exhalation. During ventilation, the structures of the respiratory system filter and humidify the air entering the system.

When percussing the patient's lung fields, the nurse notes a long, hollow, loud pitched sound over the chest. The nurse uses which term to describe this in the health record? 1. Dull 2. Tympany 3. Resonance 4. Hyperresonance

3 : Dull sounds are medium pitched, thud sounds observed when percussing over the liver, heart, kidney, or diaphragm. Dull sounds may be assessed in patient with pneumonia, atelectasis, or a lung mass. Tympany is a drum-like, loud, empty quality heard over a gas-filled stomach or intestine.Low-pitched sounds heard over normal lungs during percussion indicate resonance. Hyperresonance is a loud, lower-pitched sound heard when percussing hyperinflated lungs, which can occur in patients who are experiencing an acute asthma exacerbation.

The nurse is assessing a patient who is admitted with pulmonary edema and presents with a persistent cough. Which assessment finding is consistent with this diagnosis? 1. Foul-smelling sputum 2. Wheezing 3. Coarse crackles 4. Stridor

3 : Foul-smelling sputum indicates an infection process often seen in patients with pneumonia.Wheezing is associated with inflammation and bronchoconstriction and often observed in patients with asthma or COPD. Coarse rhonchi (rales) are caused by secretions in the lower airway and often observed in patients with pulmonary edema, congestive heart failure, pneumonia, and COPD. Stridor is caused by an obstruction of the throat or upper airway and often observed in patients with laryngitis, epiglottis, and allergic reactions.

The nurse is preparing an educational program pneumonia and includes information about which pathogen that is the major pathogen associated with hospital-acquired pneumonia (HAP)? 1. Haemophilus influenzae 2. Mycoplasma pneumoniae 3. Pseudomonas aeruginosa 4. Streptococcus pneumoniae

3 : Haemophilus influenzae, Mycoplasma pneumoniae, and Streptococcus pneumoniae are examples of common causative organisms for community-acquired, not hospital-acquired, pneumonia. 2 Haemophilus influenzae, Mycoplasma pneumoniae, and Streptococcus pneumoniae are examples of common causative organisms for community-acquired, not hospital-acquired, pneumonia. 3. Pseudomonas aeruginosa is the major pathogen associated with hospital-acquired pneumonia (HAP). 4. Haemophilus influenzae, Mycoplasma pneumoniae, and Streptococcus pneumoniae are examples of common causative organisms for community-acquired, not hospital-acquired, pneumonia.

The nurse is reviewing the anatomy of the heart with a patient scheduled for cardiac surgery. Which patient statement indicates additional teaching is required? 1. "Oxygenated blood returns to the left atrium through the pulmonary vein." 2. "The right atrium receives blood from the superior and inferior vena cava." 3. "Blood leaves the right ventricle and travels through the pulmonary vein to the lungs." 4. "Blood leaves the right ventricle and travels through the pulmonary artery to the lungs."

3 : Oxygenated blood returns to the left atrium through the pulmonary vein. The right atrium receives blood from the superior and inferior vena cava. Blood leaves the right ventricle and travels through the pulmonary artery to the lungs, not the pulmonary vein. Deoxygenated blood is delivered to the pulmonary circuit through the pulmonary artery. Blood leaves the right ventricle and travels through the pulmonary artery to the lungs.

Which clinical manifestation in the patient with chronic obstructive pulmonary disease (COPD) is more indicative of emphysema? 1. Cyanosis 2. Obese 3. Reddish complexion 4. Non-productive cough

3 : Patients with chronic bronchitis are typically obese, have hypoxemia, and appear cyanotic, or as a "blue bloater," and have excessive mucus production with a productive cough. Patients with chronic bronchitis have hypoxemia and appear cyanotic, or as a "blue bloater," and have excessive mucus production with a productive cough. Patients with emphysema sometimes have a reddish complexion, appear to be puffing, and may be referred to as a "pink puffer." The cough is usually productive in both chronic bronchitis and emphysema.

The nurse is assessing several patients at a community clinic. The nurse questions administration of the annual influenza vaccination in which patient? 1. A 3-year-old with cystic fibrosis 2. A 25-year-old pregnant woman at 20 weeks' gestation 3. A 35-year-old man with a severe allergy to eggs 4. A 65-year-old woman with diabetes

3 : Patients with chronic disorders, especially diabetes and cardiac, renal, or pulmonary diseases, are more susceptible to complications from the flu; therefore, this patient should receive an annual influenza vaccine. 2 Pregnant women, particularly during the second and third trimesters, are at increased risk of complications from the flu; therefore, this patient should receive the annual influenza vaccine. A 35-year-old man with a severe allergy to eggs should not get a flu shot, because the annual vaccine contains an inactive ingredient with an egg protein. This patient needs to be assessed for an alternative form of vaccine without this egg protein. 4 People at increased risk of influenza or its complications include infants, young children, and anyone age 50 or older; therefore, this patient should receive an annual influenza vaccine.

The nurse correlates a patient's blood pressure of 174/98 mm Hg most directly to which aspect of cardiac output? 1. Preload 2. Heart rate 3. Afterload 4. Contractility

3 : Preload is the amount of blood in the ventricles at the end of diastole. Extreme overfilling decreases the effectiveness of the contraction, decreasing cardiac output. The heart rate is not directly impacted by afterload. It is primarily impacted by stimulation from the autonomic nervous system. Afterload is the resistance to flow the ventricle must overcome to open the semilunar valves and eject its contents. This is related to blood pressure, vessel lumen diameter, and/or vessel compliance. Hypertension on the right or left is implicated in the negative effects of increased after load. Contractility refers to the force of the mechanical contraction, which can be increased with sympathetic stimulation or calcium release or decreased when hypoxia or acidosis occurs.

The nurse is providing care to a patient admitted after experiencing an acute asthma attack. Which assessment findings indicate the need for an immediate intervention?1. Retractions and fatigue 2. Tachycardia and tachypnea 3. Inaudible breath sounds 4. Diffuse wheezing and the use of accessory muscles when inhaling

3 : Retractions and fatigue are also a progression of symptoms that occur with an asthma attack and represent a more severe episode, but they are not the worst or most serious set of symptoms listed, because air is still moving and exchanging. 2. 1.During an asthma attack, tachycardia, tachypnea, and prolonged expirations are common. They are early symptoms of the disease process and can be addressed without urgency. 3. Inaudible breath sounds, reduced wheezing, and ineffective cough indicate that little or no air movement into and out of the lungs is taking place. Therefore, this set of symptoms represents the most urgent need, which is immediate intervention by the nurse to open up the lungs with drug management to prevent total respiratory failure. 4. Diffuse wheezing and the use of accessory muscles when inhaling indicate a progression of the severity of the symptoms, but airflow is still occurring; therefore, they do not require the most urgent action.

Although all of these patients do not currently smoke cigarettes, the nurse recognizes that which patient is at greatest risk factor for the development of lung cancer? 1. 60-year-old black male with a history of smoking in his teens 2. 64-year-old white female who went to an elementary school that had asbestos 3. 68-year-old black male with a 40 pack-year history of smoking 4. 72-year-old white male with a 10 pack-year history of smoking

3 : Risk factors for lung cancer include age older than 65, male gender, black race, exposure to occupational pollutants/asbestos, and cigarette smoking history. The risk of developing lung cancer increases as the number of cigarettes smoked and the length of time spent smoking increase. This patient has two risk factors—race and smoking history. Risk factors for lung cancer include age older than 65, male gender, black race, exposure to occupational pollutants/asbestos, and cigarette smoking history. The risk of developing lung cancer increases as the number of cigarettes smoked and the length of time spent smoking increase. This patient has one risk factor—exposure to asbestos. 3 Risk factors for lung cancer include age older than 65, male gender, black race, exposure to occupational pollutants/asbestos, and cigarette smoking history. The risk of developing lung cancer increases as the number of cigarettes smoked and the length of time spent smoking increase. This patient has four risk factors—age, race, gender, and significant smoking history. Risk factors for lung cancer include age older than 65, male gender, black race, exposure to occupational pollutants/asbestos, and cigarette smoking history. The risk of developing lung cancer increases as the number of cigarettes smoked and the length of time spent smoking increase. This patient has three risk factors—age, gender, and smoking history (but less significant than the patient with a 40 pack-year history).

The nurse is assessing a patient who was recently diagnosed with chronic rhinitis. Which statement by the patient is most consistent with this diagnosis? 1. "My nasal drainage is always clear and watery." 2. "My nose has been runny for 4 weeks." 3. "My nasal drainage has gotten thicker over the last 12 weeks." 4. "I always get a runny nose in the spring when everything blooms."

3 : Runny and clear nasal secretions are consistent with acute rhinitis. In chronic rhinitis, symptoms last 12 weeks or longer with discolored nasal discharge.In chronic rhinitis, symptoms last 12 weeks or longer with discolored nasal discharge and radiographical evidence of fluid levels within the sinuses. Allergic rhinitis is categorized as perennial, occurring throughout the year or seasonally. Chronic rhinitis last 12 weeks or longer with discolored nasal discharge.

The nurse finds a patient admitted with asthma and pneumonia restless, anxious, complaining of increasing difficulty breathing, and stridor. Because of suspected status asthmaticus, an arterial blood gas is ordered. Which result requires an immediate intervention? 1. pH 7.30 2. PaCO2 48 mm Hg 3. PaO2 50 mm Hg 4. SpO2 88%

3 : Status asthmaticus is an acute exacerbation of asthma that is unresponsive to repeated doses or treatment with the typical rescue medications. Although the acidotic pH is concerning, the severe hypoxemia is the priority at this time. 2. Status asthmaticus is an acute exacerbation of asthma that is unresponsive to repeated doses or treatment with the typical rescue medications. Although the mildly acidotic PaCO2 is concerning, the severe hypoxemia is the priority at this time. 3. Status asthmaticus is an acute exacerbation of asthma that is unresponsive to repeated doses or treatment with the typical rescue medications. Although all of the arterial blood gas (ABG) results are concerning, the severe hypoxemia indicated by the low PaO2 is the priority at this time. 4. Status asthmaticus is an acute exacerbation of asthma that is unresponsive to repeated doses or treatment with the typical rescue medications. The significantly low PaO2 is a higher priority than the SpO2 at this time.

A patient's QRS complex is becoming increasingly wider. What is occurring within the heart muscle that is reflected on this tracing? 1. The ventricles are repolarizing. 2. Atrial repolarization is occurring. 3. Ventricular depolarization is prolonged. 4. The atria depolarize and the impulse at the atrioventricular (AV) node is delayed.

3 : The T wave corresponds to ventricular repolarization.Atrial repolarization occurs during ventricular contraction. That waveform is not visible but is buried in the QRS complex.The QRS complex corresponds to ventricular depolarization. If the complex is widening, then ventricular depolarization is taking longer to complete.The PR interval reflects the time required for atrial depolarization and the delay of the impulse at the atrioventricular (AV) node.

The nurse is caring for a patient with respiratory acidosis. Based on the actions of these chemoreceptors to this acidosis, the nurse assesses for which physiological response? 1. Decreased blood pressure 2. Decreased heart rate 3. Increased respiratory rate 4. Increased temperature

3 : The blood pressure is increased. The chemoreceptors respond to the acidosis by inducing vasoconstriction to increase blood pressure (BP) and increase blood flow to the lungs, facilitating oxygen and CO2 exchange. Chemoreceptors do not impact the heart rate or temperature. Chemoreceptors respond to changes in oxygen and CO2 concentrations. Decreased levels of oxygen with increased levels of CO2 produce acidosis. The chemoreceptors respond to the acidosis by inducing vasoconstriction to increase BP and increase blood flow to the lungs, facilitating oxygen and CO2 exchange. Respiratory rate is also increased. Chemoreceptors do not impact the heart rate or temperature.

The patient recently released from the hospital after recovering from complications related to influenza is being seen in the outpatient clinic. Which patient statement indicates the need for further intervention? 1. "I went back to work." 2. "I am drinking at least 8 glasses of water daily. " 3. "I continue to wake up coughing at night." 4. "I only had a low-grade fever once since I left the hospital."

3 : The patient is allowed to return to work once cleared by the provider and having no clinical manifestations of complications.Adequate fluid intake is important to facilitate decreasing viscosity of secretions for expectoration. A patient who continues to be awakened during the night because of coughing requires further assessment, particularly in relation to the color and viscosity of secretions. Primary viral pneumonia from influenza can cause coughing that lasts up to 2 weeks. Secretions that are white are consistent with viral infection. Purulent nasal discharge/sputum indicates a secondary bacterial infection. This patient statement does not indicate the need for further intervention by the nurse.

The nurse is caring for a patient in a community clinic who wishes to quit smoking. The patient asks the nurse, "If I quit smoking, will my risk of lung cancer be the same as a nonsmoker?" Which is the best response by the nurse? 1. "No one knows for sure what the risk is for someone who quits smoking." 2. "Your risk of lung cancer will be equal to that of a nonsmoker." 3. "Your risk of lung cancer will decline if you quit, but it will remain higher than a nonsmoker's." 4. "Your risk of lung cancer will never drop because the damage has already been done."

3 : The risk for someone who quits is known to be dramatically less than for someone who continues to smoke.Although the patient's risk for lung cancer will diminish sharply on quitting smoking, it will not drop to the level of someone who never smoked. The risk for someone who quits is known to be dramatically less than for someone who continues to smoke.Although damage has been done, the patient's risk will drop dramatically on quitting smoking.

A patient who is admitted for a potential heart attack is prescribed serial blood draws for creatine kinase (CK), creatine kinase myocardial bands (CK-MB), and troponin. After blood is drawn for the baseline, which frequency does the nurse schedule for the follow-up blood draws for the next 12 hours? 1. Every 1 to 2 hours 2. Every 2 to 3 hours 3. Every 3 to 4 hours 4. Every 4 to 5 hours

3 : This is not the appropriate frequency for the nurse to schedule the follow-up blood draws for CK, CK-MB, and troponin levels over a 12-hour period. This is not the appropriate frequency for the nurse to schedule the follow-up blood draws for CK, CK-MB, and troponin levels over a 12-hour period. After the baseline blood draw is completed, the nurse schedules subsequent draws every 3 to 4 hour for a period of 12 hours. This is not the appropriate frequency for the nurse to schedule the follow-up blood draws for CK, CK-MB, and troponin levels over a 12-hour period.

In reviewing arterial blood gas results on a patient admitted with a severe influenza infection, the nurse correlates which value to the patients' respiratory acidosis? 1. pH 7.50 2. PaO2 75 mm Hg 3. PaCO2 50 mm Hg 4. O2 Saturation 88%

3 : This pH is consistent with an alkalosis.The PaO2 is low but does not have a direct effect on the acid-base status. Primary respiratory infections may initially cause a respiratory alkalosis (increased pH, decreased carbon dioxide [CO2]) in response to increased respiratory rate. As the condition progresses, a respiratory acidosis (decreased pH, increased CO2) will develop.The O2 saturation is low but does not have a direct effect on the acid-base status.)

The nurse teaches a patient how to implement pursed-lip breathing. Which patient statement indicates a need for additional instruction? 1. "I will sit up with my shoulders relaxed." 2. "I will be sure to exhale twice as long as I inhale." 3. "I will inhale through my mouth while slowly counting." 4. "I will exhale through my mouth as if I was blowing out candles."

3 : This patient statement indicates a correct understanding for the implementation of pursed-lip breathing.This patient statement indicates a correct understanding for the implementation of pursed-lip breathing. The patient inhales through the nose, not the mouth. This patient statement indicates the need for additional instruction.This patient statement indicates a correct understanding for the implementation of pursed-lip breathing.

The nurse uses which the term to describe abnormal breath sounds? 1. Vesicular 2. Bronchial 3. Adventitious 4. Bronchovesicular

3 : Vesicular sound is relatively soft, low-pitched, gentle, rustling sounds. Bronchial sounds are louder, higher-pitched, and sound like air blowing through a hollow pipe.Adventitious is the term used to describe abnormal breath sounds such as crackles, rhonchi, wheezes, and a pleural friction rub. Bronchovesicular sounds have a medium pitch and intensity and are heard anteriorly over the main stem bronchi on either side of the sternum and posteriorly between the scapulae.

The nurse working in the community health clinic places a purified protein derivative (PPD, or Mantoux) test on an adult's forearm and instructs him to return when for reading of the response? 1. The following day 2. Between 24 and 48 hours after placement of the PPD 3. Between 48 and 72 hours after placement of the PPD 4. Between 72 and 96 hours after placement of the PPD

3 : Within 48 to 72 hours after injection of the purified protein derivative (PPD), the administration site should be observed for any reaction. This is too early. Within 48 to 72 hours after injection of the PPD, the administration site should be observed for any reaction. This is too early. The skin test is a positive purified protein derivative (PPD) screen skin test, also called a Mantoux test. The test is administered by injecting 0.1 mL of PPD intradermally into the tissue of the forearm. Within 48 to 72 hours after injection, the administration site should be observed for any reaction. Within 48 to 72 hours after injection of the PPD, the administration site should be observed for any reaction. This is too late.

The nurse assesses for coarse crackles (coarse rales) in patients admitted with which respiratory disorders? Select all that apply. 1. Asthma 2. Bronchitis 3. Chronic obstructive pulmonary disease (COPD) 4. Pneumonia 5. Pulmonary edema

3,4,5 1 Feedback 1. This is incorrect. Patients with asthma typically develop wheezing that is described as a squeaky musical instrument caused by bronchoconstriction and inflammation. 2 This is incorrect. Patients with bronchitis may have fine rhonchi (fine rales), described as the sound of rubbing hair follicles together caused by inflation of previously deflated lung tissue, or rhonchi, described as snoring sounds caused by obstruction in the airways. 3. This is correct. Coarse crackles (coarse rales), described as a popping/coarse sound caused by fluid or secretions in lower airways, may be observed in patients with COPD, sputum in the airways, pneumonia, pulmonary edema, and congestive heart failure. 4. This is correct. Coarse crackles (coarse rales), described as a popping/coarse sound caused by fluid or secretions in lower airways, may be observed in patients with COPD, sputum in the airways, pneumonia, pulmonary edema, and congestive heart failure. 5 This is correct. Coarse crackles (coarse rales), described as a popping/coarse sound caused by fluid or secretions in lower airways, may be observed in patients with COPD, sputum in the airways, pneumonia, pulmonary edema, and congestive heart failure.

The nurse assesses for rhonchi in patients admitted with which respiratory disorders? Select all that apply. 1. Asthma 2. Chronic obstructive pulmonary disease (COPD) 3. Foreign body in airway 4. Lung cancer 5. Pneumonia

3,4,5 This is incorrect. Patients with asthma typically develop wheezing that is described as a squeaky musical instrument caused by bronchoconstriction and inflammation. This is incorrect. Patients with COPD typically have fine or coarse crackles on auscultation. 3 This is correct. Rhonchi are auscultated in patients with bronchitis, foreign body obstruction, masses or malignancies in the lungs, and pneumonia. 4 This is correct. Rhonchi are auscultated in patients with bronchitis, foreign body obstruction, masses or malignancies in the lungs, and pneumonia. 5 This is correct. Rhonchi are auscultated in patients with bronchitis, foreign body obstruction, masses or malignancies in the lungs, and pneumonia.

Which interventions does the nurse implement when providing care for a patient who is admitted for the treatment of active tuberculosis? Select all that apply. 1. Places the patient on droplet precautions 2. Wears a surgical mask when providing patient care 3. Places the patient in a private, negative airflow room 4. Wears eye protection when collecting sputum samples 5. Places a surgical mask on the patient to transport to radiology

3,4,5 This is incorrect. The patient who is admitted for the treatment of active tuberculosis requires airborne, not droplet, precautions. 2 This is incorrect. The nurse wears an N95 mask when providing care to the patient with active tuberculosis. 3 This is correct. Immediate isolation of the patient with suspected or confirmed TB infection in a private room with negative airflow capabilities is a priority. Negative airflow occurs when air moves into the contaminated area or into the patient's room from bordering areas. 4 This is correct. Eye protection is personal protective equipment that is used for implementing standard precautions when respiratory sprays may occur. This is an appropriate nursing action. 5 This is correct. The patient's movement and transportation to other departments should be limited to essential needs only. Patients who must leave the negative pressure room should also wear a surgical mask.

The nurse conducts an admission assessment for a patient who presents in the emergency department (ED). Which patient statements support the healthcare provider's differential diagnosis of pericarditis? Select all that apply. 1. "I have a productive cough on most days." 2. "I have swelling in my lower legs and feet." 3. "I get short of breath when walking to my mailbox." 4. "I have chest pain that seems to radiate to my shoulder blades." 5. "I have to sleep with several pillows or I have trouble breathing."

3,4,5: 1. This is incorrect. A productive cough does not support the healthcare provider's differential diagnosis of pericarditis. This clinical manifestation is expected for the patient diagnosed with left-sided heart failure. 2. This is incorrect. Dependent edema in the lower extremities does not support the healthcare provider's differential diagnosis of pericarditis. This clinical manifestation is expected for the patient diagnosed with right-sided heart failure. 3. This is correct. Dyspnea, or shortness of breath, is a clinical manifestation that supports the healthcare provider's differential diagnosis of pericarditis. 4. This is correct. The chest pain that radiates to one or both trapezius muscle ridges supports the healthcare provider's differential diagnosis of pericarditis. This is because the phrenic nerve innervates these muscles and also traverses the pericardium. This type of chest pain tends to be relieved by sitting up and leaning forward. 5. This is correct. Orthopnea is shortness of breath that occurs when lying in a flat position. The patient who states the need to sleep with several pillows to prevent shortness of breath supports the healthcare provider's differential diagnosis of pericarditis.

In providing care to a patient who is diagnosed with hypertension, which assessment data are risk factors for this disease process? Select all that apply. 1. Current age 45 years 2. Body mass index 28 kg/m 3. A history of cigarette smoking 4. Glomerular filtration rate 58 mL/min 5. Concurrent diagnosis of diabetes mellitus

3,4,5: 1. This is incorrect. Age (older than 55 for men, 65 for women) is a major risk factor for the development of hypertension. 2. This is incorrect. Obesity (body mass index greater than or equal to 30 kg/m2) is a major risk factor for the development of hypertension. 3. This is correct. Cigarette smoking is a major risk factor for the development of hypertension. 4. This is correct. Microalbuminuria or estimated glomerular filtration rate less than 60 mL/min are major risk factors for the development of hypertension. 5. This is correct. Diabetes mellitus is a major risk factor for the development of hypertension.

Which criteria is used to diagnosis acute bacterial rhinosinusitis (ABRS) in adult patients? Select all that apply. 1. A 1-day history of facial pain 2. Decrease in nasal discharge after 6 days 3. New onset of headache after 5 or 6 days 4. Symptoms that last more than 10 days without clinical improvement 5. Temperature greater than or equal to 102°F (39°C) with purulent nasal discharge for 4 days

3,4,5: 1 This is incorrect. Facial pain with fever that lasts longer than 3 to 4 days would indicate acute bacterial rhinosinusitis (ABRS). 2 This is incorrect. An increase, not decrease, in nasal discharge after 6 days would indicate ABRS. 3 This is correct. A new onset of headache after this length of time with symptoms often indicates ABRS. 4 This is correct. Symptoms that last more than 10 days without clinical improvement often indicates ABRS. 5 This is correct. This data support the diagnosis of ABRS.

The nurse recognizes which hemodynamic parameters as measurements of afterload? Select all that apply. 1. Central venous pressure 2. Left atrial pressure 3. Pulmonary vascular resistance 4. Right atrial pressure 5. Systemic vascular resistance

3,5 This is incorrect. CVP represents preload. This is incorrect. This is a measurement of left sided preload. This is correct. Afterload refers to the resistance to flow the ventricle must overcome to open the semilunar valves and eject its contents, and is measured as systemic vascular resistance and pulmonary vascular resistance. This is incorrect. This is a measurement of right sided preload. Because there is no valve between the vena cava and right atrium, this reflects CVP. This is correct. Afterload refers to the resistance to flow the ventricle must overcome to open the semilunar valves and eject its contents, and is measured as systemic vascular resistance and pulmonary vascular resistance.

The nurse provides discharge instructions to a patient who is neutropenic. Which patient statements indicate the need for additional teaching? Select all that apply. 1. "My plants are being moved outside." 2. "I will avoid eating raspberries and blackberries." 3. "I will use a humidifier to moisten the air at night." 4. "I will wash all raw vegetables before eating them." 5. "My prescribed antibiotic can be stopped once I feel better."

3,5: 1. This is incorrect. This statement indicates a correct understanding of the information presented. There should not be any live plants or cut flowers in the home environment because they breed bacteria and mold. 2. This is incorrect. This statement indicates a correct understanding of the information presented. It is best to avoid fruits such as raspberries and blackberries that have little bumps and ridges and cannot be washed well. 3. This is correct. The patient who is neutropenic is taught to avoid standing water in appliances such as humidifiers because this also breeds mold and bacteria; therefore, this statement indicates the need for additional teaching. 4. This is incorrect. This statement indicates a correct understanding of the information presented. The patient is instructed to wash all raw fruits and vegetables well before eating. 5. This is correct. If patients are started on prophylactic antibiotic, antiviral, and antifungal therapies, stress the importance of taking their medications daily and completing the entire course of antibiotics; therefore, this statement indicates the need for additional teaching.

A patient is scheduled for an exercise stress test. For which cardiac health problems is this patient being tested? Select all that apply. 1. Pericarditis 2. Heart failure 3. Valvular disease 4. Infective endocarditis 5. Coronary artery disease

3,5: 1. This is incorrect. An exercise stress test is not used to diagnose pericarditis. 2. This is incorrect. An exercise stress test is not used to diagnose heart failure. 3. This is correct. Diagnostic tests used to diagnose valvular disease include stress testing to identify functional capacity. 4. This is incorrect. An exercise stress test is not used to diagnose infective endocarditis. 5. This is correct. A patient with suspected coronary artery disease may undergo an exercise stress test. This is done to assess the function of the heart during exercise.

The nurse monitors which laboratory results in the patient with a clotting disorder caused by the intrinsic pathway? 1. Prothrombin time (PT) 2. Reticulocyte count 3. Complete blood cell (CBC) count 4. Partial thromboplastin time (PTT)

4 : 1. A prothrombin time is anticipated for a patient experiencing issues with the extrinsic pathway. 2. Reticulocytes are non-nucleated, immature red blood cells that remain in the peripheral blood for 24 to 48 hours while maturing. The reticulocyte count is useful for evaluating anemia. 3. A complete blood count is anticipated for a patient who is experiencing issues with blood components. 4. A partial thromboplastin time (PTT) is anticipated for a patient experiencing issues with the intrinsic pathway. The purpose of the PTT or activated PTT (aPTT) is to screen for deficiencies of the clotting factors in the intrinsic pathways and to monitor response to thrombin inhibitor therapy (heparin, argatroban).

In reviewing a patient's complete blood count (CBC) results, the nurse notes a "shift to the left." What is the significance of these results? 1. There is an elevated number of mature erythrocytes. 2. There is an elevated number of immature neutrophils (bands). 3. There is an elevated number of mature neutrophils (segs). 4. There is an elevated number of immature thrombocytes.

4 : 1. Activity intolerance is a priority for the patient who has oxygenation issues, anemia for example. 2. Fatigue is more related to having sufficient energy for activities. This patient has decreased white blood cell count. 3. Risk for bleeding is a priority for the patient with decreased platelet count. This patient has decreased white blood cell count. 4. Decreased neutrophil counts significantly increase the risk of infection because of a decreased immune response.

What term does the nurse use to document a decreased platelet count? 1. Anemia 2. Neutropenia 3. Polycythemia 4. Thrombocytopenia

4 : 1. Anemia is the term used to describe a decreased red blood cell (RBC) count. 2. Neutropenia is a term used to describe a decreased white blood cell (WBC) count. 3. Polycythemia is a term used to describe an increased RBC count. 4. Thrombocytopenia is a term used to describe a decreased platelet count.

In providing care for a patient with malignant lymphoma, the nurse correlates which clinical manifestations to the development of superior vena cava syndrome? 1. Decreased peripheral pulses 2. Decreased reflexes 3. Tachycardia 4. Shortness of breath

4 : 1. Edema of the face and one or both arms, shortness of breath, chest pain, or dysphagia develops in patients with superior vena cava (SVC) syndrome. There is no decrease in peripheral pulses. 2. Edema of the face and one or both arms, shortness of breath, chest pain, or dysphagia develops. Reflexes are not affected by SVC syndrome. 3. Edema of the face and one or both arms, shortness of breath, chest pain, or dysphagia develops in SVC syndrome. Tachycardia is not associated with complication of malignant lymphoma. 4. In SVC syndrome, the enlarging lymph node compresses the flexible superior vena cava. Edema of the face and one or both arms, shortness of breath, chest pain, or dysphagia develops because there is engorgement in the upper torso and impaired venous return.

A patient in sickle cell crisis reports taking a recent skiing trip that caused a respiratory infection from the cold weather. Which nursing diagnosis is a priority for this patient? 1. Fluid Volume Excess 2. Impaired Mobility 3. Knowledge Deficit 4. Acute Pain

4 : 1. Fluid Volume Deficit, not excess, is likely to be a priority for a patient who is experiencing a sickle cell crisis. 2. Impaired Mobility may be relevant secondary to pain and fatigue, but the acute pain is the priority. 3. The patient has reportedly been skiing, which would be in an area of high altitude, which is contraindicated for someone with sickle cell. This patient appears to have a knowledge deficit about self-care. This diagnosis, however, does not take priority. 4. Acute pain is a physiological issue and is the priority for the patient who is experiencing a sickle cell crisis.

In planning care for a patient with pericarditis, the nurse understands which pathophysiological process contributes to the development of this disease process? 1. Heart cell dysfunction 2. Plaque buildup in vessels 3. Infection of the innermost layer of the heart 4. Inflammation of the tissue surrounding the heart

4 : 1. Heart cell dysfunction describes heart failure. 2. Plaque buildup in vessels describes atherosclerosis. 3. Infection of the innermost layer of the heart describes infective endocarditis. 4. Inflammation of the tissue surrounding the heart describes pericarditis.

Which is an age-related hematological change the nurse anticipates when reviewing a complete blood count (CBC) for an older adult patient? 1. Increased hematocrit 2. Increased hemoglobin 3. Increased B and T cells 4. Increased sedimentation rate

4 : 1. Hematocrit is not affected by the aging process. 2. Hemoglobin is not affected by the aging process. 3. A decrease in B and T cells is anticipated with the aging process. 4. An increased sedimentation rate is anticipated with the aging process.

The nurse is monitoring a patient with peripheral arterial disease (PAD). Which clinical manifestations does the nurse correlate to stage IV PAD? 1. Pedal pulses are decreased. 2. Muscle pain experienced with exercise. 3. Numbness and tingling of the extremity. 4. Tissue that is blackened on the great toe.

4 : 1. In stage I the patient has no complaints of pain but does have decreased pedal pulses. 2. In stage II the patient has muscle pain, burning, and cramping with exercise that is relieved with rest. 3. In stage III the patient has pain at rest, with numbness and burning in the distal extremities. 4. In stage IV the patient experiences ulcers and blackened tissue on the toes, the forefoot, or the heel of the foot.

A patient experiencing fatigue, pallor, and dyspnea on exertion has a complete blood count completed. The nurse correlates these clinical manifestations to which hematological disorder? 1. Polycythemia 2. Thrombocytopenia 3. Neutropenia 4. Anemia

4 : 1. Polycythemia is an abnormally high red blood cell count. 2. Thrombocytopenia is a decreased platelet count and makes the patient at risk for bleeding. 3. Neutropenia is a low white blood count and makes the patient at risk for infection. 4. Anemia is the most common red blood cell disorder, involving a low count and decreased hemoglobin content. Signs and symptoms of anemia can include pallor of the skin and mucous membranes and dyspnea on exertion.

The nurse monitors for which clinical manifestation in a patient diagnosed with right-sided heart failure (HF)? 1. Fatigue 2. Shortness of breath 3. Crackles with auscultation 4. Edema in the lower extremities

4 : 1. The nurse does not monitor the patient diagnosed with right-sided heart failure (HF) for fatigue because this is a clinical manifestation associated with left-sided HF. In left-sided HF, the weakened contraction results in poor peripheral perfusion and a backflow of blood that results in fluid accumulation in the lungs. This produces classic symptoms, including fatigue. 2. The nurse does not monitor the patient diagnosed with right-sided HF for shortness of breath because this is a clinical manifestation associated with left-sided HF. In left-sided HF, the weakened contraction results in poor peripheral perfusion and a backflow of blood that results in fluid accumulation in the lungs. This produces classic symptoms, including dyspnea (shortness of breath). 3. The nurse does not monitor the patient diagnosed with right-sided HF for shortness of breath because this is a clinical manifestation associated with left-sided HF. In left-sided HF, the weakened contraction results in poor peripheral perfusion and a backflow of blood that results in fluid accumulation in the lungs. This produces classic symptoms, including crackles on auscultation. 4. With right-sided HF, the weakened contraction of the right ventricle results in a backflow of blood into the right atrium and venous circulation and is characterized by jugular vein distention (JVD), generalized dependent edema, hepatomegaly, and ascites. Left-sided HF can eventually cause right-sided HF. If that happens, the symptom classification becomes less clear. In severe exacerbations that patient may present with hypotension, cool extremities, decreased or no urine output, and poor or decreasing mentation.

The nurse is reviewing the complete blood cell (CBC) count for a female patient recently admitted to the medical-surgical unit. The CBC count is as follows: platelet 180,000/mm3; hemoglobin 15 3 33- g/dL; red blood cells 4.4 million cells/mm ; white blood cells 6 10 /mm. What is the nurse's analysis of these results? 1. The patient is anemic. 2. The patient has leukopenia. 3. The patient has thrombocytopenia. 4. The patient's laboratory values are within normal limits.

4 : 1. The patient does not have decreased red blood cell count; therefore, does not have anemia. 2. The patient does not have decreased white blood cell count; therefore, does not have leukopenia. 3. The patient does not have decreased platelet count; therefore, does not have thrombocytopenia. 4. Platelets 150,000 to 450,000/mm , hemoglobin 11.7 to 15.5 g/dL in a female 3adult, red blood cell count 3.61 to 5.11 million cells/mm , and white blood cell count 4.5 to 11.1 103/mm3- indicate normal values in this patient.

The nurse recognizes that a patient who presents in the emergency department with symptoms of a thrombotic stroke should be evaluated to receive which medication? 1. Warfarin 2. Vitamin K 3. Abciximab 4. Streptokinase

4 : 1. Warfarin is an anticoagulant medication that is used to prevent the formation of clots. It is not effective for a patient who presents with symptoms of a thrombotic stroke. 2. Vitamin K does not prevent the formation of clots. This medication is administered for an anticoagulant overdose. 3. Abciximab is a platelet inhibitor. 4. Streptokinase is a thrombolytic, or clot buster. This medication is used within 12 hours of the initiation of symptoms for a thrombotic stroke.

In administering a calcium channel blocker to patient diagnosed with primary hypertension, the nurse correlates a decrease to which mechanism of action of this medication? 1. Blocks formation of angiotensin II, a potent vasoconstrictor 2. Blocks the action, not the formation, of angiotensin II 3. Decreases blood vessels' ability to contract and cause vasoconstriction 4. Relaxes the muscles of the blood vessels

4 : 1. Angiotensin-converting enzyme (ACE) inhibitors, not calcium channel blockers, dilate blood vessels by blocking formation of angiotensin II. 2. Angiotensin II receptor blockers (ARBs), not calcium channel blockers, help relax/dilate blood vessels by blocking the action of angiotensin II. 3. Central agonists, not calcium channel blockers, decrease blood vessels' ability to contract and cause vasoconstriction. 4. Calcium channel blockers help relax/dilate the muscles of the blood vessels.

A patient with heart failure is having a B-type natriuretic peptide (BNP) level drawn. The nurse correlates that the results of this diagnostic provide which data? What is the purpose of this laboratory test? 1. Measure ejection fraction 2. Rule out an ischemic event 3. Differentiate right- from left-sided heart failure 4. Measure "overstretching" of the ventricles

4 : 1. Echocardiograms measure ejection fraction. 2. Cardiac enzymes are used to rule out an acute ischemic event. 3. Ejection fraction is used to differentiate right-sided from left-sided heart failure. 4. Cardiac biomarkers such as B-type natriuretic peptide (BNP) are used to track a patient's response to therapy.

Which assessment finding in patient with Raynaud's phenomena requires an immediate intervention by the nurse? 1. Pitting edema in the lower extremities 2. Leg pain when ambulating 3. Numbness and tingling in the lower extremities 4. Weak distal pulses audible by Doppler only

4 : 1. Pitting edema is not associated with Raynaud's disease. It may be observed in patients with venous obstruction. 2. Numbness and pain on ambulation, along with changes in color of the distal extremities, are clinical manifestations of Raynaud's disease, but weak or absent pulses is the priority. 3. Numbness and pain on ambulation, along with changes in color of the distal extremities are clinical manifestations of Raynaud's disease, but weak or absent pulses is the priority. 4. Weak or absent pulses require an immediate intervention as this indicates arterial insufficiency that might lead to permanent tissue loss.

The nurse correlates which data from a patient's history to a diagnosis of bronchitis? 1. A history of rhinitis 2. A history of eczema 3. A family history of allergies 4. A history of progressive dyspnea

4 : 1. This is expected for the patient with asthma, not bronchitis. 2. This is expected for the patient with asthma, not bronchitis. 3. This is expected for the patient with asthma, not bronchitis. 4. Progressive dyspnea is expected for the patient who is diagnosed with bronchitis.

The nurse correlates which clinical manifestation to the pathophysiological process of infective endocarditis? 1. Friction rub 2. Chest pain at rest 3. Jugular vein distention 4. Painless spots on the palms and soles

4 : The most common clinical manifestation of pericarditis is a friction rub. Chest pain at rest or unstable angina is a manifestation of coronary artery disease.Right-sided heart failure is characterized by jugular vein distention.Clinical manifestations of infective endocarditis include red, painless spots on the palms and soles, called Janeway's lesions.

The nurse is providing education to a patient receiving radiation therapy for the treatment of laryngeal cancer. Which patient statement indicates the need for further education regarding oral care? 1. "I should increase my oral intake of water." 2. "I will avoid spicy foods." 3. "I can chew gum to decrease dry mouth." 4. "I should use a firm-bristle toothbrush to clean my teeth."

4 : A decrease in production of saliva, which may be permanent, can occur with radiation therapy, resulting in dry mouth or xerostomia. Changes in taste sensation, halitosis, increased risk of dental cavities, and oral infections can occur, and increasing fluid intake is an appropriate action. • Oral care: Use a soft-bristle toothbrush and brush and floss after each meal and at bedtime; suck on hard candy or chewing gum; examine the oral cavity for signs of infection, ulcerations, or bleeding. Avoiding spicy and irritating food is indicated as part of oral care for a patient undergoing radiation therapy.Sucking on hard candy or chewing gum can relieve the xerostomia associated with radiation therapy. A soft-bristle brush, as well as flossing after every meal and at bedtime, is indicated to decrease the risk of irritation and inflammation.

The nurse correlates which percussion sound to the patient diagnosed with emphysema? 1. Flat 2. Dull 3. Tympany 4. Hyperresonance

4 : A flat percussion sound is anticipated when assessing the patient who is diagnosed with pleural effusion.A dull percussion sound is anticipated when assessing the patient who is diagnosed with pneumonia, atelectasis, or a lung mass. A tympany percussion sound is anticipated when assessing the patient who is diagnosed with a large pneumothorax.A hyperresonance percussion sound is anticipated when assessing the patient who is diagnosed with emphysema, chronic asthma, or a pneumothorax.

Which percussion sound does the nurse expect when conducting percussion between the ribs during a respiratory assessment? 1. Flat 2. Dull 3. Tympany 4. Resonance

4 : A flat sound is expected when percussing over bone.A dull sound is expected when percussing over the liver, heart, kidney, and diaphragm.A tympany sound is expected when percussing over the stomach.The nurse expects a resonance sound when percussing between the rib during a respiratory assessment.

The nurse is reviewing data collected during the assessment of an older patient. Which finding does the nurse consider as being an age-related change of the cardiovascular system? 1. First heart sound louder 2. Friction rub auscultated 3. Heart rate 64 and regular 4. Blood pressure 168/96 mm Hg

4 : A louder first heart sound may be normal depending on the area being auscultated. A friction rub is associated with pericarditis, which is not an age-related change to the cardiovascular system. A heart rate of 64 and regular is within normal limits. Hypertension is a common cardiovascular health issue related to aging.

Friends of a patient hospitalized with asthma would like to bring the patient a gift. Which gift would the nurse recommend for this patient? 1. A basket of flowers 2. A stuffed animal 3. Fruit and candy 4. A book

4 : A patient with asthma must not be exposed to items that can exacerbate the disease process. Specific allergens, chemicals, and foods must be avoided. Flowers, food, and items that may contain dust, such as a stuffed animal, should be avoided. A patient with asthma must not be exposed to items that can exacerbate the disease process. Specific allergens, chemicals, and foods must be avoided. Flowers, food, and items that may contain dust, such as a stuffed animal, should be avoided. 3. A patient with asthma must not be exposed to items that can exacerbate the disease process. Specific allergens, chemicals, and foods must be avoided. Flowers, food, and items that may contain dust, such as a stuffed animal, should be avoided. Objects void of irritants, such as a book, would be an appropriate gift.

The nurse correlates nasal itching, sneezing, nasal congestion, and rhinorrhea that is thin and clear to which disorder? 1. Acute bacterial rhinosinusitis (ABRS) 2. Acute viral rhinosinusitis 3. Chronic rhinitis 4. Rhinitis

4 : Acute rhinosinusitis is diagnosed by the presence of three symptoms: purulent nasal discharge, nasal obstruction, and facial pain, pressure, or fullness.Acute rhinosinusitis is diagnosed by the presence of three symptoms: purulent nasal discharge, nasal obstruction, and facial pain, pressure, or fullness. In chronic rhinitis, symptoms last 12 weeks or longer with discolored nasal discharge and radiographical evidence of fluid levels within the sinuses. Symptoms of rhinitis include nasal itching, sneezing, nasal congestion, and rhinorrhea (runny nose, which is excessive nasal drainage).

The nurse is caring for an older adult patient who is very thin and emaciated. The patient reports new onset of shortness of breath. A chest x-ray examination reveals a spot on the lungs that the physician believes is an inoperable lung cancer. Because of the patient's poor nutritional status, chemotherapy is not an option. The healthcare provider also believes that the location of the cancer would make radiation therapy unsuccessful. In advocating for this patient, what approach does the nurse collaborate with the healthcare team to offer the patient? 1. Provide palliative care to keep the patient comfortable without diagnostic testing 2. Perform any procedure necessary to diagnose the patient properly 3. Promote the use of blood tests to diagnose the suspected cancer 4. Determine the patient's and family's wishes regarding diagnostic testing

4 : An older adult emaciated patient may have few options for treatment of cancer, if confirmed. The best course of treatment may be palliative care, but it is the choice of the patient and family that should direct the plan of care and choices of diagnostic testing. 2. An older adult emaciated patient may have few options for treatment of cancer, if confirmed. The best course of treatment may be palliative care, but it is the choice of the patient and family that should direct the plan of care and choices of diagnostic testing. 3. An older adult emaciated patient may have few options for treatment of cancer, if confirmed. The best course of treatment may be palliative care, but it is the choice of the patient and family that should direct the plan of care and choices of diagnostic testing. 4. An older adult emaciated patient may have few options for treatment of cancer, if confirmed. The best course of treatment may be palliative care, but it is the choice of the patient and family that should direct the plan of care and choices of diagnostic testing.

Which diagnostic study is the most effective in determining whether a patient has obstructive sleep apnea?1. Arterial blood gas 2. Bronchoscopy 3. Computed tomography (CT) of the chest 4. Polysomnography

4 : Arterial blood gases are used to evaluate oxygenation and acid-base status. They may be evaluated in patients with suspected obstructive sleep apnea (OSA) but are not diagnostically definitive. A bronchoscopy is used to evaluate the bronchioles and is not a part of the diagnostic evaluation for OSA. Chest x-ray examinations (CXR) are used to assess the lung for inflammation, infection, or masses. A CXR may be part of the diagnostic workup for OSA but is not diagnostically definitive. A polysomnography (sleep study) is performed to diagnosis OSA and can be conducted in an overnight sleep laboratory or using home portable monitoring.

The nurse develops the nursing diagnosis "Impaired Gas Exchange related to decreased lung elasticity and hyperinflation of the alveoli secondary to exposure to inhaled pollutants." For which disorder is the nursing diagnosis most relevant? 1. Asthma 2. Chronic bronchitis 3. Cystic fibrosis 4. Emphysema

4 : Asthma is a chronic lung disease characterized by an intermittent, reversible airway obstruction resulting from inflammation of the lung's airways and a tightening of the muscles that surround the airways. The condition affects the bronchial airways, not the alveoli. 2. Chronic bronchitis affects the small airways and is defined as the presence of cough and sputum production for at least 3 months in each of 2 consecutive years. The chronic airflow limitation is caused by a mixture of small airways disease and destruction of the lung tissue. 3. Cystic fibrosis is a genetic disease of the exocrine glands (glands that secrete hormones into ducts also called duct glands) such as the sweat, salivary, or pancreas glands. It is a multisystem disease that produces increased amounts of thick mucus in the respiratory, gastrointestinal (GI), and reproductive systems. 4. Emphysema causes loss of lung elasticity leading to hyperinflation of the alveoli. The small airways collapse prematurely causing trapping of air in the alveoli and subsequent distention. Carbon dioxide cannot leave the alveoli, and oxygen cannot enter, resulting in an ineffective exchange of oxygen and carbon dioxide.

A patient is scheduled for a transesophageal echocardiogram (TEE). Results from this diagnostic evaluation provides which specific information related to the cardiac system? 1. Cardiac filling pressures 2. Integrity of cardiac arteries 3. Heart function during stress 4. Presence of clots in the atria

4 : Cardiac catheterization is an invasive x-ray procedure during which a radiopaque catheter is advanced through an artery or vein to the heart under fluoroscopy to evaluate cardiac filling pressures. Coronary angiography is the primary reason cardiac catheterization is performed. It is a left-sided cardiac catheterization with the purpose of inspecting the coronary arteries for blockage. A cardiac stress test is done to evaluate heart functioning during times of increased workload.Information about the presence of clots in the atrium, a risk factor for stroke, is more easily viewed through TEE.

The nurse questions which order for the patient admitted with the flu who is experiencing tachypnea? 1. Maintain adequate hydration. 2. Keep the head of the bed elevated. 3. Teach the patient coughing, deep breathing, and hydration. 4. Prepare the patient for intubation.

4 : Coughing, deep breathing, and hydration are essential for achieving airway clearance.Keeping the head of the bed elevated improves lung excursion and reduces the work of breathing. Coughing, deep breathing, and hydration are essential for achieving airway clearance. This patient is experiencing tachypnea and there are no data (difficulty maintaining an airway, decreased oxygen saturation, etc.) that support that the patient requires intubation at this time.

A patient is being evaluated for a possible myocardial infarction, but the patient is not sure when the pain started because he has a history of gastroesophageal reflux. He has had intermittent chest pain, with some episodes of dizziness and fatigue, over the last week. Which diagnostic result will be most helpful in determining whether the patient has suffered cardiac injury? 1. Elevated creatine kinase (CK) 2. Elevated creatine kinase myocardial bands (CK-MB) 3. Elevated myoglobin 4. Elevated troponin

4 : Creatine kinase (CK) is a general marker of cellular injury. It is released from cells in the brain, skeletal muscle, and cardiac tissue after muscle damage has occurred. Creatine kinase myocardial bands (CK-MB) is the marker specific to cardiac tissue. When myocardial damage occurs, CK-MB is released from the cells. Increased levels can be seen at 3 hours and can remain elevated for up to 36 hours before returning to normal. Myoglobin is released and elevated in muscle damage but is not specific for cardiac tissue. It can be used in conjunction with the other values to help rule out or rule in a myocardial infarction. Troponin is a specific marker of cardiac muscle damage and is the preferred method for diagnosing cardiac injury. It is a protein released from damaged tissue and can elevate within 4 hours of injury. Because it can stay elevated up to 10 days (longer than CK-MB), it is a valuable marker when attempting to diagnose injury in the recent past.

The nurse notes that a patient has bilateral lower extremity edema. For which health problem should the nurse assess further? 1. Pericarditis 2. Cardiac tamponade 3. Lymph obstruction 4. Venous insufficiency

4 : Distended jugular veins are associated with pericarditis.Distended jugular veins are associated with cardiac tamponade.Unilateral extremity edema can indicate a lymphatic obstruction.Bilateral lower extremity edema generally indicates venous insufficiency or heart failure.

The nurse is providing care to a patient receiving radiation in the treatment of laryngeal cancer. Which patient statement indicates the need for further education regarding radiation treatments? 1. "My skin may become red, tender, and peel." 2. "I should avoid the sun while I am receiving this therapy." 3. "I will wear soft, loose-fitting clothing made of cotton to limit irritation." 4. "My therapy includes washing my skin with a deodorant soap and applying lotion."

4 : During radiation therapy, skin can become red and tender and may peel.Skin care for radiation therapy includes avoiding heating pads, ice packs, lotions or powders, sun exposure, or extremes of temperature.Skin care for radiation therapy includes wearing loose-fitting, soft clothing. A mild, not harsh (like deodorant), soap should be used to cleanse the site receiving radiation. Lotion should only be applied if prescribed by the radiologist.

The nurse monitors for which gastrointestinal clinical manifestations in the patient diagnosed with cystic fibrosis? 1. Diarrhea 2. Hematemesis 3. Melena 4. Steatorrhea

4 : Gastrointestinal clinical manifestations of cystic fibrosis include constipation, not diarrhea.Gastrointestinal clinical manifestations of cystic fibrosis include greasy, fatty stools, not hematemesis (blood in vomitus). Gastrointestinal clinical manifestations of cystic fibrosis include greasy, fatty stools, not melena (blood in the stool).Gastrointestinal clinical manifestations of cystic fibrosis include steatorrhea (frequent greasy, bulky, fatty stools) or difficult bowel movements.

The nurse is preparing to determine a patient's cardiac output. Which measurement should be used for preload? 1. Heart rate 2. Blood pressure 3. Mean arterial pressure 4. Central venous pressure

4 : Heart rate (HR) is the number of cardiac contractions per minute; preload is the amount of blood in the ventricles at the end of diastole. Blood pressure provides information on the force of the heart's contraction and volume of cardiac output as well as the resistance offered by the arterial vascular system. Blood pressure is impacted by afterload. Mean arterial pressure is the average pressure maintained throughout the cardiac cycle; preload is the amount of blood in the ventricles at the end of diastole. Preload, the amount of blood in the ventricles at the end of diastole or the amount of stretch of the muscle tissue at the end of filling, is reflected by measurements obtained through a centrally located intravenous line. For preload, the central venous pressure is used.

The nurse monitors for which therapeutic effect when administering leukotriene receptor antagonists to patients diagnosed with asthma? 1. Cause bronchodilation of the airways 2. Decrease viscosity of secretions 3. Decrease work of breathing by relaxing muscles around bronchi 4. Inhibit the inflammatory process

4 : Leukotriene receptor antagonists interfere with the inflammatory response, not bronchodilation (bronchodilators have this action).Leukotriene receptor antagonists interfere with the inflammatory response. Mucolytics decrease the viscosity of secretions. Anticholinergics relax the muscles around the larger airways or bronchi to ease the work of breathing. Leukotriene receptor antagonists are another class of drugs that may be used to enhance asthma control if the usual medications are not effective. They are not steroids; they inhibit the leukotriene-mediated inflammatory process.

In providing care to a patient admitted with an acute asthma exacerbation, the nurse prepares which "rescue" medication for administration first? 1. Inhaled anti-inflammatories 2. Mucolytics 3. Long-acting beta2-adrenergic agonists 4. Short-acting beta2-adrenergic agonists

4 : Long-term control medications are used daily regardless of the symptoms a patient is experiencing to achieve and maintain control of the asthma. The most effective long-term control medications for asthma are inhaled anti-inflammatories. Mucolytics decrease viscosity of secretions but are not considered "rescue" medications. Anticholinergics are another group of bronchodilators. Different from beta2-adrenergic agonists, which relax the bronchioles or small airways, anticholinergics relax the muscles around the larger airways or bronchi. 4. "Rescue" drugs are those medications used once an asthma attack has started; these are usually short-acting bronchodilators. Short-acting beta2-adrenergic agonists are the gold standard because they are most effective. Short-acting beta2-adrenergic agonists should be used for acute exacerbations of asthma only.

The nurse is providing care to a patient who is diagnosed with rhinitis and prescribed a first generation antihistamine. The nurse includes data regarding which medication? 1. Loratadine 2. Fluticasone 3. Guaifenesin 4. Diphenhydramine

4 : Loratadine is a second generation antihistamine. Fluticasone is a corticosteroid nasal spray. Guaifenesin is a decongestant.Diphenhydramine is a first generation antihistamine.

How does the nurse interpret these arterial blood gas results? pH 7.48PaCO2 30 mm HgHCO3 24 mEq/L 1. Metabolic acidosis 2. Respiratory acidosis 3. Metabolic alkalosis 4. Respiratory alkalosis

4 : Metabolic acid-base disorders are based on changes in HCO3- (elevated in alkalosis and decreased in acidosis).PaCO2 greater than 45 mm Hg and pH less than 7.35 correlate with respiratory acidosis. Metabolic acid-base disorders are based on changes in HCO3- (elevated in alkalosis and decreased in acidosis).PaCO2 less than 35 mm Hg and pH greater than 7.45 correlate with respiratory alkalosis.

While auscultating a patient's chest, the nurse notes wheezing, and correlates this finding with which disorder? 1. Bronchitis 2. Pleural effusion 3. Pulmonary edema 4. Chronic obstructive pulmonary disease

4 : Rhonchi, described as snoring sounds, are observed in patients with bronchitis. Diminished breath sounds are observed in patients with pleural effusion. Coarse crackles are observed in patients with pulmonary edema. Wheezes are continuous high-pitched squeaking or rapid sounds caused by the rapid vibration of the bronchial walls, which is caused by a blockage in airways that often occurs with chronic obstructive pulmonary disease.

Which structure of the respiratory system is responsible for filtering, warming, and humidifying inhaled air? 1. Alveoli 2. Pharynx 3. Sinuses 4. Turbinates

4 : The alveoli are in the terminal bronchioles and are the location for gas exchange. The oropharynx is the mucous membrane located directly behind the mouth. Within the oropharynx are the palatine tonsils, which are responsible for filtering out foreign objects or microorganisms that passed the turbinates. 3 The sinuses are empty air-filled cavities that humidify and warm inspired air; absorb shock, providing protection from facial trauma; provide voice resonance; and decrease the weight of the skull. 4 Turbinates filter the air, and any foreign matter is filtered out through the cilia. In addition to filtering the air entering the nostril, turbinates are responsible for humidifying and warming the air.

The nursing diagnosis "Ineffective Breathing Pattern related to decreased chest wall compliance" is most relevant to the older adult patient with which condition? 1. Decreased diaphragmatic strength 2. Delays in gas exchange 3. Depressed cough reflex 4. Kyphosis

4 : The decrease in function and strength of the intercostals and diaphragm increases inspiratory effort to maintain adequate ventilation.Delays in gas exchange and depressed cough reflex increased the potential for episodes of hypercapnia and hypoxia. Delays in gas exchange and depressed cough reflex increased the potential for episodes of hypercapnia and hypoxia.Kyphosis and osteoporosis of the thoracic vertebrae cause a decrease in chest wall compliance.

A patient is recovering from a cardiac catheterization that was performed via the right groin. Which assessment requires the nurse to notify the healthcare provider immediately? 1. Pain at the insertion site 2. Urine output 250 mL/2 hours 3. Oozing serosanguinous drainage at the puncture site 4. Decreased pulses in the right foot

4 : The limb of the puncture site should be monitored. The patient may complain of pain at the insertion site, but this is not unexpected and not as much of a priority as the decreased pulses. Urine output should be monitored post procedure due to the dye used during the procedure. A urine output of 250 mL/2 hours indicates adequate renal perfusion postprocedure. There may be some oozing of serosanguinous drainage at the site because of the large catheter used. This requires monitoring by the nurse, but is not as significant as the decreased peripheral pulses on the affected side. Decreased pulses distal to the insertion site for the procedure may indicate occlusion of the femoral artery by a hematoma and should be immediately reported to the healthcare provider.

In providing health education to a group of middle school students, how does the nurse describe the function of the epiglottis? 1. Aids in the sensation of smell 2. Conducts gases to the alveoli 3. Filters small particles before air enters the lungs 4. Prevents the entry of solids and liquids into the lungs

4 : The olfactory nerve endings in the roof of the nose are responsible for the sense of smell.The bronchi and the trachea act as a pathway to conduct gases to the alveoli. The nose functions to protect the lower airway by warming and humidifying air and filtering small particles before the air enters the lungs. The epiglottis is a small flap located behind the tongue that closes over the larynx during swallowing. The function of the epiglottis is to prevent solids and liquids from entering the lungs.

In providing care to a patient with laryngeal trauma, which order does the nurse question? 1. Place an intubation tray at the bedside 2. Maintain NPO status 3. Elevate the head of the 45 degrees or higher 4. Administer warm, humidified air

4 : The patient may have a sudden loss of the airway requiring emergency tracheostomy.Elevation of the head of the bed will assist in decreasing edema and maintaining a patent airway. Maintain NPO status to protect the airway in the presence of edema and/or bleeding. Also, maintain NPO status until the need for surgery has been evaluated. Cool humidified air will help decrease airway edema. Warm air may exacerbate airway edema.

The nurse is providing care to a patient who will need a bronchoscopy. Which patient statement indicates that pre-procedure teaching was effective? 1. "I will be awake and fully conscious during the procedure." 2. "I will require mechanical ventilation after the procedure." 3. "I will need to have my prothrombin time drawn after the test." 4. "I will abstain from eating or drinking for 8 hours before the procedure."

4 : The patient will be sedated during the procedure. Moderate sedation involves giving the patient an intravenous pain medication and sedative. The patient may be able to respond to verbal stimuli but is not fully awake. The patient will not require mechanical ventilation after this procedure.The patient will need to have the prothrombin time evaluated before the procedure, not after the procedure.A bronchoscopy is the insertion of a tube in the airways to view airway structure and obtain tissue sample for biopsy or culture. The patient will need to be NPO for 8 hours before the procedure to decrease the risk for aspiration.

The nurse is caring for a patient hospitalized for a severe bacterial pneumonia with a left lung infiltration. In what position does the nurse place the patient to maximize perfusion to functional alveolar units? 1. Prone 2. Supine with head of bed elevated 30 degrees 3. On the left side 4. On the right side

4 : The prone position is not best to promote alveolar perfusion. Elevating the head of the bed facilitates breathing effort and minimizes risk of aspiration but does not promote alveolar perfusion. For infiltrates of only one lung, when turning, preferentially position the patient with the good lung down to maximize perfusion to functional alveolar units. Because this patient has a left lung infiltrate, the right side is preferred. For infiltrates of only one lung, when turning, preferentially position the patient with the good lung down to maximize perfusion to functional alveolar units. Because this patient has a left lung infiltrate, the right side is preferred.

Before an arterial blood gas is collected from the radial artery, an Allen's test is preformed to access the patency of which artery? 1. Brachial 2. Medial 3. Radial 4. Ulnar

4 : The ulnar, not the brachial, artery is assessed for collateral circulation when an Allen's test is performed before a radial artery stick.The ulnar, not the radial, artery is assessed for collateral circulation when an Allen's test is performed before a radial artery stick. The ulnar, not the radial, artery is assessed for collateral circulation when an Allen's test is performed before a radial stick.If the nurse elects to use the radial artery, assessment of the ulnar circulation needs to be evaluated using Allen's test to ensure adequate collateral circulation.

The nurse correlates which blood pressure readings with stage 2 hypertension? 1. The patient with average blood pressure readings of 128/70 on three separate occasions 2. The patient with average blood pressure readings of 128/90 on three separate occasions 3. The patient with average blood pressure readings of 138/88 on three separate occasions 4. The patient with average blood pressure readings of 142/92 on three separate occasions

4 : This is classified as elevated blood pressure because the systolic blood pressure (SBP) is greater than 120 mm Hg and the diastolic blood pressure (DBP) is less than 80 mm Hg. This is classified as stage 1 hypertension because the DBP is between 80 and 89 mm Hg.This BP is classified as stage 1 hypertension because the SBP is between 130 and 139 mm Hg. This BP is classified as stage 2 hypertension because the SBP is greater than 140 mm Hg and the DBP is greater than 90 mm Hg.

The nurse provides care to patients in the urgent care center. Which patient has the greatest risk for developing influenza? 1. A patient aged 26 years with a history of systemic lupus erythematosus (SLE) who works as an accountant 2. A patient aged 44 years who is a nurse 3. A patient aged 60 years with a history of asthma 4. A patient aged 66 years with a history of diabetes mellitus

4 : This patient has one risk factor for developing influenza (a history of asthma). There is another patient with a greater risk. This patient has one risk factor for developing influenza (healthcare provider occupation). There is another patient with a greater risk. This patient has one risk factor for influenza (chronic health condition). There is another patient with a greater risk.This patient has two risk factors for influenza (age and diabetes); therefore, this patient is at the greatest risk for developing influenza.

Which statement by the patient being treated with intranasal corticosteroids indicates that teaching was effective regarding this medication? 1. "I will have to monitor my glucose levels because of this medication." 2. "I may gain a few pounds because of this medication." 3. "I will have to gradually wean myself from this medication." 4. "I may not see result from this medication for a few days or weeks."

4 : This statement indicates that further teaching is needed because intranasal corticosteroids have no systemic effects, so they do not impact blood glucose levels. This statement indicates that further teaching is needed because intranasal corticosteroids have no systemic effects, so weight gain is not expected. This statement indicates that further teaching is needed because intranasal corticosteroids have no systemic effects, so tapering this medication is not needed. Intranasal corticosteroids are the most effective treatment of allergic rhinitis and related symptoms when used on a regular basis. There is no systemic effect from use of intranasal corticosteroid medication. Intranasal corticosteroids may take days or weeks to have full symptom relief.

Which patient statement regarding the use of a normal saline nasal rinse requires intervention by the nurse?1. "I will lean over the sink with my head tilted to one side." 2. "I will insert the spout of the irrigating device in the upper nostril." 3. "I will slowly pour the normal saline into the nostril while breathing through my mouth." 4. "I will allow fluid to drain out the upper nostril to remove excess mucous from my nose."

4 : This statement indicates understanding of the normal saline irrigation. In this procedure the patient pours or sprays normal saline into one nostril; it flows through the nasal cavity and pours out the other nostril. It is beneficial in removal of infectious debris, bacteria, allergens, and inflammatory mediators. It also decreases edema of the nasal mucosa and the viscosity of nasal secretions. This statement indicates understanding of the normal saline irrigation. In this procedure the patient pours or sprays normal saline into one nostril; it flows through the nasal cavity and pours out the other nostril. 3 This statement indicates understanding of the normal saline irrigation. In this procedure the patient pours or sprays normal saline into one nostril; it flows through the nasal cavity and pours out the other nostril. This patient statement requires intervention by the nurse. The fluid drains from the lower, not upper, nostril.

A patient with atherosclerosis asks why smoking cessation is important. What should the nurse respond to this patient? 1. "Tobacco toxins increase your HDL-C." 2. "Tobacco reduces the effects of cholesterol in the body." 3. "Tobacco causes the blood pressure to drop and changes the cells within the arteries." 4. "Tobacco smoke speeds the growth of atherosclerosis in coronary arteries, aorta, and the legs."

4 : Toxins in tobacco smoke lower a person's high-density lipoprotein cholesterol (HDL-C) while raising levels of low-density lipoprotein cholesterol (LDL-C). Tobacco increases circulating cholesterol.Cigarette smoke causes hypertension and changes endothelial cells. Tobacco smoke greatly worsens atherosclerosis and speeds its growth in the coronary arteries, aorta, and arteries in the legs.

The nurse is planning care for a patient diagnosed with influenza. Which intervention by the nurse is the priority when planning this patient's care? 1. Restricting all visits from family and friends 2. Providing staff with N95 mask respirators 3. Placing the patient in a negative air flow room 4. Placing the patient on droplet precaution

4 : Visitation may be limited, especially during the first few days, but visitation is not totally restricted. Visitors should wear a mask while in the room.N95 mask respirators are indicated for patients on airborne precautions. Patients with influenza ae placed on droplet and contact precautions. Negative air flow rooms are for diseases such as chicken pox, measles, and severe acute respiratory syndrome (SARS). To prevent the spread of influenza, the patient is placed in a private room with signs for droplet and contact precautions. It is appropriate for the healthcare workers to use appropriate personal protective equipment (PPE) for these transmission-based precautions. Place patient in droplet precautions to avoid viral transmission. Personal protective equipment required includes mask, gown, gloves, and eye protection if there is a risk of splash of body fluids.

A patient is prescribed a 12-lead electrocardiogram. In which order should the nurse apply the V leads? (Enter the number of each step in the proper sequence) 1. Midaxillary line 2. Between V2 and V43. Midclavicular line fifth intercostal space 4. Fourth intercostal space left of the sternum 5. Fourth intercostal space right of the sternum 6. Between V4 and V6 anterior axillary line

5,4,2,3,6,1 The chest positions for the V leads are: V1: fourth intercostal space, just to the right of the sternum; V2: fourth intercostal space, just to the left of the sternum; V3: Between V4 and V2; V4: on the midclavicular line and fifth intercostal space; V5: between V6 and V4 on the anterior axillary line; and V6: on the midaxillary line, horizontal with V4.

Which of the following indicates an abnormal assessment finding of the respiratory system? a. Symmetrical chest expansion b. Muscle retractions with inhalation c. The sound of resonance with percussion of lung tissue d. Pink mucosa of the mouth and nares

B

The nurse is teaching the patient recently diagnosed with asthma the correct use of the peak flow meter. In which order does the nurse instruct the patient? (Enter the letter of each step in the proper sequence) A Place the mouthpiece in my mouth. b. Ensure the slide marker is at zero (or lowest marking on the scale). C. Stand up straight. D. Take a deep breath. E. Note/record the marking on the scale. F. Blow a "fast hard blast."

B,C,D,A,F,E

Which of the following is included in an arterial blood gas analysis? a. End-tidal CO2 b. Hemoglobin c. PaCO2 d. Sodium

C. Explanation: PaCO2 is the partial pressure of carbon dioxide and is included in all arterial blood gas analysis. The other laboratory values are not included in an arterial blood gas.

The nurse has just received a report on assigned patients. Prioritize which patient should be seen first, second, then third. Patient A: Mr. Albert, a 56-year-old white male with a history of diabetes mellitus and hypertension. He was admitted for chest pain. His troponin levels have been negative. He is scheduled for a CT scan to rule out aortic dissection. He is now complaining of back pain, and his blood pressure is noted to be 90/50 mmHg which is down from 120/78 from 30 minutes ago. Patient B: Mrs. Casey is a 72-year-old female admitted with hypertensive urgency. She has been started on several antihypertensive medications and Lasix. Patient C: Mr. Stanley is a 63-year-old male postoperative day one after a carotid endarterectomy. His pressure is now 118/70. He is complaining of a headache.

Explanation: A,C,B As the nurse taking care of these patients, you should assess patient A, Mr. Albert. Back or flank pain is a classic symptom of aortic dissection. He is also noted to have a considerable drop in his blood pressure. This drop could be secondary to internal bleeding from a dissection. Patients with aortic dissection require immediate surgical intervention. Mr. Stanley (patient C) would be the second patient to go assess. The headache could be a sign of reocclusion—you will want to complete a neuro exam, which is indicated for patients post carotid endarterectomy. Mrs. Casey (patient B) is the last patient you would assess. Mrs. Casey is a stable patient. She will need a blood pressure check and will be on intake and output for Q4 hours.

What does a beta blocker do?

Inhibits the sympathetic nervous system response to physical activity, which decreases cardiac workload and oxygen consumption Example: Metoprolol

What does a calcium channel blocker do? Why would we give it to a pt?

Lowers BP and increases blood flow through the coronaries; used when pts cannot tolerate beta blockers or in pts with HF Example: Nifedipine, Amlodipine

Why is nasal breathing more effective than mouth breathing is?

More effective in cleaning, filtering and warming your air then the mouth does

A patient teaching plan should focus on which risk factors for atherosclerosis? (select all that apply) a. Obesity b. Hyperlipidemia c. Smoking d. Age e. Race

a,b,c

What medications would you suspect a patient who is diagnosed with coronary artery disease to be on? (select all that apply) a. Atorvastatin (Lipitor) b. Heparin c. Clopidogrel (Plavix) d. Diphenhydramine

a,b,c

On the basis of patient history and the results of diagnostic tests, what are the priority assessments for Mr. More? (Select all that apply.) a. Strict intake and output b. Vital signs c. Finger stick blood glucose d. Cranial nerve assessment e. Swallowing evaluation

a,b,c Explanation: Frequent blood pressure assessments are necessary due to this hypertension. His creatinine and BUN were elevated initially indicating renal failure, so intake and output is a priority. His blood glucose was elevated on admission requiring continued monitoring. There are no acute neurological changes evident that would make cranial nerve assessment or swallowing evaluation a priority.

A patient teaching plan should focus on which risk factors for atherosclerosis? (Select all that apply.) a. Obesity b. Hyperlipidemia c. Smoking d. Age e. Race

a,b,c Explanation: The patient teaching plan should focus on risk factors that the individual can have an impact on changing. Patients have the opportunity to impact their weight, lipid levels, and smoking. Patients are not able to modify their age or race.

What medications would you suspect a patient who is diagnosed with coronary artery disease to be on? (select all that apply) a. Metoprolol b. Lisinopril c. Nitroglycerin d. Aspirin e. Simvastatin (Zocor)

a,b,c,d,e

Monitoring of which of the following biophysiological parameters occurs during a sleep study? (select all that apply) a. Cardiac rhythm b. Electroencephalogram c. Eye movement d. Blood pressure e. Oxygen saturation

a,b,c,e Explanation: Blood pressure is not monitored during a sleep study. The remaining items are closely monitored to determine depth of sleep and presence of cardiac dysrhythmias during sleep

What is the most important information for the nurse to include in the teaching plan about management of home oxygen therapy? (select all that apply) a. Do not change the setting of the oxygen without the health-care provider's permission. b. Place "No smoking" signs within the home c. Oxygen is a fire hazard d. Others may smoke around you as long as you do not smoke e. Keep a fire extinguisher near the oxygen source

a,b,c,e Explanation: Changing oxygen levels without the permission of the healthcare provider could lead to oxygen toxicity. No smoking signs will indicate that oxygen is in the home. Oxygen is a fire hazard. A fire extinguisher is required due to the severity of complications if there is a fire related to the oxygen delivery system.

Potential complications of rhinosinusitis include which of the following? (select all that apply) a.Orbital cellulitis b. Osteomyelitis c. Meningitis d. Pharyngitis e. Orbital abscess

a,b,c,e Explanation: Infectious processes close to the sinuses can develop. The larynx is remote from the sinuses.

In management of heart failure what might the pts medications include? (select all that apply) a. Angiotensin-converting enzyme inhibitors (ACE inhibitors) (captopril, enalapril) b. Diuretics: loop diuretics, aldosterone receptor antagonists (furosemide, spironolactone) c. Beta blockers (carvedilol) d. Bupropion (Wellbutrin) e. Hydrocodine f. Cardiac glycosides (digoxin/Lanoxin)

a,b,c,f

he nursing plan of care for a patient diagnosed with lung cancer includes which of the following? (select all that apply) a. Education about disease management b. Frequent assessment of pain levels c. Reminders that recovery is coming soon d. Frequent assessment of oxygenation e. Weaning from oxygen because the disease is terminal

a,b,d Explanation: Education and assessment of pain and oxygenation are key for disease management. Providing oxygen is necessary to maintain and SPO2 above 90%. False reassurance is not appropriate.

Which statements by a patient with a AAA indicate that teaching has been effective? (select all that apply) a. "I need to quit smoking." b. "I need to go to the ED immediately if I have new severe abdominal pain." c. "The doctor may put me on blood thinners." d. "I need to stay on my blood pressure medication." e. "I should keep my legs elevated whenever possible."

a,b,d,

Which statements by a patient with an AAA indicate that teaching has been effective? (Select all that apply.) a. "I need to quit smoking." b. "I need to go to the emergency department immediately if I have new severe abdominal pain." c. "The doctor may put me on blood thinners." d. "I need to stay on my blood pressure medication." e. "I should keep my legs elevated whenever possible.

a,b,d, Explanation: Smoking is the most modifiable risk factor. Abdominal pain may indicate an emergent need for resection or progression towards aortic dissection. Antihypertensives reduce blood pressure and the pressure placed on the aneurysm, therefore slowing the progression. Crossing or elevating the legs increases the pressure in the intrathoracic and abdominal area.

The nurse includes which information in the teaching plan about the management of warfarin? (Select all that apply.) a. Checking urine for blood b. Using an electric razor when shaving c. Using a hard toothbrush for effective plaque removal d. Avoiding any activity or sport that may cause traumatic injury e. Avoiding kale, spinach, collard greens, broccoli, okra, cabbage

a,b,d,e, Explanation: A: Bleeding risk with Coumadin should check for blood in urine to assess for internal bleeding; B, D: use electric razor, and avoid dangerous activities to reduce risk of bleeding; E: foods containing vitamin K increase platelet aggregation and has the opposite effect of warfarin, decreasing the medications effectiveness and increasing risk for clot formation. A hard toothbrush may cause bleeding.

The nurse is providing pt education regarding the use of the CPAP machine to the pt and his wife. Which of the following statements regarding CPAP are correct? (select all that apply) a. The machine utilizes a small compressor b. You will be provided with extra oxygen while you sleep c. The face mask should snugly fit over the nose and mouth d. There is an adjustment period to using the CPAP machine e. You should use it only on nights you need to get up early in the morning.

a,c,d Explanation: CPAP for home use provides only pressure support to maintain a patient airways only, not oxygen. It should be used every night.

A nurse is assessing a patient in the ED with the complaint of sudden onset of severe back pain, tachycardia, and hypotension. Which interventions should the nurse anticipate? (select all that apply) a. Electrocardiogram b. Aortic arteriography c. Ultrasonography d. Chest x-ray e. Computed tomography scan

a,c,e

The nurse is screening pts for their risk of developing hypertension. The nurse should consider which pts at greatest risk? (select all that apply) a. A 40 y/o lation male who is obese and smokes two packs of cigarettes per day b. A 35 y/o Asian female who has a familial history of diabetes mellitus type 1 c. A 78 y/o African American male with chronic renal insufficiency d. A 25 y/o African American female track athlete with a healthy body mass index (BMI) who takes oral contraceptives e. A 60 y/o Caucasian male with vitamin D deficiency and a history of cocaine use

a,c,e

What symptom is an indicator of a resolving TB infection? (select all that apply) a. Decreased sputum protection b. Productive cough c. Stable body weight d. Fevers only at night e. Cessation of night sweats

a,c,e Explanation: Continued fevers and a productive cough indicate active disease

A nurse is providing care for a patient newly diagnosed with heart disease. Which dietary, activity, or lifestyle modification(s) should be included in the plan of care? (select all that apply) a. Stop smoking b. Drink lots of water c. Limit sedentary lifestyle d. Eat a diet rich red meat protein e. Limit alcohol

a,c,e Explanation: Smoking, exercise, and alcohol intake are modifiable risk factors for heart disease. Patients should not be told to drink lots of water as the heart may not be able to handle excessive fluids. A diet rich in red meat is a risk factor

A nurse is assessing a patient in the emergency department with the complaint of sudden onset of severe back pain, tachycardia, and hypotension. Which interventions should the nurse anticipate? (Select all that apply.) a. Electrocardiogram b. Aortic arteriography c. Ultrasonography d. Chest x-ray e. Computed tomography scan

a,c,e Explanation: Ultrasonography can be done quickly and efficiently at the bedside to rule out aortic dissection. CT scan may also be done to definitively diagnose or rule out aortic dissection. An ECG is necessary to rule out MI. Aortic arteriography is a highly invasive and lengthy procedure at a time when quick and uncomplicated is essential. CXR cannot provide a definitive diagnosis of aortic dissection.

A nurse is providing care for a patient newly diagnosed with heart disease. Which dietary, activity, or lifestyle modification(s) should be included in the plan of care? (select all that apply) a. Stopping smoking b. Drinking lots of water c. Limiting sedentary lifestyle d. Eating a diet rich in red meat and protein e. Limiting alcohol intake

a,c,e,

The nurse is screening patients for their risk of developing hypertension. The nurse should consider which patients at greatest risk? (Select all that apply.) a. A 40-year-old Latino male who is obese and smokes two packs of cigarettes per day b. A 35-year-old Asian female who has a familial history of diabetes mellitus type 1 c. A 78-year-old African American male with chronic renal insufficiency d. A 25-year-old African American female track athlete with a healthy body mass index (BMI) who takes oral contraceptives e. A 60-year-old Caucasian male with vitamin D deficiency and a history of cocaine use

a,c,e, Explanation: Patient A has the three major risk factors of gender (male), cigarette smoking, and obesity. Recent studies claim that obesity is a risk factor for hypertension because of the activation of the RAAS and SNS in adipose tissue. The nicotine in cigarettes causes SNS activation and vasoconstriction.

Which structures is/are located in the lower respiratory tract? Select all that apply a. Alveoli b. Larynx c. Pharynx d. Trachea

a,d

The nurse is caring for a patient who is receiving oxygen therapy via the venturi mask. Which complications are associated with oxygen therapy? (select all that apply) a. Oxygen toxicity b. Cachexia c. Bradycardia d. Absorption atelectasis e. Increased dead space

a,d Explanation: High levels of oxygen can damage the lungs. The high levels of oxygen can cause damage to the alveolar-capillary membrane and inactive surfactant production in the lungs. Absorption atelectasis occurs when high levels of oxygen are administered and cause alveoli collapse. The high levels of oxygen dilute the nitrogen that normally prevents alveoli from collapsing and replaces nitrogen with oxygen in the alveoli.

What factors would be considered modifiable? (select all that apply) a. Cigarette smoking b. Gender c. Race d. Diabetes e. Stress

a,d,e

Mr. Smith's BP increases his risk of heart disease through what action(s)? (select all that apply) a. Prolonged increased afterload b. Prolonged increased stress on vasculature c. Increased cardiac workload d. Increased automaticity e. Decreased contractility

a,e Explanation: Prolonged increases in afterload that occurs with hypertension increases workload on the heart and hypertension produces prolonged stress on the vasculature; both increase the risk of heart disease. Decreasing contractility occurs as a result of heart disease. Increased automaticity is not associated with hypertension.

A patient presents to the emergency room. Upon assessment you discover the patient has dyspnea, crackles on auscultation, pale color weak pulses, cool temperature in extremities, delayed capillary refill, fatigued and weak. What is your suspected diagnosis? a. Left sided heart failure b. Right sided heart failure c. Myocardial infarction d. Nothing is wrong with the patient

a.

A valvular disease with stiffening and thickening of the valve leaflets, caused by calcium deposits or scarring, narrow the opening and obstruct flow is what? a. Stenosis b. Regurgitation or insufficiency c. Prolapse

a.

Blood pressure is controlled by the following factors: a. Vascular resistance, blood volume b. Systolic pressure, diastolic pressure c. RASS, blood viscosity d. Vasodilation, diastolic pressure

a.

Inhaled air is warmed and humidified in the: a. Oropharynx b. Trachea c. Bronchioles d. Alveoli

a.

Perfusion at the tissue level is done through the abundant network of: a. Capillaries b. Arteries c. Arterioles d. Venules

a.

The nurse is screening patients for their risk of developing heart disease. The nurse should consider which patient at greatest risk? a. A 60-year-old obese male smoker with a family history of heart disease b. A 40-year-old female who drinks two glasses of wine every day c. A 70-year-old male smoker with a long-standing history of hypertension d. A 50-year-old female smoker with a family history of heart disease

a. Explanation : Patient has three risk factors for heart disease

During the assessment of a pt admitted with pneumonia, the nurse notes that the pt is experiencing agitation and restlessness not evident on prior assessments. What should the nurse's initial response be to this assessment finding? a. Obtain an oxygen saturation measurement with pulse oximetry b. Notify the physician c. Auscultate breath sounds d. Comfort and calm the patient

a. Explanation: Agitation and restlessness may be a sign of decreased oxygen levels. After completing the assessment, which would include auscultating breath sounds, the nurse should notify the physician and start oxygen, all the while attempting to comfort and calm the patient.

What is the most likely procedure to determine the cause of severe chest pain in the patient newly admitted to the hospital? a. Coronary angiography b. Nuclear stress testing c. Right heart catheterization d. TEE

a. Explanation: Coronary angiography is done to specifically evaluate coronary arteries. Stress testing Evaluates heart functioning during times of increased workload. Right heart catheterization evaluates cardiac filling pressures, cardiac output and valvular function. Transesophageal echo evaluates the posterior aspect of the heart and the presence of clots.

The nurse correlates which clinical manifestation observed relates to hypoxia? a. Shortness of breath b. Chills c. Productive cough d. Fever

a. Explanation: Early signs of hypoxia including a change in mental status, anxiety, restlessness, and/or confusion because the cells in the brain are not receiving an adequate supply of oxygen. An increase in respiratory rate and shortness of breath are also associated with hypoxia as the body attempts to increase oxygen delivery. Fever, chills and productive correlate to possible pneumonia, and hypoxia does not always manifest with pneumonia.

What is the composition of the heart? a. Four chambers with four valves that control flow through the heart and lungs through changes in pressure b. Four chambers and four valves that control flow through the heart and lungs through changes in oxygen levels c. Two chambers on the right receiving blood from the high-pressure venous system and two chambers on the left sending blood into the low-pressure arterial system d. Two chambers on the right receiving oxygenated blood from the venous system and two chambers on the left receiving deoxygenated blood from the pulmonary circuit

a. Explanation: Flow through the heart is controlled through changes in pressure, not oxygen. The arterial system is a high-pressure system; the venous system is a low pressure system. Deoxygenated blood is returned to the right heart to be circulated through the lungs for gas exchange - oxygenated blood then flows through the left heart to the systemic circulation.

A patient who is receiving mechanical ventilation is at risk for which complications? a. Hypotension, infection, and barotrauma b. Anxiety, hypertension, and fatigue c. VAP, atelectasis, and hypothermia d. Dyspnea, hypoxia, and normothermia

a. Explanation: Hypotension: Secondary to changes in the pressure in the chest cavity. This increased pressure in the chest causes a decrease in venous return and can ultimately decrease CO. Infection: Because the normal defense of the upper and lower airway are bypassed. The ETT leads directly into the lungs and can introduce infection. Barotrauma: Due to the increased positive pressure that can cause alveolar rupture.

The nurse has just received a report on assigned patients. Which of the following patients should be assessed first? a. The patient with indigestion and increased troponin levels b. The patient with indigestion and increased CK levels c. The patient admitted 2 days ago with a BNP of 75 pg/mL d. The patient admitted 2 days ago with increased LDLs and C-reactive protein

a. Explanation: Indigestion is sometimes an indicator of myocardial infarction, and increased troponin levels are an indicator of acute injury. Increased CK is nonspecific for cardiac injury so the patient probably just has indigestion. The patient with a BNP of 75 two days after admission without shortness of breath is not acutely worrisome.

The nurse is caring for a patient who has just returned from the post-anesthesia care unit (PACU) after having a tracheotomy performed. What is the nurse's first priority? a. Maintain a patent airway b. Assess for a pneumothorax c. Attach a trach collar to provide humidified oxygen d. Suction the patient's airway

a. Explanation: Maintaining a patent airway is always priority for all patients. Without a patent airway ventilation and perfusion cannot take place effectively.

Prior to admission to the unit, Mr. More's BP is 180/122 mm Hg and he is complaining of a headache and blurry vision. These signs and symptoms combined with his laboratory results suggest a concern for which of the following? a. Hypertensive emergency b. Stroke c. Hypertension urgency d. Diabetes

a. Explanation: Mr. More meets the criteria of a hypertensive emergency due to his diastolic BP of 122 and indications of target organ damage. A patient with hypertensive urgency does not have signs of TOD. Diabetes is considered when the blood sugar is consistently high requiring medication and/or diet and exercise for control. Stroke is considered when there are signs of neurological damage such as weakness, loss of vision, dizziness, ataxia, or dysphasia

Upon admission, Mr. More's BP is 188/100 mm Hg. Which order is most important for the nurse to implement first? a. Administer labetalol 400 mg PO. b. Obtain a finger stick glucose reading. c. Begin oxygen via nasal cannula at 2 L. d. Administer Lipitor 40 mg PO.

a. Explanation: Mr. More's antihypertensive medication is the priority to start to decrease his blood pressure and prevent risk of an acute event. A glucose level and administering his statin are important, but not immediate priorities. He does not currently need O2 because his sat is 96%.

A nurse is providing Patient education regarding OSA to a newly diagnosed patient. Patient understanding is indicated by which of the following statements? a. "I guess I should really lose weight" b. "I do not have to lose weight as long as I can use CPAP at night" c. "A glass of wine in the evening will help me sleep through the night." d. "I will not have to take my blood pressure meds if I use CPAP at night"

a. Explanation: Obesity is a risk factor for the development of sleep apnea. All of the other statements are false

The nurse includes which nursing diagnosis in the plan of care for the patient admitted with pneumonia? a. Anxiety related to hypoxia b. Deficient knowledge related to cause of disorder c. Fear related to the threat of pain d. Ineffective breathing pattern related to decreased surfactant

a. Explanation: Patients experiencing hypoxia usually have signs of irritability, restlessness, anxiety and confusion. As the brain cells receive less oxygen the patient becomes more confused.

The nurse should intervene when a pt who is being admitted to the unit with the diagnosis of COPD is noted to have which clinical manifestation? a. Confusion b. Depression c. Cough d. Fatigue

a. Explanation: Patients experiencing hypoxia usually have signs of irritability, restlessness, anxiety and confusion. As the brain cells receive less oxygen the patient becomes more confused.

You are caring for a patient who is currently receiving continuous pulse oximetry. Which of the following findings might indicate inaccurate readings? a. The patient has a core body temperature of 94 F (34.5 C) b. The heart rate obtained through pulse oximetry correlates with the electrocardiogram heart rate. c. The patient has the probe attached to her earlobe d. The patient's pulse oximetry is 95% and the patient denies shortness of breath

a. Explanation: Pulse oximetry values obtained in individuals whose core body temperature is below normal levels should be interpreted cautiously because hypothermia is known to affect pulse oximetry values. All of the other findings are appropriate and do not impact the accuracy of the pulse oximetry reading.

A patient with laryngeal cancer is receiving radiation therapy. The nurse determines that teaching has been effective if the patient states which of the following? a. "I should avoid exposure to the sun" b. "I will scrub the radiation site dail to remove dead skin" c. "I should apply lotion twice a day to the radiation site" d. "My clothing should be snug fitting and made from polyester"

a. Explanation: Radiation therapy damages the skin, increasing the risk for sunburn and addition skin irritation

The nurse is caring for a patient who has just been orally intubated. Which action should the nurse take first? a. Assess for symmetrical chest rise and fall with ventilation b. Provide mouth care to assure a clean oral cavity c. Assess the need for nutritional support by an oral nasogastric tube d. Provide a communication board for the patient and family

a. Explanation: The first action is to assess endotracheal tube (ETT) placement. The nurse verifies proper placement by assessing for symmetrical chest rise and fall with ventilation, equal and bilateral lung sounds, and absence of sounds over the abdomen. If breath sounds and lung expansion are only present on the right side the ETT may have been placed in the right main bronchus. Right mainstem bronchus intubation occurs because the right bronchus has a lesser angle to the trachea and is often wider than the left bronchus. In the event of a right mainstem intubation, the ETT needs to be repositioned and reassessed for correct placement.

The nurse is caring for a patient following a thoracentesis. Which of the following indicates the patient needs additional teaching regarding his or her discharge instructions? a. "I will be short of breath following the procedure, and my symptoms should resolve by the morning." b. "There may be a small amount of drainage noted on the dressing" c. "I will come back to the hospital if I experience chest pain." d. "I can eat my normal diet following discharge."

a. Explanation: The pt should not report SOB following the procedure and this indicates that the pt needs additional teaching about the procedure. The other statements demonstrate appropriate understanding of the procedure.

The nurse recognizes that the second physical compensatory Mechanism of decreased oxygen delivery is which of the following? a. Increased heart rate b. Increase respiratory rate c. Decrease heart rate d. Decreased respiratory rate

a. Explanation: The second compensatory mechanism is increasing heart rate. The first compensatory mechanism is increased respiratory rate. A decreased heart rate or decreased respiratory rate would hinder oxygen therapy.

What statement demonstrates adequate teaching for a pt with TB? a. "Wow! I'm going to be on these drugs for a long time!" b. "I can stop taking the drugs when I feel better." c. "My family doesn't need to wear a mask when they come to visit." d. "I'm looking forward to going down to the cafeteria to get something to eat."

a. Explanation: This patient will be on this drug therapy for 9-12 months. They can't stop taking the drugs until the treatment period is complete. Family members need to wear a mask for protection and the pt can only leave his room for essential procedures.

The movement of oxygen and carbon dioxide in caused by: a. The level of CO2 and O2 in the vascular system b. The pressure gradient between the vascular system and the alveoli c. The amount of CO2 in the alveoli d. The force of air in the tracheobronchial tree

b

You are caring for a patient who is currently receiving continuous pulse oximetry. Which of the following findings might indicate an inaccurate reading? a. The patient has a core body temperature of 97.8 F (36.6 C) b. The heart rate obtained through pulse oximetry does not correlate with the ECG heart rate c. The pt has the probe attached to their index finger d. The pt's pulse oximetry is 95% and the pt denies shortness of breath

b

In assessing for the presence of OSA, the nurse is aware that there are numerous signs and symptoms. Select the signs and symptoms from the list below. (select all that apply) a. Hypotension b. Daytime sleepiness c. Insomnia d. Cardiac dysrhythmias e. Increased appetite

b,c,d Explanation: All of these are symptoms frequently seen in pts experiencing sleep apnea.

The nurse is aware that there are several causes of laryngitis. Which of the following causes laryngitis? (select all that apply) a. Food allergies b. Upper respiratory infection c. Environmental pollutants d. Gastroesophageal reflux e. Obstructive sleep apnea

b,c,d Explanation: Food allergies are not limited to just the larynx but involve the complete Airway system. OSA does not cause laryngitis.

Priority teaching needs for Mr. More include which of the following? (Select all that apply.) a. Anticoagulation therapy b. Smoking cessation c. DASH diet d. Slow posture changes e. Eat a banana a day

b,c,d,e Explanation: Smoking cessation and DASH diet are two modifiable risk factors that can improve Mr. More's outcome. Slow posture changes are important when on a beta blocker to reduce orthostatic hypotension and risk of fainting. Because Mr. More is on Lasix he will need to include more K+ in his diet. Mr. More is in sinus rhythm at this time and does not require anticoagulation treatment. Insulin treatment at this time is not necessary. Diabetes can be controlled with diet

After 4 months of using the CPAP machine, the pt determines that he would like to try to sleep without it. He feels slightly claustrophobic and thinks the weight loss may have improved his OSA. What does the nurse ask the pt and wife to monitor if he stops using the CPAP machine? (select all that apply) a. Increased belching b. Irritability c. Snoring d. Voice quality e. Blood pressure

b,c,e Explanation: Increased snoring and irritability from lack of sleep are signs he may still need his CPAP machine. Hypertension is associated with OSA.

What are the benefits of a venturi mask? (select all that apply) a. It delivers low levels of oxygen b. It minimizes rebreathing exhaled CO2 c. It does not require high flow rates of oxygen d. It provides a more precise concentration of oxygen e. It provides higher concentrations of oxygen

b,d,e Explanation: The venturi mask is the most commonly used high flow delivery device because this device delivers the most accurate oxygen concentration. This device can deliver oxygen concentrations of 24% - 50% by using different adaptors and by adjusting the oxygen concentration from 4-10 L/min. The adaptor indicated what the flow rate of oxygen should be maintained to provide the prescribed percentage to the patient.

A patient presents to the emergency room. Upon assessment you discover the patient has dependent edema, hepatomegaly, ascites and a jugular vein distention. What is your suspected diagnosis? a. Left sided heart failure b. Right sided heart failure c. Myocardial infarction d. Nothing is wrong with the patient

b.

A patient presents with SOB, angina, syncope, dizziness, dysrhythmias, weight gain, edema, cool, pale extremities. What are all of these signs due to? a. An infection b. Decreased cardiac output c. Stroke d. Fever

b.

A valvular disease that includes blood flows or leaks backward - ventricle to atria, aorta to the left ventricle, pulmonic circulation to the right ventricle - because of incomplete closing of the valve is what? a. Stenosis b. Regurgitation or insufficiency c. Prolapse

b.

After a percutaneous coronary angioplasty, what assessment should most concern the nurse? a. Back discomfort b. Chest pain c. Capillary refill of less than 3 seconds d. Hypoactive bowel sounds

b.

Arteries bring and from the heart to tissues. a. deoxygenated blood and electrolytes b. oxygenated blood and nutrients c. red blood cells and waste products d. deoxygenated blood and nutrients

b.

The nurse hears a loud systolic murmur at the second intercostal space right sternal border. What valve problem is the patient likely experiencing? a. Aortic regurgitation b. Aortic stenosis c. Pulmonic regurgitation d. Pulmonic stenosis

b.

The nurse is reviewing orders for a newly admitted patient with PAD in the right lower extremity. The nurse should follow up with the provider about which order? a. Begin plavix 75 mg PO daily b. Keep affected extremity elevated c. Begin lisinopril 10 mg PO daily d. Encourage light exercise as tolerated

b.

What is the % of O2 in room air? a. 15% b. 21% c. 35% d. 70%

b.

A pt with MRSA pneumonia can be cohorted with which of the following pts? a. A pt with an MRSA-infected foot ulcer b. A pt with a confirmed influenza infection c. A postoperative pt with open reduction/internal fixation (ORIF) of the right hip d. A COPD pt with chronic obstructive pulmonary disease who is receiving steroids

b. Explanation: A pt with pneumonia cannot be in a room with a postoperative pt or a COPD pt on steroids due to the risk of infection for those pts. The pt with an MRSA foot ulcer does not need droplet precautions as a pt with influenza.

The nurse should question which order concerning a pt with TB? a. Humidified oxygen via nasal cannula b. NPO c. Vital signs with oxygen saturation every 4 hours d. Activity as tolerated

b. Explanation: Adequate nutrition is essential for recovery. Humidified oxygen protects the airways and maintains oxygen levels, vital signs are necessary to monitor his recovery, and light activity in the room as tolerated is encouraged.

A nursing preceptor is observing a newly hired nurse prioritizing patient rounds. Correct prioritization of assigned patients is indicated if the new nurse assesses which patient first? a. A patient with cellulitis of the right lower leg b. A patient who was extubated 2 hours ago c. A patient who was ordered nebulizer treatment prn d. A patient with an IV of D5 ½ NS at 125 mL/hr

b. Explanation: After extubation it is important for the nurse to frequently check the patient's respiratory status due to respiratory complications.

What is a priority nursing diagnosis for a patient with lung cancer? a. Impaired nutrition b. Pain control c. Impaired mobility d. Risk for infection

b. Explanation: All are important factors to consider in the treatment of lung cancer patients, but pain control is of priority importance in a diagnosis with such a poor prognosis

What is the priority treatment for a patient with cystic fibrosis? a. Nutritional counseling b. Airway clearance c. Medication administration d. Pt education

b. Explanation: Although nutritional counseling, medication administration, and pt education are essential, airway and breathing are always the priority.

Mr. Jones is undergoing an open lung biopsy in the AM, and the nurse is educating the pt regarding the postoperative period. Which of the following statements should be include in the education? a. "You will be discharged home immediately following the procedure" b. "You most likely will have a chest tube inserted to allow your lung to re-expand" c. "You will need to collect a sputum specimen" d. "You will not need any supplemental oxygen"

b. Explanation: An open lung procedure requires the use of a chest tube following the surgical procedure, and the pt should be educated about the need for a chest tube prior to the procedure. Pts who have an open biopsy are typically in the hospital for a day following the procedure and thus are not immediately D/Ced home. The use of supplemental O2 will need to be assessed following the procedure and may be required in the immediate recovery period.

What laboratory values are significant indicators of acute heart injury? a. Increased CK and myoglobin b. Increased CK-MB and troponin c. Decreased BNP and platelets d. Increased CK with low levels of CK-MB

b. Explanation: CK-MB and troponin are increased in acute injury, CK and myoglobin are nonspecific for cardiac injury, increased BNP is an indicator of heart failure. Increased CK is not specific for heart injury. Low levels of CK-MB indicates the damage is not cardiac.

Effective management of the asthmatic patient requires which of the following? a. Pulmonary rehabilitation and anti inflammatories b. Regular assessment and control of environmental factors c. Short-acting bronchodilators and stress reduction d. Weight control and regular assessment

b. Explanation: Effective management of the asthmatic pt should consist of regular assessment and control of environmental factors. Pulmonary rehabilitation, anti-inflammatories, stress reduction and short-acting bronchodilators and weight control are not all components of the management of the management of the asthmatic patient.

Which statement by the patient about his medication regimen indicates the need for further teaching? a. "I will call my doctor if I am dizzy and short of breath." b. "I will take my BP medication only if my blood pressure is up." c. "One of my medications, Lasix, will make me urinate a lot." d. "I am able to take my labetalol and lisinopril at the same time."

b. Explanation: His care provider should give him parameters, but the patient should take his BP medication even if his BP is in the normal range to keep a consistent effect as long as he is asymptomatic.

A patient with hypertension has which physical symptom? a. Decreased resistance, which may increase CO b. Increased resistance, which may decrease CO c. Increased resistance, which may increase CO d. Decreased resistance, which may decrease CO

b. Explanation: Hypertension increases resistance. Increase resistance may decrease cardiac output. Decreased resistance may increase cardiac output.

When should palliative care for patients diagnosed with lung cancer be initiated? a. Upon diagnosis b. After chemotherapy c. After surgery d. Upon patient request

b. Explanation: Ideally palliative care for the pt diagnosed with lung cancer should be initiated upon diagnosis due to poor prognosis.

The nurse should intervene immediately if patient with DVT is noted to: a. Have redness and warmth in the lower calf of right leg b. Experience new shortness of breath and a decrease in O2 sat c. Have pain and tenderness in right thigh d. Begin having nausea and diarrhea

b. Explanation: If the patient is experiencing new shortness of breath and a decrease in O2 there is concern for a PE which is an emergency and needs immediate intervention. The redness and warmth in the lower calf of the right leg is having normal signs and symptoms of the DVT. Nausea and diarrhea are unrelated to DVT.

What is the rationale for ordering Pepcid (famotidine) 20 mg IV push daily when a patient is intubated and placed on the mechanical ventilator? a. To decrease inflammation in the airways b. To reduce gastric acidity c. To minimize constipation d. To enhance gastric emptying

b. Explanation: Medications that reduce gastric acidity have been shown to protect patients from developing peptic ulcer disease and gastrointestinal bleeding due to the stress of intubation. Steroids are indicated to decrease inflammation and stool softeners to minimize constipation. Medications to enhance gastric emptying are not typically indicated in ventilated patients.

Which of the following indicates an abnormal assessment finding of the respiratory system? a. Symmetrical chest expansion b. Muscle restrictions with inhalation c. The sound of resonance with percussion of lung tissue d. Pink mucosa of the mouth and nares

b. Explanation: Muscle retractions during inhalation indicate that significant respiratory effect is being demanded and thus use of the accessory muscles is required. All of the other assessment findings are normal assessment findings.

The nurse is reviewing orders for a newly admitted patient with PAD in the right lower extremity. The nurse should follow up with the provider about which order? a. Begin Trental 400 mg PO twice daily with meals. b. Keep the affected extremity elevated. c. Begin lisinopril 10mg PO daily. d. Encourage light exercise as tolerated.

b. Explanation: The affected extremity should be kept dependent to maintain adequate blood flow.

The provider has ordered an ABG analysis for a patient. The nurse has selected the left radial artery to collect the specimen, Which of the following is essential to ensure adequate perfusion during the procedure? a. Placing the specimen immediately on ice following the collection b. Assessing for left ulnar artery perfusion c. Holding pressure for 30 seconds post procedure d. Assessment of the radial pulse

b. Explanation: The nurse needs to assess perfusion of the ulnar artery prior to completing the arterial blood gas, since disruption of the radial arterial during specimen collection without sufficient ulnar artery blood supply results in hypoxia of tissue to the hand. Assessing radial pulse doesn't demonstrate adequate perfusion during the procedure.

Providing care for a patient with potential laryngeal trauma, and priority of care is: a. Monitoring oxygen saturation every 2 hours b. Having an emergency tracheostomy tray at the bedside c. Continue monitoring d. Ensuring adequate oral intake of food

b. Explanation: The patient is at risk for loss of a Patent Airway so emergency tracheostomy or intubation equipment should be at the bedside

The nurse should intervene if a patient with AAA is noted to experience: Mild back pain, increased BP, decreased RBC Severe lower back pain, decreased BP, decreased RBC Intermittent lower back pain, decreased BP, decreased RBC Severe lower back pain,increased BP, increased RBC

b. Explanation: The patient is having an aortic dissection. The nurse identifies this due to classic clinical signs and symptoms. The shearing force and tearing of the aorta causes severe lower back or abdominal pain. BP decreases due to hypovolemia from blood loss from the central venous system. RBCs decrease due to profuse blood loss.

A patient with COPD is given a high dose of oxygen in response to a low SPO2. What is the priority intervention for this pt? a. Cardiovascular assessment b. Assessment of respiratory rate c. Medication administration d. Respiratory therapy referral

b. Explanation: The primary assessment/intervention for the COPD pt receiving high levels of oxygen is the respiratory rate. The respiratory rate will decrease due to the fact that his breathing is stimulated by a decrease in his oxygen level. With high levels of oxygen, the patient has lost his stimulus to breathe.

The nurse recognizes that the first physical compensatory someone mechanism of decreased oxygen delivery is which of the following? a. Increase heart rate b. Increased respiratory rate c. Decrease respiratory rate d. Decrease heart rate

b. Explanation: The second compensatory mechanism is increasing heart rate. A decrease heart rate or decreased respiratory rate would hinder oxygen delivery

The nurse anticipates what procedure for the patient at risk for stroke because of the potential presence of blood clots in the right atrium? a. Nuclear stress test b. TEE c. Coronary angiography d. CXR

b. Explanation: Transesophageal echocardiogram can identify the presence of clots, nuclear stress test identifies areas of poor perfusion, coronary angiography looks at blockages in the coronary arteries, and chest x-ray is a general screening exam which also can identify long injury or infection.

The nurse is screening patients for their risk of developing VAP. The nurse considers which patient at greatest risk? a. A patient who was extubated within 24 hours of being intubated b. A patient intubated and placed on mechanical ventilation less than 72 hours ago c. A patient with the head of the bed elevated 45 degrees d. A patient who was placed on a nasal cannula after being extubated

b. Explanation: Ventilator-associated pneumonia (VAP) is a serious health care associated infection, resulting in high morbidity, high mortality, and high costs of treatment. Aspiration of oropharyngeal and/or gastric fluids is presumed to be an essential step in the development of VAP. VAP typically develops between 48 to 72 hours after endotracheal intubation.

The nurse understands the priority intervention with the patient with TB is which of the following? a. Antibiotic Administration b. Initiation of isolation c. TB test d. Chest x-ray

b. Explanation: all right now it is imperative that the TB patient be isolated as soon as possible to prevent the spread of the disease.

A nurse providing care for a pt whose BP readings are consistently 130/85 mm Hg should anticipate which medical plan of care? (select all that apply) a. Diagnostic testing for TOD b. Initiation of diuretics c. Modifications of diet and exercise d. Echocardiogram e. A stress test

c

The function of the respiratory system is to: a. Transport oxygen and carbon dioxide to the systemic system b. Maintain the acid-base c. Provide oxygen to and remove carbon dioxide from the alveoli d. Clear the system of mucous

c

Which diagnostic test/exam would best measure a client's level of hypoxemia? a. Chest x-ray b. Pulse oximeter reading c. Arterial blood gas d. Peak expiratory flow rate

c

What factors would be considered non modifiable? (select all that apply) a. High LDL b. Diabetes c. Gender d. Race e. Age

c,d,e

A nurse is performing the immediate postoperative assessment of a patient who has just undergone CEA. What is the most important assessment to be reported immediately? a. A complaint of 7/10 pain b. Falling back to sleep after assessment c. An asymmetric smile d. Complaint of a sore throat

c.

A patient has been admitted to the hospital for a PE. What is the priority nursing intervention? a. Insert an IV line b. Begin heparin drip as ordered c. Check oxygen saturation d. Determine patient allergies

c.

A valvular disease that includes valve leaflets bulge backward and do not close, causing regurgitation is what? a. Stenosis b. Regurgitation or insufficiency c. Prolapse

c.

The first line natural protective mechanism for the upper respiratory tract consist of: a. Mucociliary escalator b. External environment interface c. Vibrissae d. Adenoids

c.

The mechanism that moves particles to be expelled by sneezing and coughing is called: a. Cilia b. Epithelium c. Mucociliary escalator d. Oropharynx

c.

The nurse determines which patient is at greatest risk for developing Infective Endocarditis (IE)? a. A 22 y/o student undergoing a dental procedure b. A 35 y/o man with a past medical history of IV drug use c. A 65 y/o male heart transplant patient on immunosuppressive therapy undergoing a colonoscopy d. A 70 y/o female with heart failure with an intravascular access device for home infusion

c.

What information should be included in the teaching plan for a 67-year old patient after a physical? a. Initiating a new strenuous exercise regimen is recommended b. Limit physical activity and exercise c. Report any new or excessive fatigue d. Excessive fatigue is not unusual as you age

c.

What is an important nursing action following a cardiac catheterization intervention? a. Early mobilization to prevent clot formation b. Fluid restriction to avoid fluid overload c. Bedrest to avoid stress on cannula insertion site d. Head of the bed at 30 degrees for respiratory support

c.

Which of the following are the most important respiratory structures for gas exchange? a. Bronchioles b. Trachea c. Alveoli d. Epithelium

c.

Which order would the nurse question in the initial management of acute pericarditis? a. Aspirin b. Colchicine c. Prednisone d. NSAIDs

c.

Which statement by the pt about the need for antihypertensive medication indicates the need for further teaching? a. "I'm worried about a stroke if my BP is not controlled." b. "Can my kidneys fail if I don't control my BP?" c. "My BP is only slightly elevated, so I am okay." d. "I guess I need to take this medication even if I feel OK."

c.

The nurse is caring for John, a 42-year-old male who was admitted to the hospital for pneumonia. He has a past medical history of hypertension. Which of the following abnormal assessment findings should the nurse expect as a result of the history obtained from John? a. An AP to lateral diameter of 1:2 b. Vesicular lung sounds over the trachea c. A productive cough d. Symmetrical chest expansion

c. Explanation: A productive cough while an abnormal assessment finding is expected in a patient who has pneumonia. The other findings are normal assessment findings.

A patient has been admitted to the hospital for a PE. What is the priority nursing intervention? a. Insert an IV line. b. Begin heparin drip as ordered. c. Check oxygen saturation. d. Determine patient allergies.

c. Explanation: Airway and breathing is the priority in this situation as PE can often cause a decrease in O2 saturation and severe SOB. After that you would want to confirm that the patient has working central line and/or IV access, check allergy status for heparin allergy, and then initiate ordered heparin bolus and continuous infusion to prevent further clot formation.

The nurse assesses three asthmatic pts who present to the emergency department with the following symptoms. Which pt should the nurse treat first? a. The pt with an oxygen saturation (SPO2) of 91% b. The pt with a peak flow less than personal best c. The pt with inspiratory and expiratory wheezes d. The pt able to speak in short sentences

c. Explanation: All are symptoms of respiratory compromise in asthma, but the pt with inspiratory and expiratory wheezes is demonstrating a severe exacerbation of asthma needing immediate treatment.

Altered levels of sodium, potassium, and calcium may result in what condition? a. Fluid loss b. Fluid retention c. Dysrhythmias d. Shortness of breath

c. Explanation: Altered levels of one or all three will produce dysrhythmias due to disruptions in the action potential. Increased sodium can produce fluid retention or shortness of breath. Decreased sodium may produce fluid loss resulting in decreased potassium.

A nurse is performing the immediate postoperative assessment of a patient who just underwent CEA. What is the most important assessment to be reported immediately? a. A complaint of 7/10 pain b. Falling back to sleep after assessment c. An asymmetric smile d. Complaint of a sore throat

c. Explanation: An asymmetric smile may indicate damage to cranial nerve seven—the physician should be made aware immediately. Sleepiness is normal postanesthesia, a sore throat is not unusual post-intubation, and pain should be treated after notifying the physician about the asymmetric smile.

A patient undergoes a surgical repair of a laryngeal fracture. Immediately following surgery, which intervention has the highest priority? a. Assessing the surgical site for hemorrhage b. Assessing the suture line for infection c. Monitoring respiratory status for signs of obstruction d. Asking the patient to speak to assess for pain

c. Explanation: As a result of the trauma and surgical procedure, inflammation of the laryngeal and surrounding tissue can obstruct the airway.

What information should be in the teaching plan of a 76-year-old pt after a physical? a. Initiating a new strenuous exercise regimen is recommended b. Limit physical activity and exercise c. Report any new or excessive fatigue d. Excessive fatigue is not unusual as you age

c. Explanation: Excessive fatigue is not normal and may indicate the onset of heart failure. While physical activity should be continued, even if healthy, a new strenuous exercise regime should be carefully monitored.

A pt is started on an antihypertensive after his diagnosis of hypertension. Which statement indicates he understands his education regarding the new drug? a. "I can stop taking this medication when I start sleeping better." b. "Sleeping through the night is the most important thing to consider." c. "I'm glad we found out I have high blood pressure so I can treat it!" d. "This drug will help me sleep better."

c. Explanation: HTN is a great risk factor for heart disease and stroke and should be treated. The meds need to be taken as prescribed. Sleeping is important, but the high risk for stroke and heart disease caused by HTN is very important.

The nurse is reviewing orders on the patient who is to receive supplemental oxygen. It is a priority for the nurse to follow up with the provider about which order? a. Nasal cannula at 2 L/min b. Venturi mask at 24% concentration c. Nasal cannula at 8L/min d. Venturi mask at 50% concentration

c. Explanation: Nasal cannula oxygen should not be ordered at a level greater than 6 L/min NC. Rates greater than 6L/min NC do not increase oxygenation due to the anatomic dead space being full. Greater levels of oxygen causes irritation to mucous membranes.

The nurse monitors for which complication in a patient receiving oxygen therapy? a. Hyperventilation b. VAP c. Drying of mucous membranes d. SaO2 level greater than 94%

c. Explanation: Oxygen has a drying effect on the mucous membranes because the humidification of inspired air will be lessened for patients receiving oxygen therapy without humidification.

The nurse correlates which ventilator setting is most important in preventing alveolar collapse in an intubated patient? a. FIO2 of 100% b. VT of 550 mL c. PEEP of 5 cm H2O d. SIMV with rate of 12

c. Explanation: Positive and expiratory pressure (PEEP) is positive pressure applied at the end of expiration to help prevent alveolar collapse. PEEP improves oxygenation, allowing the FIO2 level to be lowered, FIO2 correlates to the concentration of oxygen being delivered. Tidal volume is the volume of air delivered with each mechanical breath, and SIMV is a ventilator mode that synchronizes intermittent mandatory ventilation breaths with the patient's spontaneous breaths.

The nurse is caring for a client with seasonal allergy-related rhinitis. The nurse correlates which of the following symptoms to rhinitis? a. Productive cough b. Earache c. Rhinorrhea d. Headache

c. Explanation: Rhinorrhea is an increase in nasal drainage and is often seen in allergic response to season allergens.

Which statement by the patient about the need for anti-hypertensive medication indicates the need for further teaching? a. "I'm worried about a stroke if my BP is not controlled." b. "Can my kidneys fail if I don't control my BP?" c. "My BP is only slightly elevated so I am okay." d. "I guess I need to take this medication even if I feel okay."

c. Explanation: Stroke and renal failure are complications of hypertension. Anti-hypertensives need to be taken all of the time! Even slight elevations in BP, prehypertension, require lifestyle changes.

What is the primary difference between emphysema and chronic bronchitis? a. Chronic bronchitis predominantly affects the large airways. b. Emphysema predominantly affects the large airways. c. Emphysema predominantly affects the alveoli. d. Chronic bronchitis predominantly affects the alveoli

c. Explanation: The alveoli are the part of the tracheobronchial tree affected by emphysema. The airways are affected by chronic bronchitis because of the increased production of mucous. The alveoli are affected in emphysema due to the air trapping and premature closing of the airways.

An asthmatic patient presents with which symptoms? a. Cough, elevated blood pressure b. Decreased respirations, fatigue c. Decreased respirations, wheezes d. Increased sputum, decreased respirations

c. Explanation: The asthmatic pt presents with increased respirations and wheezes. The pt may have a cough, but does not always have an elevated BP. The pt does not typically present with decreased respirations. He may be fatigued.

Which is true of the electrical conduction system of the heart? a. It is primarily controlled by the movement of uncharged ions b. It has a positive resting membrane potential c. It is reflected in the waveforms on the electrocardiogram d. It requires cells that respond only to stimulus from the autonomic nervous system

c. Explanation: The electrical conduction system is controlled by the movement of charged ions across the cell membrane, has a negative resting membrane potential, and is composed of cells with the property of automaticity, which is reflected in the waveforms on the electrocardiogram.

What is the predominant diagnostic test for lower airway disorders? a. Arterial blood gases b. Chest x-ray c. Pulmonary function tests d. Oxygen saturation

c. Explanation: The major diagnostic test for lower airway disorders is the pulmonary function test which assesses air movement. Although important in evaluating lung disease, arterial blood gases and oxygen saturation are not diagnostic. A chest x-ray identifies pulmonary lesion or consolidation.

The nurse understands adequate teaching has been done by which of the following statements? a. "I got a flu shot last year on cover for a while," b. "I don't need a flu shot I never get sick" c. "I guess it's important to get a flu shot every year" d. "I don't get a flu shot but I make sure my kids get one"

c. Explanation: The patient and children should get a flu shot every year

The nurse is caring for a patient post carotid endarterectomy that is experiencing new onset hypotension. Which of the following actions should the nurse plan to take first? a. Assess Glasgow coma scale b. Head of bed at 30° c. Administer fluids as ordered d. Assess cranial nerves

c. Explanation: The patient may be having a bleeding complication and will need fluids to help normalize BP while other definitive action is considered.

Acceptable room assignments for a pt with suspected or confirmed tuberculosis (TB) should be which of the following? a. Cohorted only with other tuberculosis TB pts b. Cohorted only with pts who are not infected with TB c. Isolated in a private room d. Isolated in a private room until confirmed and then cohorted once treatment has begun

c. Explanation: The patient with TB should always be isolated in a private negative pressure room

Which of the following structures actively participates in gas exchange? a. Bronchi b. Visceral pleura c. Respiratory bronchioles d. Terminal bronchioles

c. Explanation: The respiratory bronchioles participate in the princess of gas exchange (respiration). The other structures are involved in the process of ventilation or movement of oxygen to the level of the alveoli.

A nurse providing care for a patient whose BP readings are consistently 130/85 mm Hg should anticipate which medical plan of care? a. Diagnostic testing for TOD b. Initiation of anti-hypertensive therapy c. Modifications of diet and exercise d. Echocardiogram

c. Explanation: This patient is within the range for prehypertension. Hypertension can still be prevented with lifestyle changes or modifications such as DASH diet and an exercise regimen

The nurse is aware that the greatest risk factor for laryngeal cancer is: a. Age b. Alcohol abuse c. Tobacco use d. Occupational hazards

c. Explanation: Tobacco use is the highest risk factor development for laryngeal cancer

Which of the following assessment findings indicates an emergency and the need for immediate action on the part of the nurse? a. Clubbing of the fingernails b. Concentration of the diaphragm c. Tracheal deviation d. Auscultation of rhonchi throughout the lungs

c. Explanation: Tracheal deviation demonstrates unequal intrathoracic pressure and is associated with trauma of the lungs. Clubbing of the fingernails and rhonchi while abnormal do not require immediate action.

After a sleep study confirms that a pat has OSA, the nurse is preparing to provide pt education regarding home management. The nurse includes which of the following instructions for the pt? a. The use of albuterol nebulization treatments to promote bronchodilation b. Use of nasal cannula oxygen delivery during sleep c. Use of a CPAP machine during sleep d. Keeping the head of the bed flat during sleep

c. Explanation: Use of continuous positive airway pressure keeps the airways open preventing obstruction and apnea.

What is an important nursing action post cardiac catheterization interventions? a. Early mobilization to prevent clot formation b. Fluid restriction to avoid fluid overload c. Bedrest to avoid stress on cannula insertion site d. Head of bed at 30 degrees for respiratory support

c. Explanation: avoiding stress at the cannula insertion site by maintaining flat bed rest helps reduce the incidence of bleeding. Encouraging fluids helps clear the contrast dye.

The correct statement about the prevalence of pneumonia is which of the following? a. Pneumonia is no longer a major health problem b. The prevalence of pneumonia is increasing dramatically c. The prevalence and mortality are highest in persons older than 65 years d. The frequency has not declined but mortality has declined

c. Explanation: pneumonia is still prevalent and the mortality rate is highest in persons greater than 65 years of age

The exchange of oxygen and carbon dioxide takes place at the: a. Carina b. Mainstem bronchi c. Terminal airways d. Alveoli

d

The greatest oxygen saturation is found in which structure? a. Right subclavian vein b. Right atrium c. Pulmonary artery d. Left atrium

d.

The phagocytic action on Bacteria that gain access to the lower respiratory tract is carried out by which of the following cells? a. Red blood cells b. Platelets c. Scavenger cells d. Macrophages

d.

Which of the following is a finding attributable to age related changes? a. Kyphosis b. AP to lateral diameter of 1:1 c. PaCO2 of 47 on an ABG d. PaCO2 of 60 on an ABG

d. Explanation: Aging procedures many changes in the respiratory system. Kyphosis and a barrel chest are expected in older adults due to changes in the thoracic cage, loss of lung compliance, and changes in vertebrae from osteoporosis. Older adults may experience hypercapnia, and increased levels of PaCO2. A PaCO2 reading of 60 is an indication of hypoxemia and should be treated immediately.

The nurse is taking care of four patients admitted for uncontrolled hypertension. Which of the following patients should the nurse see immediately? a. The patient with a BP of 200/95 that denies any symptoms but has a strong family history of stroke b. The patient with a BP of 158/95 with an elevated BUN and creatinine complaining of nocturia c. The patient with a BP of 162/75 complaining of noticeable vision impairment in the left eye over the last 2 months d. The patient with a BP of 155/92 complaining of left sided weakness, facial drooping, and slurred speech

d. Explanation: Even though patient D's BP is lower than the other patients, patient D is experiencing symptoms of a right sided stroke and should be immediately placed on stroke protocol. This is an emergency. The other patients require significant intervention as well but are stable at this time and are experiencing chronic effects.

A pt with the diagnosis of COPD requires supplemental oxygen therapy because of hypoxemia. Which statement is true concerning oxygen therapy with a pt diagnosed with COPD? a. High oxygen concentrations have no effect on patients with COPD. b. The patient will become dependent on oxygen. c. There is no indication for supplemental oxygen for a patient with COPD. d. High oxygen concentrations may decrease the hypoxic drive to breathe.

d. Explanation: Patients who have a history of COPD may retain CO2. Gradually over time the patient's body adjusts to the higher level of CO2. The CO2 may no longer cause the respiratory drive in the patient with COPD. The lower levels of oxygen become the major respiratory stimulant. High doses of oxygen may take the respiratory drive away and cause hypoventilation.

When the nurse is performing postural drainage for the patient, which factor promotes the movement of secretions to the upper airways? a. Dilation of alveoli b. Action of the cilia c. Muscle contraction d. Force of gravity

d. Explanation: Postural drainage uses gravity to help remove secretions from smaller airways to larger airways.

In caring for pts with OSA, the nurse understands the system affected the most is which of the following? a. Neurological b. Respiratory c. Endocrine d. Cardiovascular

d. Explanation: Sleep apnea can result in hypoxia leading to hypertension, stroke, and lethal cardiac dysrhythmias such as ventricular fibrillation

Which statement is correct related to isolation and droplet precautions for TB? a. The pt should wear a mask at all times b. Staff entering the room should wear a surgical mask c. Visitors entering the room should wear an N95 mask respirator d. Staff entering the room should wear an N95 respirator

d. Explanation: Staff should wear an N95 fit tested mask respirator for protection when caring for a pt with TB. Family members can wear a surgical mask when visiting and the pt should wear a surgical mask if leaving the room for any reason.

Which of the following is not a function of the respiratory system? a. Acid-base balance b. Temperature regulation c. Fluid balance d. Tissue perfusion

d. Explanation: The circulatory system is responsible for tissue perfusion, not the respiratory system. Acid/base balance, temperature regulation, and fluid balance are all functions of the respiratory system.

Which findings correlate with worsening hypoxia? a. Decrease Paco2 b. Decreased respiratory rate c. Decreased heart rate d. Decreased Pao2

d. Explanation: The decreased Pao2 are most closely correlated to hypoxia. Decreased Paco2 is associated with hyperventilation. Clinical manifestations of hypoxia also include increased respiratory rate, heart rate, and dyspnea.

The nurse has provided pt education regarding the sleep study. Which statement made by the pt indicates he understands the procedure? a. "The study will occur in the cardiology department and will last 2 hrs." b. "The sleep study will help determine the amount of sleep I need each night." c. "I will receive some sedation for the procedure and will need to have a ride home." d. "The test determines how deeply and what type of sleep I have."

d. Explanation: The pt will be monitored for the stages of sleep. A sleep study determines if and how long the pt is in REM sleep

A nurse is caring for a patient who has undergone a flexible bronchoscopy for evaluation of a laryngeal tumor. Which of the following symptoms, if noted, should be reported immediately to the provider? a. Blood-tinged sputum b. Hematuria c. Hacking cough d. Wheezing

d. Explanation: Wheezing indicated narrowing of the airways which could result in hypoxia.

Pts with TB can be characterized as which type of infection? a. An LTBI b. A PTBI c. A symptomatic, noncontagious TB infection d. A symptomatic TB infection

d. Explanation: a TB pt who has a symptomatic TB infection as evidenced by his productive cough, fever, and fatigue. A latent TB infection has no symptoms and is not contagious. Asymptomatic infection is contagious.

The nurse is caring for a patient on a heparin drip who was admitted for DVT 2 days ago. Which laboratory value is most important to report to the provider immediately? a. A normal INR b. An increases hematocrit c. An increase platelet count d. A normal aPTT

d. Explanation: aPTT is used to monitor heparin effectiveness. A normal value indicates the need to increase the dose.


Conjuntos de estudio relacionados

Accounting Chapter 12 Vocabulary

View Set

Bates' Advanced Health Assessment Chapters 1-4, & 7 (Health History, HEENT)

View Set

TestOut Security Pro Semester 1 Review (Thorough version)

View Set

Microbiology by Body System - Bauman - Ch 13 Objectives

View Set

Compensation: Chapter 8: Designing Pay Levels, Mix, and Pay Structures

View Set

ACC 205 Business Law NAU Final Exam Review

View Set

Exam Review for Chapter 14: Lymphatic system and Immunity

View Set

(Exam 2) Chapter 8 Study Questions

View Set